MATINFmatinf.upit.ro/MATINF1/RevistaMATINF.pdf · i c a adecvat a atestat a prin studiile de...

173
DEPARTAMENTUL MATEMATIC ˘ A-INFORMATIC ˘ A UNIVERSITATEA DIN PITES , TI MATINF PUBLICAT , IE BIANUAL ˘ A DE MATEMATIC ˘ AS , I INFORMATIC ˘ A PENTRU ELEVI S , I PROFESORI Anul I, nr. 1 AUGUST 2018 ISSN 2601-9426 ISSN-L 2601-8829 Editura Universit˘at , ii din Pites , ti

Transcript of MATINFmatinf.upit.ro/MATINF1/RevistaMATINF.pdf · i c a adecvat a atestat a prin studiile de...

DEPARTAMENTULMATEMATICA-INFORMATICAUNIVERSITATEA DIN PITES, TI

MATINFPUBLICAT, IE BIANUALA

DE MATEMATICA S, I INFORMATICAPENTRU ELEVI S, I PROFESORI

Anul I, nr. 1AUGUST 2018

ISSN 2601-9426ISSN-L 2601-8829

EdituraUniversitat,ii din Pites, ti

Editata de: DEPARTAMENTUL MATEMATICA-INFORMATICA,UNIVERSITATEA DIN PITES, TI

Comitetul de redact, ie:

Stelian Corneliu ANDRONESCU Eduard ASADURIAN

Tudor BALANESCU Costel BALCAU - redactor s,ef

Loredana BALILESCU Doru CONSTANTINSerban COSTEA Laurent, iu DEACONU

Ionut, DINCA Mihaela DUMITRACHEMihai Armand IONESCU Florentin IPATEConstantin GEORGESCU Raluca Mihaela GEORGESCUCamelia GHELDIU Marius MACARIEMaria MIROIU Emil MOLDOVEANU

Gheorghe NISTOR Antonio Mihail NUICA

Viorel PAUN Doru Anastasiu POPESCU

Marin POPESCU Doru Nicolae STANESCUAlina Florentina S, TEFAN Cristina TUDOSEAdrian TURCANU Corneliu UDREA

Tehnoredactare computerizata: Mihail TANASE, e-mail: [email protected]

Redact, ia: Departamentul Matematica-Informatica, Universitatea din Pites,ti., Str. Targudin Vale, nr. 1, Pites,ti, tel. 0348453247, e-mail: [email protected]

Forma digitala a revistei poate fi accesata la adresa: http://matinf.upit.ro

Publicata de: Editura Universitat, ii din Pites,ti, https://www.upit.ro/ro/relatii-cu-mediul-socio-economic/centre-suport/editura

Anul I, Nr. 1, August 2018

Cuprins 3

Cuprins

DIN ACTIVITATEA DEPARTAMENTULUI 5

D. Constantin

Prezentarea Departamentului Matematica-Informatica . . . . . . . . . . . . . . . . . . . 5

D.A. Popescu, C. Balcau, D. Constantin

Prezentarea Concursului de Informatica ,,Programming Day for High School”, Edit, ia I,Pites,ti, 27 mai 2017 . . . . . . . . . . . . . . . . . . . . . . . . . . . . . . . . . . . . . . 10

ARTICOLE S, I NOTE DE MATEMATICA 21

V. Alexandru, S.C. Andronescu

Numere prime de forma p = x2 +My2 cu x, y ∈ Z s, i M ∈ 1, 2, 3 . . . . . . . . . . . . . 21

M. Chirciu

In legatura cu o inegalitate din ,,Mathematical Reflections” nr. 6/2015 . . . . . . . . . . 25

L.M. Giugiuc, D. Trailescu Gogan

Coordonate analitice practice . . . . . . . . . . . . . . . . . . . . . . . . . . . . . . . . . 27

M. Ionescu

Asupra unei probleme propuse la Olimpiada Nat, ionala de Matematica . . . . . . . . . . 31

F. Stanescu

Probleme de calcul integral. Inegalitat, i integrale I . . . . . . . . . . . . . . . . . . . . . . 33

ARTICOLE S, I NOTE DE INFORMATICA 39

C. Balcau

Optimalitatea algoritmului de cautare binara . . . . . . . . . . . . . . . . . . . . . . . . 39

ROWEB Development

Perspective profesionale pentru pasionat, ii de programare . . . . . . . . . . . . . . . . . . 52

MATEMATICA PENTRU PROGRAMATORI S, I PROGRAMARE PENTRU MATE-MATICIENI 55

L. Deaconu

Rezolvarea sistemelor de ecuat, ii liniare . . . . . . . . . . . . . . . . . . . . . . . . . . . . 55

R.M. Georgescu

Determinarea pozit, iilor relative a mai multor drepte ın plan cu ajutorul mediului Maple . 65

RUBRICA DE ROBOTICA 70

D.A. Popescu

Programarea robot, ilor LEGO folosind mediul Mindstorms EV3 . . . . . . . . . . . . . . 70

4 Cuprins

PROBLEME DE MATEMATICA PENTRU EXAMENE 79

Teste pentru examenul de Evaluare Nat, ionala . . . . . . . . . . . . . . . . . . . . . . . 79

Teste pentru examenul de Bacalaureat, specializarea S, tiint,e ale naturii . . . . . . . . . . 86

Teste pentru examenul de Bacalaureat, specializarea Matematica-Informatica . . . . . . 91

Teste pentru admiterea la facultate . . . . . . . . . . . . . . . . . . . . . . . . . . . . . 97

Teste grila pentru admiterea la facultate . . . . . . . . . . . . . . . . . . . . . . . . . . 100

PROBLEME DE INFORMATICA PENTRU EXAMENE 110

Teste pentru examenul de Bacalaureat, specializarea S, tiint,e ale naturii . . . . . . . . . . 110

Teste pentru examenul de Bacalaureat, specializarea Matematica-Informatica . . . . . . 120

Teste pentru admiterea la facultate . . . . . . . . . . . . . . . . . . . . . . . . . . . . . 135

Teste grila pentru admiterea la facultate . . . . . . . . . . . . . . . . . . . . . . . . . . 136

PROBLEME DE MATEMATICA PENTRU CONCURSURI 143

Probleme propuse . . . . . . . . . . . . . . . . . . . . . . . . . . . . . . . . . . . . . . . 143

PROBLEME DE INFORMATICA PENTRU CONCURSURI 150

Probleme propuse . . . . . . . . . . . . . . . . . . . . . . . . . . . . . . . . . . . . . . . 150

ISTORIOARE DIN LUMEA MATEMATICII S, I A INFORMATICII 172

S.C. Andronescu

Formula de alungare a lui L. Euler . . . . . . . . . . . . . . . . . . . . . . . . . . . . . . 172

DIN ACTIVITATEA DEPARTAMENTULUI

Prezentarea Departamentului Matematica-Informatica

Doru Constantin 1

Invat, amantul superior din Pites,ti a debutat ın anul 1962 prin ınfiint,area Institutului Pedago-gic din Pites,ti. Din anul 1974 Institutul Pedagogic s-a transformat ın Institutul de Invat, amantSuperior Pites,ti, iar ın cadrul acestuia a funct, ionat specializarea Matematica.

Universitatea din Pites,ti, ca institut, ie de ınvat, amant superior cu statut juridic propriu, a fostınfiint,ata prin ordinul Ministerului Invat, amantului nr. 4844 / 22.03.1991. In cadrul Facultat, iide S, tiint,e au funct, ionat specializarile Matematica s, i Matematica-Informatica, iar din anul 2002s-a acreditat programul de studii universitare de licent, a Informatica.

Facultatea de Matematica-Informatica a fost ınfiint,ata ın 2002, iar din 2016 ın urma procesuluide reorganizare administrativa a Universitat, ii din Pites,ti, ın conformitate cu H.G. 654/2016, aformat ımpreuna cu entitat, ile anterioare reprezentate de Facultatea de S, tiint,e s, i de Facultateade Educat, ie Fizica s, i Sport, facultatea actuala denumita Facultatea de S, tiint,e, Educat, ie Fizicas, i Informatica.

Facultatea de S, tiint,e, Educat, ie Fizica s, i Informatica (F.S, .E.F.I.) este ın prezent alcatuitadin 5 departamente (Matematica-Informatica, S, tiint,e ale Naturii, Ingineria Mediului s, i S, tiint,eIngineres, ti Aplicate, Asistent, a Medicala s, i Kinetoterapie, Educat,ie Fizica s, i Sport) la carepredau peste 130 de cadre didactice titulare s, i asociate.

Facultatea de S, tiint,e, Educat, ie Fizica s, i Informatica organizeaza, coordoneaza s, i verificaactivitatea de producere, de transmitere s, i de valorificare a cunoas,terii ın 13 programe de studiiuniversitare de licent, a, 10 programe de studii universitare de masterat, 4 programe de studiiuniversitare de doctorat s, i 4 programe de studii postuniversitare de conversie profesionala activeın anul universitar curent. De asemenea, ın cadrul facultat, ii sunt acreditate 2 programe destudii universitare de masterat cu predare ın limba straina (un program de studii universitarede masterat ın limba engleza ın domeniul Informatica s, i unul ın limba franceza ın domeniulEducat,ie Fizica s, i Sport).

Departamentul Matematica-Informatica (D.M.I.) ofera posibilitatea dezvoltarii s,tiint, ificela nivel superior prin oferta educat, ionala ce cuprinde: 2 programe de studii universitare delicent, a (ın domeniile Informatica s, i, respectiv, Matematica), 4 programe de studii universitarede masterat (ın domeniile Informatica s, i, respectiv, Matematica), precum s, i 2 programe destudii universitare de doctorat (ın domeniile Informatica s, i, respectiv, Matematica).

In Departamentul Matematica-Informatica activitat, ile didactice de specialitate din cadrulprogramelor de studii universitare de licent, a, masterat s, i doctorat ın domeniile Matematica s, iInformatica sunt acoperite de peste 30 de cadre didactice titulare s, i asociate care au pregatirea

1Conf. univ. dr., Universitatea din Pites,ti, [email protected]

5

6 D. Constantin

s,tiint, ifica adecvata atestata prin studiile de licent, a s, i/sau doctorat ın domeniile disciplinelorpredate.

Programele de studii universitare de licent, a, masterat s, i doctorat gestionate, ın prezent, deDepartamentul Matematica-Informatica sunt sintetizate ın urmatorul tabel.

1. Domenii de licent, a s, i programe de studii universitare de licent, aDomeniul de

licent, aProgramul de

studiiStatutul

programuluiCifra Aracis des, colarizare

Informatica Informatica Acreditat 100Matematica Matematica Acreditat 30

2. Programele de studii universitare de masterat

Programul de studiiStatutul

programuluiCifra Aracis des, colarizare

Tehnici avansate pentru prelucrareainformat, iei (Advanced Techniques for

Information Processing)Acreditat 50

Modelarea, proiectarea si managementulsistemelor software Acreditat 50

Matematica aplicata Acreditat 50Matematica didactica Acreditat 50

3. Programele de studii universitare de doctoratProgramul de studii S, coala Doctorala

Informatica S, coala doctorala de InformaticaMatematica S, coala doctorala de Matematica

In concordant, a cu misiunea Universitat, ii, prezentata ın Carta Universitat,ii din Pites, tis, i cu solicitarile actuale de pe piat,a fort,ei de munca regionale s, i nat, ionale, DepartamentulMatematica-Informatica din cadrul Facultatat, ii de S, tiint,e, Educat, ie Fizica s, i Informatica areurmatoarele misiuni didactice s, i de cercetare s,tiint, ifica:

1. asigurarea pentru fiecare program de studiu a unor planuri de ınvat, amant s, i programeanalitice adecvate, care sa permita recunoas,terea competent,elor obt, inute, atat ın t,ara cats, i ın strainatate;

2. pregatirea de specialis,ti de performant, a care sa fie capabili a asigura dezvoltarea societat, iiinformat, ionale din Romania ın contextul integrarii t, arii noastre ın Comunitatea Europeana;

3. participarea la programe europene de mobilitate a student, ilor s, i a cadrelor didactice;4. formarea de specialis,ti cu ınalta calificare ın domeniile informatica s, i matematica, la un

nivel compatibil cu cel din UE, cat s, i cadre didactice necesare ınvat, amantului nat, ional s, iinternat, ional ın domeniile matematica s, i informatica;

5. pregatirea psiho-pedagogica s, i metodica adecvata pentru exercitarea funct, iei de profesorın ınvat, amantul preuniversitar s, i universitar;

6. formarea de specialis,ti ın domeniile matematica s, i informatica, profesori de matematica,profesori de informatica, matematicieni, informaticieni, etc;

7. asigurarea pregatirii continue ın domeniile matematica s, i informatica;8. efectuarea de servicii sociale s, i culturale pentru membrii comunitat, ii academice s, i din afara

ei, prin dezvoltarea s, i difuzarea valorilor culturale, s,tiint, ifice, realizarea echilibrului necesarıntre cultura s,tiint, ifica, tehnologica s, i umanista, schimbarea mentalitat, ilor s, i atitudinilor,promovarea relat, iilor internat, ionale;

9. valorificarea potent, ialului s,tiint, ific pe care-l reprezinta cadrele didactice ale departamen-

Prezentarea Departamentului Matematica-Informatica 7

tului, ın activitatea de cercetare s,tiint, ifica s, i de publicare, prin prezentarea rezultatelorcercetarii s, i a experient,ei s,tiint, ifice acumulate prin participarea la conferint,e nat, ionale s, iinternat, ionale de specialitate.

In acest context, programele de studii universitare din domeniile Matematica s, i Informaticacontribuie la ındeplinirea misiunii facultat, ii s, i aduce elemente de specificitate s, i oportunitate,dupa cum rezulta din obiectivele generale s, i din obiectivele specifice ale fiecarui program destudiu.

Obiectivele generale al programelor de studii universitare se pot sintetiza astfel:

1. asigurarea unei pregatiri de ınalt nivel a student, ilor ın domeniile matematica s, i informatica,astfel ıncat aces,tia sa poata ocupa un loc de munca ın ınvat, amant (profesor de matematica,informatica sau tehnologia informat, iei), ın mediul socio-economic (analist programator,administrator de ret,ele, proiectant sisteme informatice, statistician etc.) sau sa fie capabilia continua studiile ın cadrul masteratelor din universitatea noastra sau din alte universitat, i(din t,ara sau din strainatate);

2. formarea de specialis,ti cu ınalta calificare, cu competent,e bine definite ın domeniultehnologiilor informat, ionale s, i al sistemelor informatice, pentru mediul de afaceri, ın toatetipurile de activitat, i economico-sociale, la un nivel compatibil cu cel din UE, cat s, i cadrenecesare ınvat, amantului;

3. adaptarea ofertei educat, ionale la evolut, ia structurala a mediului economic romanesc;4. integrarea ın ret,ele universitare nat, ionale s, i internat, ionale pe criterii de performant, a s, i

complementaritate;5. sust, inerea accesului absolvent, ilor la oportunitat, ile existente pe piat,a muncii nat, ionale s, i

europene;6. prestarea de activitat, i specifice solicitate de beneficiari ın condit, ii comerciale competitive.

La nivelul Departamentului Matematica-Informatica funct, ioneaza Centrul de Cercetare,,Modele de calcul si metodologii de programare” (MCMP). Planul de cercetare alCentrului de Cercetare include o serie de obiective s, i de activitat, i, structurate pe direct, ii decercetare, teme de cercetare s, i colective de cadre didactice.

Obiectivele principale ale planului de cercetare vizeaza ımbunatat, irea calitat, ii activitat, ii ıncercetare prin:

- eficientizarea activitat, ii de cercetare s,tiint, ifica a cadrelor didactice din DepartamentulMatematica-Informatica, prin orientarea activitat, ilor specifice pe direct, iile prioritare stabilite lanivel nat, ional;

- realizarea unor programe de cercetare cu implicat, ii directe asupra modernizarii s, i eficien-tizarii bazei materiale a facultat, ii, prin participarea la competit, iile de proiecte de cercetarenat, ionale s, i internat, ionale;

- implicarea unui numar tot mai mare de student, i, masteranzi s, i doctoranzi ın activitatea decercetare;

- identificarea s, i realizarea unor noi colaborari cu firme software sau alt, i parteneri din mediulsocio-economic ın vederea valorificarii potent, ialului de cercetare;

- realizarea unor colective de cercetare interdisciplinara, ın colaborare cu alte institut, ii de peplan nat, ional s, i internat, ional.

8 D. Constantin

Activitatea de cercetare a cadrelor didactice ale Departamentului Matematica-Informaticase desfas,oara ın urmatoarele domenii de cercetare:

1. Aplicat, ii WEB s, i E-learning;2. Modele de calcul s, i metodologii de programare;3. Teoria informat, iei s, i sisteme avansate de baze de date;4. Inteligent, a artificiala s, i modele matematice cu aplicat, ii ın informatica;5. Recunoas,terea formelor s, i ret,ele neuronale;6. Verificarea s, i testarea sistemelor de programe, specificarea s, i proiectarea sistemelor software

complexe;7. Algebra s, i geometrie computat, ionala, calcul numeric, optimizari, teoria codurilor s, i

criptografie;8. Metode numerice ın mecanica fluidelor s, i solidelor, sisteme dinamice, teoria bifurcat, iei,

ecuat, ii diferent, iale;9. Mecanica mediilor continue, calcul optimal, metode numerice pentru ecuat, ii cu derivate

part, iale, teoria omogenizarii;10. Teoria potent, ialului, procese stocastice, analiza neliniara s, i operatori;11. Teoria geometrica a funct, iilor univalente.

Anual, Departamentul Matematica-Informatica organizeaza Sesiunea de comunicari s, tiint,ificestudent,es, ti (SCSS), la care participa un numar mare de student, i. Departamentul Matematica-Informatica coordoneaza sect, iunile aferente domeniilor Informatica s, i Matematica astfel:

- Informatica (la care participa ın special student, i de la programul de studii universitare delicent, a Informatica s, i de la programele de studii universitare de masterat Advanced Techniquesfor Information Processing s, i Modelarea, Proiectarea s, i Managementul Sistemelor Software);

- Informatica aplicata (la care participa ın special student, i de la programul de studiiuniversitare de licent, a Informatica s, i de la programele de studii universitare de masteratAdvanced Techniques for Information Processing s, i Modelarea, Proiectarea s, i ManagementulSistemelor Software);

- Matematica (la care participa ın special student, i de la programul de studii universitare delicent, a Matematica s, i de la programul de studii universitare de masterat Matematica Didactica);

- Matematica aplicata (la care participa ın special student, i de la programul de studiiuniversitare de licent, a Matematica s, i de la programul de studii universitare de masteratMatematica Aplicata).

Tot anual, departamentul organizeaza sesiunea de comunicari s,tiint, ifice pentru doctoranzi(The Doctoral Student Workshop). Sesiunea s,tiint, ifica dedicata doctoranzilor cuprinde douasect, iuni astfel:

- The Doctoral Student Workshop in Computer Science (la care participa ın special student, iide la programele de studii universitare de doctorat ın Informatica de la S, coala Doctorala deInformatica a Universitat, ii din Pites,ti, dar s, i din alte centre universitare);

- The Doctoral Student Workshop in Mathematics (la care participa ın special student, iide la programele de studii universitare de doctorat ın Matematica de la S, coala Doctorala deMatematica a Universitat, ii din Pites,ti, dar s, i din alte centre universitare).

Incepand din anul 2017 Departamentul Matematica-Informatica organizeaza anual eveni-mentul ,,Programming Day for High School” - concurs de programare C/C++ pentru

Prezentarea Departamentului Matematica-Informatica 9

elevi cu 4 sect, iuni corespunzatoare claselor a IX-a, a X-a, a XI-a s, i a XII-a s, i programarearobot,ilor LEGO Mindstorms EV3, la care participa un numar mare de elevi provenit, i de laliceele reprezentative din judet,ul Arges, dar s, i din judet,ele ınvecinate.

Tot ıncepand din anul 2016 Departamentul Matematica-Informatica organizeaza anualevenimentul ,,WAC&WS - WEB Applications Contest and Workshop for Students” -concurs de programare WEB s, i workshop WEB pentru student,i, cu participarea unui numar marede student, i de la programele de specialitate din cadrul Departamentului Matematica-Informatica,din universitate s, i din alte universitat, i.

Departamentul Matematica-Informatica implementeaza misiunea Facultat, ii de S, tiint,e, Edu-cat, ie Fizica s, i Informatica de dezvoltare ca entitate academica de elita la nivel regional s, inat, ional care sprijina activitat, ile de cercetare, de cres,tere a calitat, ii s, i eficient,ei proceselor deınvat, amant, de relat, ionare cu comunitatea academica s, i cu cea a mediului socio-economic dinregiune.

Prezentarea Concursului de Informatica ,,Programming

Day for High School”, Edit, ia I, Pites,ti, 27 mai 2017

Doru Anastasiu Popescu 1, Costel Balcau 2 s, i Doru Constantin 3

Departamentul Matematica-Informaticaal Universitat, ii din Pites,ti a organizat, ındata de 27 mai 2017, prima edit, ie a concur-sului de programare clasica (folosind limba-jele Pascal/C/C++, mediul de programareCode::Blocks) s, i de programarea robot, ilorLEGO (folosind mediul Mindstorms EV3) cunumele ,,Programming Day for High School”.La aceasta edit, ie au participat elevi de launitat, i s,colare din judet,ele Arges, s, i Olt.

Premiile cas,tigatorilor au fost sponsorizate de firma ROWEB Development, director generalViorel Costea (profesor de Informatica) s, i de firma Velox Logistics Center, director economicAdriana Neaga.

Concursul de programare clasica

Acesta a fost organizat pe trei sect, iuni, corespunzatoare clasei a IX-a, clasei a X-a s, i claselor aXI-a s, i a XII-a. Concurent, ii au avut de rezolvat cate doua probleme, fiecare avand un punctajde 100 de puncte. In continuare prezentam problemele date ın concurs, ımpreuna cu indicat, ii derezolvare.

Clasa a IX-a

Problema 1 – pasiVasile contorizeaza pas, ii facut, i de acasa pana la s,coala s, i observa ca ın unele zile numarulacestora verifica anumite proprietat, i. In unele zile este un produs de exact trei numere prime(numit 3-prim), alteori este factorialul unui numar (numit factorial), iar alteori este cubperfect (numit cub).

Cerint, aCunoscand numarul de zile N s, i numarul de pas, i din fiecare zi x1, x2, ..., xN se cere:

1. numarul de numere 3-prim;

1Conf. univ. dr., Universitatea din Pites,ti, [email protected]. univ. dr., Universitatea din Pites,ti, [email protected]. univ. dr., Universitatea din Pites,ti, [email protected]

10

Concursul de Informatica Programming Day 11

2. numarul de numere factorial;3. numarul de numere cub.

Date de intrareFis, ierul de intrare pasi.in cont, ine pe prima linie un numar natural p. Pentru toate testele deintrare, numarul p poate avea doar valoarea 1, 2 sau 3.

Pe linia a doua se afla N , iar pe linia a treia se afla numerele x1, x2, . . . , xN separate princate un spat, iu, reprezentand numarul de pas, i pana la s,coala ın fiecare din cele N zile.

Date de ies, ireDaca valoarea lui p este 1, se va rezolva numai punctul 1) din cerint, a.

In acest caz, ın fis, ierul de ies, ire pasi.out se va scrie un singur numar natural reprezentandnumarul de numere 3-prim din s, irul x1, x2, . . . , xN .

Daca valoarea lui p este 2, se va rezolva numai punctul 2) din cerint, a.

In acest caz, ın fis, ierul de ies, ire pasi.out se va scrie un singur numar natural reprezentandnumarul de numere factorial din s, irul x1, x2, . . . , xN .

Daca valoarea lui p este 3, se va rezolva numai punctul 3) din cerint, a.

In acest caz, ın fis, ierul de ies, ire pasi.out se va scrie un singur numar natural reprezentandnumarul de numere cub din s, irul x1, x2, . . . , xN .

Restrict, ii s, i precizari

• 1≤N≤100000

• 1≤x1, x2, . . . , xN≤10000

• Pentru rezolvarea corecta a cerint,ei 1 se acorda 40% din punctaj, a cerint,ei 2 se acorda30% din punctaj s, i a cerint,ei 3 se acorda 30% din punctaj

• Pentru 30% din teste N≤1000

Exemplepasi.in pasi.out Explicat, ie1366 11 30

2 p = 1In s, irul dat exista doua numere 3-prim s, ianume: 66 = 2·3·11, 30 = 2·3·5.

pasi.in pasi.out Explicat, ie2366 24 30

1 p = 2In s, irul dat exista un numar factorial s, ianume: 24 = 1·2·3·4.

pasi.in pasi.out Explicat, ie3327 11 125

2 p = 3In s, irul dat exista doua numere cub s, ianume: 27 = 33, 125 = 53.

Timp maxim de execut, ie: 0.3 secunde/test.

Memorie totala disponibila 4 MB, din care 2 MB pentru stiva.

Doru Anastasiu Popescu, Costel Balcau, Pites,ti, Gabriel Boroghina, Bucures,ti

12 D.A. Popescu, C. Balcau, D. Constantin

Solut, ie de 30 de puncte

1. Se citesc p s, i N .

2. Nr = 0.

3. Pentru fiecare numar din s, ir, ın funct, ie de valoarea lui p (1, 2, 3) se verifica dacaındeplines,te una din condit, iile de 3-prim, factorial sau cub. Daca este ındeplinita condit, ia,atunci Nr = Nr + 1.

4. Se afis,eaza Nr.

Solut, ie de 100 de puncte

1. Se construiesc trei tablouri unidimensionale a, b, c de dimensiune 10000, unde:

a[i] = 1, daca i este 3-prim s, i 0 ın caz contrar,

b[i] = 1, daca i este factorial s, i 0 ın caz contrar,

c[i] = 1, daca i este cub s, i 0 ın caz contrar,

unde i ia valorile 1, 2, ..., 10000.

2. Se citesc p s, i N .

3. Nr = 0.

4. Pentru fiecare numar din s, ir, memorat ın variabila k, ın funct, ie de valoarea lui p (1, 2, 3),se verifica daca ındeplines,te una din condit, iile a[k] == 1 (3-prim), b[k] == 1 (factorial) sauc[k] == 1 (cub). Daca este ındeplinita condit, ia, atunci Nr = Nr + 1.

5. Se afis,eaza Nr.

Problema 2 – continentPe un continent se dores,te organizarea unei olimpiade de robotica. Pentru a stabili ordinea dealegere a t, arilor organizatoare pe ani calendaristici se stabiles,te drept criteriu suprafat,a. Astfel,o t,ara care are o suprafat, a mai mare va organiza mai repede o edit, ie de olimpiada.

Pentru stabilirea suprafet,elor t, arilor continentului se foloses,te o harta data folosind untablou bidimensional cu M linii s, i N coloane. Un element din harta reprezinta 1 km2 din teren.Elementele tabloului sunt numere consecutive 0, 1, 2, . . .. T, arile sunt codificate cu 1, 2, . . . . Cifra0 ın tablou reprezinta teren sau apa care nu apart, in niciunei t, ari.

Daca exista mai multe t, ari cu aceeas, i suprafat, a, cea care are codul asociat mai mic vaorganiza prima o edit, ie de olimpiada.

Cerint, aCunoscand numerele M,N s, i tabloul ce codifica harta, se cere:

1. suprafat,a t, arii cu codul cel mai mare;

2. codurile t, arilor ın ordinea descrescatoare a suprafet,ei.

Date de intrareFis, ierul de intrare continent.in cont, ine pe prima linie un numar natural p. Pentru toatetestele de intrare, numarul p poate avea doar valoarea 1 sau 2.

Concursul de Informatica Programming Day 13

Pe linia a doua se afla M,N , separate printr-un spat, iu, iar pe liniile urmatoare tabloul cecodifica harta, pe fiecare linie componentele sunt separate prin cate un spat, iu.

Date de ies, ireDaca valoarea lui p este 1, se va rezolva numai punctul 1) din cerint, a.

In acest caz, ın fis, ierul de ies, ire continent.out se va scrie un numar ce reprezinta suprafat,at, arii cu codul cel mai mare

Daca valoarea lui p este 2, se va rezolva numai punctul 2) din cerint, a.

In acest caz, ın fis, ierul de ies, ire continent.out se vor scrie codurile t, arilor ın ordineaorganizarii olimpiadelor, cate unul pe un rand.

Restrict, ii s, i precizari

• 1≤M,N≤300

• 0≤ coduri t, ari ≤M ·N

• Pentru rezolvarea corecta a fiecarei cerint,e se acorda 50% din punctaj

Exemplecontinent.in continent.out Explicat, ie13 44 0 1 14 4 0 12 1 3 3

3 p = 1Pe continent exista 4 t, ari. T, ara cu codul celmai mare, 4, are suprafat,a egala cu 3.

continent.in continent.out Explicat, ie23 44 0 1 14 4 3 12 4 3 3

4132

p = 2Cele patru t, ari de pe continent codificate cu1, 2, 3, 4 au suprafet,ele: 3, 1, 3, respectiv 4.Ordinea de desfas,urare a olimpiadelor este 4,1, 3, 2.

Timp maxim de execut, ie: 0.2 secunde/test.

Memorie totala disponibila 2 MB, din care 1 MB pentru stiva.

Doru Anastasiu Popescu, Costel Balcau, Pites,ti, Gabriel Boroghina, Bucures,ti

Solut, ie cerint,a 1Se determina cel mai mare numar din tablou (notat cu max) s, i frecvent,a sa F . Nu trebuiememorat tabloul. Valoarea lui F se va afis,a.

Solut, ie cerint,a 2Se determina cel mai mare numar din tablou (notat cu max). Pentru fiecare numar i =1, 2, ...,max se determina de cate ori apare ın tablou: x[i]. Se noteaza cu o[i] = i, ordineainit, iala a t, arilor, dupa care se ordoneaza descrescator tabloul unidimensional x s, i se interschimbaelementele lui o odata cu cele ale lui x. Apoi se afis,eaza elementele tabloului o.

14 D.A. Popescu, C. Balcau, D. Constantin

Clasa a X-a

Problema 1 – anagramaIonica trebuie sa rezolve o problema de informatica ce prelucreaza texte. Mai precis, are ladispozit, ie un cuvant C format din litere mici s, i un text T format din litere mici s, i spat, ii.Cuvintele ın T sunt separate prin unul sau mai multe spat, ii. Problema pe care trebuie sa orezolve Ionica este de a determina numarul de vocale comune lui C s, i primului cuvant din T s, iapoi numarul de cuvinte anagrama ale lui C ın T . Un cuvant X este anagrama pentru un altcuvant Y daca X este format din exact literele lui Y , eventual ın alta ordine.

Cerint, aCunoscand C s, i T se cere:

1. numarul de vocale comune din C s, i din primul cuvant din T ;

2. numarul de cuvinte anagrama ale lui C ın T .

Date de intrareFis, ierul de intrare anagrama.in cont, ine pe prima linie un numar natural p. Pentru toate testelede intrare, numarul p poate avea doar valoarea 1 sau 2.

Pe linia a doua se afla C, iar pe linia a treia se afla T . Fiecare linie se termina cu ,,Enter”.

Date de ies, ireDaca valoarea lui p este 1, se va rezolva numai punctul 1) din cerint, a.

In acest caz, ın fis, ierul de ies, ire anagrama.out se va scrie un singur numar natural repre-zentand numarul de vocale comune din C s, i din primul cuvant din T .

Daca valoarea lui p este 2, se va rezolva numai punctul 2) din cerint, a.

In acest caz, ın fis, ierul de ies, ire anagrama.out se va scrie un singur numar natural repre-zentand numarul de cuvinte anagrama ale lui C ın T .

Restrict, ii s, i precizari

• 1 ≤ lungimea lui C≤ 100000

• 1 ≤ lungimea lui T≤ 100000

• Vocalele sunt a, e, i, o, u

• Pentru rezolvarea corecta a fiecarei cerint,e se acorda 50% din punctaj

Exempleanagrama.in anagrama.out Explicat, ie1mariamarinela se joaca

2 p = 1C este mariaPrimul cuvant din T estemarinelaSunt 2 vocale comune. aapare de doua ori, dar estenumarat o singura data.

Concursul de Informatica Programming Day 15

anagrama.in anagrama.out Explicat, ie2maravara aram dica rama aarm rama

4 p = 2Cuvintele anagrama cu C ınT sunt aram, rama, aarm s, irama. Numarul lor este 4.

Timp maxim de execut, ie: 0.2 secunde/test.

Memorie totala disponibila 2 MB, din care 1 MB pentru stiva.

Doru Anastasiu Popescu, Costel Balcau, Pites,ti, Gabriel Boroghina, Bucures,ti

Solut, ie

Cerint,a 1

1. Se citesc C s, i T .

2. Nr = 0.

3. Se determina ın i pozit, ia primului spat, iu ın T . Daca T are un singur cuvant, atunci ieste numarul de litere.

4. Pentru fiecare vocala se verifica daca se gases,te atat ın C, cat s, i ın T 0T 1 . . . T i−1. In cazafirmativ Nr = Nr + 1.

5. Se afis,eaza Nr.

Cerint,a 2

1. Se construies,te vectorul de frecvent,e al literelor mici pentru cuvantul C, fie acestax = (x0, x1, . . . , x25), xi este numarul de aparit, ii a celei de-a i− a litera din alfabet ın C.

2. Nr = 0.

3. Pentru fiecare cuvant din T se determina vectorul de frecvent,e al literelor mici: y =(y0, y1, . . . , y25). Daca x s, i y au aceleas, i componente atunci Nr = Nr + 1.

4. Se afis,eaza Nr.

Problema 2 – cuburiIonica are un sac plin cu cuburi, numerotate cu 1, 2, . . . , N s, i M cutii numerotate cu 1, 2, . . . ,Mın care ıncap x1, x2, . . . , xM cuburi. Din pacate cutiile nu sunt suficiente pentru a introducecuburile din sac s, i mai raman cuburi neintroduse. In fiecare zi dupa ce se joaca cu cuburile leintroduce ın cutii la capacitate maxima s, i ce ramane le pune ın sac. Dupa multe zile de joaca s, ias,ezare a cuburilor ın cutii, Ionica se ıntreba ın cate moduri diferite poate as,eza cuburile ıncutii, s,tiind ca ordinea cuburilor dintr-o cutie nu are important, a.

Cerint, aCunoscand numerele N,M s, i capacitat, ile cutiilor, se cere:

1. numarul maxim de cuburi pe care le poate pune ın [M/2] cutii;

2. ın cate moduri diferite se pot as,eza cuburile ın cutii, s,tiind ca ordinea cuburilor ın cutiinu are important, a.

Date de intrareFis, ierul de intrare cuburi.in cont, ine pe prima linie un numar natural p. Pentru toate testelede intrare, numarul p poate avea doar valoarea 1 sau 2.

16 D.A. Popescu, C. Balcau, D. Constantin

Pe linia a doua se afla N,M , separate printr-un spat, iu, iar pe linia urmatoare numerelex1, x2, . . . , xM separate prin cate un spat, iu.

Date de ies, ireDaca valoarea lui p este 1, se va rezolva numai punctul 1) din cerint, a.

In acest caz, ın fis, ierul de ies, ire cuburi.out se va scrie un numar ce reprezinta numarulmaxim de cuburi pe care le poate pune ın [M/2] cutii.

Daca valoarea lui p este 2, se va rezolva numai punctul 2) din cerint, a.

In acest caz, ın fis, ierul de ies, ire cuburi.out se va scrie numarul de moduri diferite ın carese pot as,eza cuburile ın cutii. Numarul va fi afis,at modulo 60013.

Restrict, ii s, i precizari

• 1≤N≤1000

• 1≤M≤1000

• 0≤x1 + x2 + ...+ xM≤N

• x modulo y reprezinta restul ımpart, irii lui x la y

• Pentru rezolvarea corecta a fiecarei cerint,e se acorda 50% din punctaj

Exemplecuburi.in cuburi.out Explicat, ie14 21 2

2 p = 1Cutia cea mai mare cont, ine 2 cu-buri.

cuburi.in cuburi.out Explicat, ie24 21 2

12 p = 2La cerint,a 2 numarul de modalitat, ide as,ezare a cuburilor este 12.

Timp maxim de execut, ie: 0.2 secunde/test.

Memorie totala disponibila 8 MB, din care 2 MB pentru stiva.

Doru Anastasiu Popescu, Costel Balcau, Pites,ti, Gabriel Boroghina, Bucures,ti

Solut, ie

Cerint,a 1

Se ordoneaza descrescator vectorul x s, i se calculeaza s = x1 + x2 + ... + x[M/2]. Apoi seafis,eaza s.

Cerint,a 2

Numarul cautat este C(N, x1) · C(N − x1, x2) · . . . · C(N − (x1 + ... + xM−1), xM) modulo60013. Combinarile din produs se calculeaza folosind triunghiul lui Pascal modulo 60013.

Concursul de Informatica Programming Day 17

Clasele a XI-a s, i a XII-a

Problema 1 – parantezeLa ora de informatica Georgica a ınvat,at sa scrie parantezarile corecte folosind N perechi deparanteze rotunde. O parantezare corecta respecta regulile din aritmetica. Adancimea uneiparantezari este numarul maxim de perechi de paranteze una ın alta. De exemplu, pentru N = 2,parantezarile corecte sunt ()(), (()). Prima are adancimea 1, iar a doua 2. Pentru numereleN,M s, i k, Georgica vrea sa determine numarul de parantezari cu M perechi de paranteze cuadancimea cel mult k, precum s, i numarul total de parantezari distincte cu 1, 2, ..., N perechi deparanteze.

Cerint, aCunoscand numerele N,M s, i k se cere:

1. numarul de parantezari cu M perechi de paranteze s, i adancimea cel mult k, modulo60013;

2. numarul total de parantezari distincte cu 1, 2, ..., N perechi de paranteze modulo 60013.

Date de intrareFis, ierul de intrare paranteze.in cont, ine pe prima linie un numar natural p. Pentru toatetestele de intrare, numarul p poate avea doar valoarea 1 sau 2.

Pe linia a doua se afla N,M s, i k cu spat, iu ıntre ele.

Date de ies, ireDaca valoarea lui p este 1, se va rezolva numai punctul 1) din cerint, a.

In acest caz, ın fis, ierul de ies, ire paranteze.out se va scrie un singur numar natural repre-zentand numarul de la cerint,a 1.

Daca valoarea lui p este 2, se va rezolva numai punctul 2) din cerint, a.

In acest caz, ın fis, ierul de ies, ire paranteze.out se va scrie un singur numar natural repre-zentand numarul de la cerint,a 2.

Restrict, ii s, i precizari

• 1≤N≤1000

• 1≤M≤15

• 1≤k≤M

• x modulo y reprezinta restul ımpart, irii lui x la y

• Pentru rezolvarea corecta a fiecarei cerint,e se acorda 50% din punctaj

Exempleparanteze.in paranteze.out Explicat, ie12 2 1

1 p = 1N = 2, M = 2, k = 1. Avem doua pa-rantezari ()(), (()). Prima are adancimea1, iar a doua adancimea 2, astfel senumara doar prima.

18 D.A. Popescu, C. Balcau, D. Constantin

paranteze.in paranteze.out Explicat, ie22 2 1

3 p = 2N = 2, M = 2, k = 1. Avem o paran-tezare cu o pereche de paranteze: () s, idoua parntezari cu doua perechi de pa-ranteze: ()(), (()). Total 3 parantezari.

Timp maxim de execut, ie: 0.5 secunde/test.

Memorie totala disponibila 32 MB, din care 4 MB pentru stiva.

Doru Constantin, Costel Balcau, Pites,ti, Gabriel Boroghina, Bucures,ti

Solut, ie

Cerint,a 1Se genereaza parantezarile de adancime cel mult k folosind metoda backtracking s, i se incre-meanteaza o variabila Nr. Apoi se afis,eaza Nr.

Cerint,a 2Numarul de parantezari cu n perechi de paranteze este numarul lui Catalan:

C(n) = comb(2n, n)/(n+ 1) = comb(2n, n)− comb(2n, n− 1).

Numarul cautat este Nr = (C(1) + C(2) + ...+ C(n)) modulo 60013.

Combinarile din suma se calculeaza cu triunghiul lui Pascal modulo 60013.

Problema 2 – grupuriLa sfars, itul clasei se contorizeaza situat, iile ın care doi elevi s-au ajutat reciproc. Elevii suntcodificat, i prin numerele 1, 2, ..., N . Sunt M perechi de elevi care s-au ajutat reciproc. Pentrufiecare astfel de pereche se cunoas,te numarul de situat, ii ın care s-au ajutat reciproc. In funct, iede aceste situat, ii s-au constituit grupuri de elevi. Doi elevi i, j fac parte din acelas, i grup dacaexista un s, ir de elevi x1, x2, . . . , xk cu x1 = i, xk = j s, i (x1, x2), (x2, x3), . . . , (xk−1, xk) s-auajutat reciproc cel put, in o data.

Cerint, aCunoscand numarul de elevi N , numarul M de perechi de elevi care s-au ajutat cel put, in odata, precum s, i perechile de elevi ımpreuna cu numarul de situat, ii ın care s-au ajutat, se cere:

1. numarul de grupuri;

2. valoarea maxima a numarului total de situat, ii de ajutor reciproc ıntr-un grup.

Date de intrareFis, ierul de intrare grupuri.in cont, ine pe prima linie un numar natural p. Pentru toate testelede intrare, numarul p poate avea doar valoarea 1 sau 2.Pe linia a doua se afla N,M cu spat, iu ıntre ele, iar pe urmatoarele M linii se afla triplete deforma i j k separate prin cate un spat, iu, reprezentand k situat, ii de ajutor reciproc ıntre elevii is, i j.

Date de ies, ireDaca valoarea lui p este 1, se va rezolva numai punctul 1) din cerint, a.

In acest caz, ın fis, ierul de ies, ire grupuri.out se va scrie un singur numar natural reprezentandnumarul de grupuri.

Daca valoarea lui p este 2, se va rezolva numai punctul 2) din cerint, a.

Concursul de Informatica Programming Day 19

In acest caz, ın fis, ierul de ies, ire grupuri.out se va scrie un singur numar natural reprezentandvaloarea maxima a numarului total de situat, ii de ajutor reciproc ıntr-un grup.

Restrict, ii s, i precizari

• 1≤N≤500

• 1≤M≤100000

• Pentru rezolvarea corecta a fiecarei cerint,e se acorda 50% din punctaj

Exemplegrupuri.in grupuri.out Explicat, ie15 31 4 52 3 103 5 15

2 p = 1Elevii sunt ımpart, it, i ın doua grupuri:Grupul 1 cu elevii 1 s, i 4;Grupul 2 cu elevii 2, 3, 5.

grupuri.in grupuri.out Explicat, ie25 31 4 52 3 103 5 15

25 p = 2Elevii sunt ımpart, it, i ın doua grupuri:Grupul 1 cu elevii 1 s, i 4 (total situatii de ajutor reciproc:5);Grupul 2 cu elevii 2, 3, 5 (total situat, ii de ajutor reciproc:10+15=25).Valoarea cea mai mare este 25.

Timp maxim de execut, ie: 0.1 secunde/test.

Memorie totala disponibila 8 MB, din care 2 MB pentru stiva.

Doru Constantin, Costel Balcau, Pites,ti, Gabriel Boroghina, Bucures,ti

Solut, ie

Cerint,a 1Se construies,te un graf neorientat ponderat ın care nodurile sunt elevii, iar o muchie are costulegal cu numarul de situat, ii ın care elevii asociat, i s-au ajutat reciproc. Pentru acest graf sedetermina numarul de componente conexe.

Cerint,a 2Se construies,te un graf neorientat ponderat ın care nodurile sunt elevii, iar o muchie are costulegal cu numarul de situat, ii ın care elevii asociat, i s-au ajutat reciproc. Pentru acest graf sedetermina componentele conexe: C1, C2, . . . , Ck. Apoi pentru fiecare componenta conexa Cise determina ın xi suma totala a costurilor muchiilor ei. Valoarea maxima din vectorul x estenumarul cautat.

Concursul de programare a robot, ilor LEGO

La acest concurs a fost propusa urmatoarea problema.

Problema – robotica

Punctaj maxim 120+20=140 puncte

20 D.A. Popescu, C. Balcau, D. Constantin

Durata deplasarii robotului: maxim 2 minute

Plans, a utilizata: WRO 2017, REGULAR CATEGORY, ELEMENTARY(http://www.wro2017.org/Regular Category/Elementary/WRO-2017-Regular-Category-Elementary-PrintV2.pdf)

Punct de plecare: patrat verde, cu robotul orientat ın orice direct, ie, dar cu toate rot, ile ınpatratul verde.

Activitate robot: sa atinga cat mai multe cuburi. Pentru atingerea unui cub indiferent dedimensiune se primesc 10 puncte (chiar daca a fost atins de mai multe ori).

Oprirea robotului:

1. Implicit dupa 2 minute.2. Cand unul din concurent, ii striga stop.3. Cand se opres,te robotul sau cand acesta parases,te tabla.4. Daca robotul se opres,te ın locul de unde a plecat (cu toate rot, ile, pe culoarea verde) atunci

echipa mai primes,te 20 puncte.5. Daca se pune mana pe robot.

Punctaj final:

Punctele se contorizeaza s, i la sfars, it constituie punctajul rundei.

O echipa are 4 runde (ıncercari pe plans, a), cel mai mare punctaj ımpreuna cu timpul ei va fifolosit la ıntocmirea clasamentului.

Masa

La masa acesul se face ın ordinea sosirii unui membru al echipei. Robotul se as,eaza lapunctul de plecare s, i se pornes,te doar cand un membru al juriului spune start, moment ın carese pornes,te cronometrul.

Nicolae Bold, Slatina

Cas, tigatorii concursului

Premiant, ii concursului de programare clasica, respectiv de programare a robot, ilor LEGOMindstorm EV3 au fost:

Sect, iunea A - Clasa a IX-a: Vadastreanu Robert de la S, c. Gimn. ,,Eugen Ionescu”Slatina – premiul I, Coneschi Vlad de la S, c. Gimn. ,,Eugen Ionescu” Slatina – premiul al II-lea,Stanciu Ciprian de la C.N. ,,Radu Greceanu” Slatina – premiul al III-lea;

Sect, iunea B - Clasa a X-a: Staicu Alexandru de la C.N. ,,Ion Minulescu” Slatina –premiul I, Mincu Daniel de la C.N. ,,Alexandru Odobescu” Pites,ti – premiul al II-lea, PopescuRazvan de la C.N. ,,Alexandru Odobescu” Pites,ti – premiul al III-lea;

Sect, iunea C - Clasele a XI-a s, i a XII-a: Popescu Ion Alexandru de la C.N. ,,RaduGreceanu” Slatina – premiul I, Dumitru Corneliu Vlad de la C.N. ,,Ion C. Bratianu” Pites,ti –premiul al II-lea, Cent,iu Emil Vasilian de la C.N. ,,Ion C. Bratianu” Pites,ti – premiul al III-lea;

Sect, iunea R - Programarea robot, ilor LEGO Mindstorm EV3: Belegan David &Nicola Alex de la C.N. ,,Ion C. Bratianu” Pites,ti – premiul I, Conseschi Vlad & VadastreanuRobert & Constantinescu Gabriel de la S, c. Gimn. ,,Eugen Ionescu” Slatina – premiul al II-lea,Popescu Ion Alexandru & Stanciu Ciprian de la C.N. ,,Radu Greceanu” Slatina – premiul alIII-lea.

ARTICOLE S, I NOTE DE MATEMATICA

Numere prime de forma p = x2 +My2 cu x, y ∈ Z s, i

M ∈ 1, 2, 3

Victor Alexandru 1 s, i Stelian Corneliu Andronescu 2

In acest articol vom prezenta un rezultat clasic asupra reprezentarii numerelor prime,ımpreuna cu cateva aplicat, ii. Primul fapt semnificativ ın aceasta direct, ie a fost descoperit de P.Fermat: numerele prime de forma 4k + 1 se pot reprezenta ca suma de doua patrate perfecte.

Problema reprezentarii numerelor prime sub forma x2 +My2, pentru M ∈ 1, 2, 3 a fostrezolvata de L. Euler.

Propozit, ia 1. Fie p ≥ 3 un numar prim cu (p,M) = 1 s, i astfel ıncat congruent,a x2 ≡−M(mod p) are solut,ie, iar M ∈ 1, 2, 3. Atunci p poate fi reprezentat sub forma x2 +My2

cu x, y ∈ Z.

Prezentam ın continuare o demonstrat, ie a lui L. Euler, care prezinta avantajul de a ficonstructiva.

Demonstratie. Deoarece −M este rest patratic modulo p, exista a, b ∈ N astfel ca p|a2 +Mb2,

iar (p, ab) = 1. Putem alege a, b cu |a| < p

2, |b| < p

2, de unde a2 + Mb2 <

p2

4+ M

p2

4≤ p2.

Prin urmare avem a2 + Mb2 = p · q, cu 1 ≤ q < p. Daca q 6= 1, putem reprezenta a, b sub

forma a = q · α + r, b = q · β + s, cu α, β, r, s ∈ Z, |r| ≤ q

2, |s| ≤ q

2. Deoarece a ≡ r(mod q) s, i

b ≡ s(mod q), rezulta r2 + Ms2 ≡ a2 + Mb2 ≡ 0(mod q) s, i r2 + Ms2 ≤ q2

4+ M

q2

4≤ q2. Sa

observam ca de fapt avem r2 +Ms2 < q2 chiar daca M = 3. Intr-adevar, pentru a avea egalitate

ar trebui ca |r| = |s| = q

2. De aici se obt, ine q|2a s, i q|2b s, i q2|4(a2 + 3b2) = 4pq. Insa p este prim,

iar 1 < q < p. Rezulta q = 2 s, i a2 + 3b2 = 2p. Insa p ≡ 1(mod 3), cum vom proba ulterior, deunde 2 ≡ a2(mod 3), ceea ce este imposibil. Prin urmare r2 + Ms2 = qq1, cu 1 ≤ q1 < q. Incontinuare folosim identitatea

(pq) · (qq1) = (a2 +Mb2)(r2 +Ms2) = (ar ±Mbs)2 +M(as∓ br)2.

Deoarece a ≡ r(mod q) s, i b ≡ s(mod q), rezulta ca as− br ≡ 0(mod q), iar de aici se obt, ine s, iar + Mbs ≡ 0(mod q). Rezulta ca exista a1, b1 ∈ Z astfel ıncat q2pq1 = (qa1)

2 + M(qb1)2 sau

pq1 = a21 +Mb21, cu 1 ≤ q1 < q. Daca q1 > 1, continuam ın acelas, i mod. Dupa un numar finitde pas, i se ajunge la reprezentarea lui p sub forma dorita.

1Prof. univ. dr., Universitatea din Bucures,ti, [email protected]. univ. dr., Universitatea din Pites,ti, [email protected]

21

22 V. Alexandru, S.C. Andronescu

Aplicat, ii

A1. Avem x2 + 1|x4 − 1|xp−1 − 1 daca p = 4t+ 1 iar congruent,a xp−1 = 1(mod p) are p− 1solut, ii (vezi Mica teorema a lui Fermat) ın 1, 2, . . . , p− 1. Deoarece numarul de solut, ii aleunei congruent,e polinomiale f(x) ≡ 0(mod p) nu poate depas, i gradul f (folosind Teorema luiLagrange) ⇒ congruent,a x2 + 1 ≡ 0(mod p) are (exact) doua solut, ii (daca p ≡ 1(mod 4)) ın1, 2, . . . , p− 1.

Fie acum p ≡ 1(mod 8). Atunci x4 + 1|x8 − 1|xp−1 − 1. Cu acelas, i argument ca mai susrezulta ca congruent,a x4 + 1 ≡ 0(mod p) are patru solut, ii ın 1, 2, . . . , p− 1.Avem x4 + 1 ≡ (x2 + 1)2 − 2x2 = (x2 − 1)2 + 2x2. Daca a4 ≡ −1(mod p)⇒ (a2 + 1)2 − 2a2 ≡(a2 − 1)2 + 2a2 ≡ 0(mod p).Cum a 6≡ 0(mod p), rezulta ca ∃b, b′ ∈ Z s, i 2 ≡ b2(mod p) s, i −2 ≡ (b′)2(mod p).

Fie acum p ≡ 1(mod 3), p = 3t + 1. Avem x3 − 1|xp−1 − 1 ⇒ (cu acelas, i argument) ca∃a ∈ Z, a 6≡ 1(mod p) s, i a3 − 1 = (a − 1)(a2 + a + 1) ≡ 0(mod p) de unde 4(a2 + a + 1) =(2a+ 1)2 + 3 ≡ 0(mod p)⇔ −3 ≡ (2a+ 1)2(mod p).Se obt, in astfel urmatoarele consecint,e:

a) Daca p ≡ 1(mod 4) atunci p = a2 + b2 cu a, b ∈ Z.

b) Daca p ≡ 1(mod 8) atunci p = a2 + 2b2 cu a, b ∈ Z.

c) Daca p ≡ 1(mod 3) atunci p = a2 + 3b2 cu a, b ∈ Z.

Observat,ia 1. Daca ∃b ∈ Z, b 6= 0 astfel ıncat b2 ≡ −3(mod p) cu p 6= 3 atunci (conformrat, ionamentului anterior), exista a ∈ N astfel ıncat a2 + a + 1 ≡ 0(mod p) ⇒ a3 ≡ 1(mod p).Daca (p− 1, 3) = 1 ar rezulta a ≡ 1(mod p) iar de aici p=3, fals. Prin urmare 3|p− 1.

A2. Daca un numar natural nenul n se scrie ın doua moduri ca suma de doua patrateperfecte, atunci n este compus.

Solut,ie. Fie n = a2 + b2 = c2 + d2, a, b, c, d ∈ N?, (a, b) 6= (c, d). Daca a s, i b au aceleas, i paritate,atunci n este par, deci compus. Deci putem presupune ca a s, i b, respectiv c s, i d sunt numerenaturale distincte cu paritat, i diferite.

Fie a s, i c pare, b s, i d impare, deci a 6= c, b 6= d.

Avem a2 + b2 = c2 + d2, a2 − c2 = d2 − b2, (a− c)(a+ c) = (d− b)(d+ b), adica

a− cd− b

=d+ b

a+ c=u

v, (u, v) = 1.

Rezulta a+ c = hv, b+ d = hu, a− c = ku, d− b = kv, cu h, k ∈ N? s, i deci

a =hv + ku

2, b =

hu− kv2

, s, i n =(u2 + v2)(k2 + h2)

4.

Se deduce us,or ca ınsus, i n admite o descompunere proprie.

Prin urmare un numar prim admite cel mult o reprezentare ca suma de doua patrate perfecte.

A3. Daca un numar n se poate scrie ca suma de doua patrate perfecte nenule distincte,atunci orice putere a sa se poate scrie de asemenea ca suma de doua patrate perfecte nenule.

Solut,ie. Fie n = a2 + b2, a, b ∈ N?.

Numere prime de forma ... 23

Daca t = 2k + 1, atunci nt = n2k · n = n2k(a2 + b2) = (nk · a)2 + (nk · b)2 cu nka, nkb ∈ N?.

Daca t = 2k demonstram afirmat, ia pentru k = 1

n2 = (a2 + b2)2 = a4 + b4 + 2a2b2 = (a2 − b2)2 + (2ab)2,

adica n2 = A2 +B2 cu A = a2 − b2, B = 2ab.

Pentru k ≥ 2, nt = n2k−2 · n2 = n2(k−1)(A2 +B2) = (nk−1A)2 + (nk−1B)2 cu ak−1A, ak−1B ∈N?.

Mai general, daca n s, i m se pot scrie ca suma de doua patrate perfecte nenule, atuncis, i produsul lor este o suma de doua patrate perfecte nenule. Intr-adevar, daca n = a2 + b2,m = c2 + d2, a, b, c, d ∈ N? atunci mn = (a2 + b2)(c2 + d2) = a2c2 + a2d2 + b2c2 + b2d2 =(ac+ bd)2 + (ad− bc)2.

Putem concluziona astfel ca mult, imea M = n ∈ N?| n = a2 + b2, a, b ∈ N? este partestabila ın raport cu ınmult, irea.

Daca p este un numar prim, p ≥ 3, iar a ∈ Z, (a, p) = 1 atunci simbolul lui Legendre

Ça

p

åse defines,te astfel:Ç

a

p

å=

®1 daca congruent,a x2 ≡ a(mod p) are solut, ii−1 daca congruent,a x2 ≡ a(mod p) nu are solut, ii

.

Ment, ionam cateva proprietat, i ale simbolului lui Legendre:

1.

Ça

p

åse poate extinde pe Z prin

Ça

p

å= 0, daca p|a.

2. Daca a ≡ b(mod p), atunci

Ça

p

å=

Çb

p

å.

3. In condit, iile de mai sus,

Ça

p

å≡ a

p−12 (mod p) (Criteriul lui Euler).

4.

Çab

p

å=

Ça

p

åÇb

p

å.

Un rezultat de seama ın teoria numerelor este Legea de reciprocitate patratica, intuita deEuler, formulata de Legendre s, i demonstrata de Gauss.

Teorema 1. Fie p ≥ 3 un numar prim. Atunci:

1.

Ç−1

p

å= (−1)

p−12 .

2.

Ç2

p

å= (−1)

p2−18 .

3. Daca q este numar prim, q ≥ 3 s, i q 6= p, atunciÇp

q

åÇq

p

å= (−1)

p−12

q−12 .

24 V. Alexandru, S.C. Andronescu

Rezultatele mai noi privind reprezentarea numerelor prime p = x2 +My2 cu x, y ∈ Z s, i Mliber de patrate (rezultate ce folosesc proprietat, ii ale simbolului lui Legendre) pot fi gasite ınlucrarea [4].

Aceste rezultate sunt legate de proprietat, i aritmetice profunde ale inelelor de ıntregi algebricidin corpurile patratice. Ele au la baza studiul formelor patratice binare cu coeficient, i ıntregirealizat de Gauss ın lucrarea [3].

Bibliografie

[1] V. Alexandru, N. Gos,oniu, Elemente de teoria numerelor, Editura Universitat, ii din Bucures,ti,1999.

[2] L. Panaitopol, A. Gica, Probleme de aritmetica s, i teoria numerelor. Idei s, i metode derezolvare, Editura Gill, 2006.

[3] C.Fr. Gauss, Cercetari aritmetice, Editura Amarcord, Timis,oara, 1999.

[4] D.A. Cox, Primes of the form x2 + ny2, Cambridge Univ. Press, 1984.

In legatura cu o inegalitate din ,,Mathematical

Reflections” nr. 6/2015

Marin Chirciu 1

In Revista ,,Mathematical Reflections” nr. 6/2015 sub semnatura Titu Andreescu, USA s, iOleg Mushkarov, Bulgaria este propusa problema O357:

,,Sa se arate ca ın orice triunghi ABC este adevarata inegalitatea:

∑ bc+ 4mbmc

a≥ 16S

R.”

Articolul ıs, i propune o generalizare s, i anume:

,,Sa se arate ca ın orice triunghi ABC este adevarata inegalitatea:

∑ bc+ k ·mbmc

a≥ (3k + 4) · S

R,

unde .k ≥ 4.”

Solut,ie. Vom folosi inegalitat, ile: a = (p− b) + (p− c) ≥ 2»

(p− b)(p− c) s, i

m2a = 2(b2+c2)−a2

4≥ (b+c)2−a2

4= p(p− a)⇒ ma ≥

»p(p− a). Obt, inem:

∑ bc+ k ·mbmc

a≥∑Ñ

(p− a)(p− c) · 2»

(p− a)(p− b)a

+k ·»p(p− b) ·

»p(p− c)

a

é=∑ »

(p− b)(p− c) [4(p− a) + kp]

a=∑Ç»

(p− b)(p− c) · (k + 4)(b+ c) + (k − 4)a

2a

åAm−Gm≥ 3

2

3

√∏ (k + 4)(b+ c) + (k − 4)a

a·∏

(p− a)

(1)

≥ 3

23

√(3k + 4)3 ·

∏(p− a) =

3

2(3k + 4) 3

√∏(p− a)

Gm−Hm≥ 3

2(3k + 4) · 3

1p−a + 1

p−b + 1p−c

=9(3k + 4)

2· 1raS

+ rbS

+ rcS

=9(3k + 4)

2· S

ra + rb + rc=

9(3k + 4)

2· S

4R + r

Euler≥ 9(3k + 4)

2· S9R

2

= (3k + 4) · SR,

1Profesor, Colegiul Nat,ional ,,Zinca Golescu”, Pites,ti, [email protected]

25

26 M. Chirciu

unde (1)⇔ ∏ (k+4)(b+c)+(k−4)aa

≥ (3k + 4)3, care rezulta din:∏ î(k + 4) · b+c

a+ k − 4

ó=∏

(x+ α) = (x+ α)(y+ α)(z + α) = xyz + α∑xy+ α2∑x+ α3,

unde x = (k + 4) · b+ca

s, i α = k − 4. Avem: xyz = (k + 4)3∏ b+c

a≥ 8(k + 4)3;

∑xy = (k + 4)2

∑ (a+b)(a+c)bc

s, i∑ (a+b)(a+c)

bc= p2−r2−Rr

Rr

Gerretsen≥ 12⇒

⇒∑

xy ≥ 12(k + 4)2;∑

x = (k + 4)∑ b+ c

a≥ (k + 4) · 6.

Obt, inem:∏

(x+ α) ≥ 8(k + 4)3 + 12(k + 4)2(k − 4) + 6(k + 4)(k − 4)2 + (k − 4)3

= [2(k + 4) + (k − 4)]3 = (3k + 4)3, pentru k ≥ 4.

Din aceeas, i clasa de probleme va propun sa rezolvat, i:

,,Sa se arate ca ın orice triunghi ABC este adevarata inegalitatea:

∑ 3bc+ 4mbmc

a≥ 24S

R.”

Solut,ie. Se foloses,te inegalitatea mediilor pentru 3bc s, i 4mbmc, inegalitat, ile din solut, ia precedenta

s, i inegalitatea∑ 1

a≥√3R

. Egalitatea are loc pentru triunghiul echilateral, caz ın care are loc s, iegalitatea 3bc = 4mbmc.

Bibliografie

[1] T. Andreescu, O. Mushkarov, Mathematical Reflections, nr. 6/2015.

[2] M. Chirciu, Inegalitat,i geometrice, de la init,iere la performant,a, Editura Paralela 45, Pites,ti,2015.

Coordonate analitice practice

Leonard Mihai Giugiuc 1 s, i Diana Trailescu Gogan 2

In general, pentru rezolvarea unor probleme de geometrie a triunghiului prin metoda analitica,punctele sunt alese astfel: A (0, a), B (b, 0), C (c, 0) cu a > 0 si b < c. Inconvenientul acesteialegeri a punctelor consta ın faptul ca vom avea numitori nedorit, i ın cazul coordonatelorortocentrului sau ale centrului cercului circumscris triunghiului.

Vom face urmatoarea alegere: A (0, a), B (−ab, 0), C (ac, 0), unde a este distant,a de lapunctul A la dreapta BC, b = ctgB, c = ctgC. Evident b+ c > 0.

Printr-o omotetie de centru O s, i raport 1a

(sau 2a), putem alege A (0, 1), B (−b, 0), C (c, 0)

sau A (0, 2), B (−2b, 0), C (2c, 0). In primul caz vom avea ortocentrul H (0, bc), iar ın cel de-aldoilea caz ortocentrul H (0, 2bc) s, i centrul cercului circumscris triunghiului O1 (c− b, 1− bc).

Ca aplicat, ii ale acestei metode vom prezenta ın cele ce urmeaza solut, ii inedite la trei problemedate la Olimpiade Internat, ionale de Matematica.

Aplicat, ia 1. Fie H ortocentrul unui triunghi ascutitunghic ABC. Cercul centrat ın mijlocullui (BC) s, i care trece prin H intersecteaza latura BC ın punctele A1 s, i A2. Analog definimpunctele B1, B2 pe latura AC s, i C1, C2 pe latura AB.

Demonstrat,i ca punctele A1, A2, B1, B2, C1 s, i C2 sunt conciclice.

I.M.O. 2008

Solut,ie. Alegem punctele A (0, 2), B (−2b, 0) s, i C (2c, 0). Cum 4ABC este ascut, itunghicrezulta b > 0, c > 0 s, i bc < 1. Evident, H (0, 2bc) s, i centrul cercului circumscris 4ABCeste O1 (c− b, 1− bc). Din modul de definire a punctelor A1, A2, B1, B2, C1, C2, obt, inem camediatoarea lui [BC] coincide cu mediatoarea lui [A1A2] s, i analog celelalte. Deci e suficient saaratam ca O1 este egal departat de cele s,ase puncte. De asemenea, exista numerele reale strictpozitive m, n, p cu:

A1 (c− b−m, 0) , A2 (c− b+m, 0) ; B1 (c (1− n) , 1 + n) ,

B2 (c (1 + n) , 1− n) , C1 (−b (1− p) , 1 + p) s, i C2 (−b (1 + p) , 1− p) .Deci:

O1A1 = O1A2, O1B1 = O1B2 s, i O1C1 = O1C2. (1)

Din ipoteza, avem 4HA1A2 dreptunghic ın H. Rezulta−−→HA1 ·

−−→HA2 = 0⇒î

(c− b−m)~i− 2bc~jó î

(c− b+m)~i− 2bc~jó

= 0

⇒ (c− b−m) (c− b+m) + 4b2c2 = 0⇒ m2 = 4b2c2 + b2 + c2 − 2bc.

1Profesor, Colegiul Nat, ional,,Traian”, Drobeta Turnu Severin, [email protected], Liceul Tehnologic Halanga, [email protected]

27

28 L.M. Giugiuc, D. Trailescu Gogan

4HB1B2 dreptunghic ın H ⇒−−→HB1 ·

−−→HB2 = 0⇒î

c (1− n)~i+ (1 + n− 2bc) ·~jó·îc (1 + n) ·~i+ (1− n− 2bc)~j

ó= 0

⇒ n2 =4b2c2 + c2 + 1− 4bc

1 + c2.

4HC1C2 dreptunghic ın H ⇒−−→HC1 ·

−−→HC2 = 0 ⇒î

−b (1− p)~i+ (1 + p− 2bc) ·~jó·î−b (1 + p)~i+ (1− p− 2bc) ·~j

ó= 0

⇒ p2 =4b2c2 + b2 + 1− 4bc

1 + b2.

O1A12 = m2 + (1− bc)2 = 5b2c2 + b2 + c2 + 1− 4bc.

O1B12 = (b− cn)2 + (bc+ n)2 = 5b2c2 + b2 + c2 + 1− 4bc.

O1C12 = (c+ bp)2 + (bc− p)2 = 5b2c2 + b2 + c2 + 1− 4bc.

Deci

O1A1 = O1B1 = O1C1. (2)

Din relat, iile (1) s, i (2) obt, inem ca punctele A1, A2, B1, B2, C1 s, i C2 sunt conciclice.

Aplicat, ia 2. Consideram 4ABC ascutitunghic. Fie P piciorul ınalt,imii din A, O centrul

cercului circumscris 4ABC. Presupunem ca B+30 ≤ C. Demonstrat,i ca A+m(÷COP) < 90.

I.M.O. 2001

Solut,ie. Consideram in s.c.c.o. xPy, punctele A (0, 2), B (−2b, 0), C (2c, 0). Cum 4ABC esteascutitunghic ⇒ b, c > 0 s, i b · c < 1. De asemenea, cum B < C ⇒ b > c.

A+m(÷COP) < 90 ⇔ ctg

(A+m

(÷COP)) > 0

⇔ ctg A · ctg(÷COP) > 1.

Avem:

ctg A = −ctg (B + C) =1− bcb+ c

,

ctg(÷COP) = ctg

(◊COM −◊POM), unde M este mijlocul lui (BC).

Notam c.c. al 4ABC cu w. Avem:

w : [x− (c− b)]2 + [y − (1− bc)]2 =Ä1 + b2

ä Ä1 + c2

ä.

In 4COM, ctg(◊COM

)=OM

CM=

1− bcb+ c

.

In 4POM, ctg(◊POM

)=OM

OP=

1− bcb− c

.

Coordonate analitice practice 29

Deci, ctg(÷COP) =

1− bcb+ c

· 1− bcb− c

+ 1

1− bcb− c

− 1− bcb+ c

=(1− bc)2 + b2 − c2

2c · (1− bc).

ctgA · ctg(÷COP) > 1 ⇔ 1− bc

b+ c· (1− bc)2 + b2 − c2

2c (1− bc)> 1

⇔ (1− bc)2 + b2 − c2

2c (b+ c)> 1⇔

Ä1 + b2

ä Ä1 + c2

ä> 4c (b+ c)⇔ R2 > 4c (b+ c) ,

unde R este raza lui w.

In 4PAC, PC = AC cosC = 2R sinB cosC ⇒ 2c = 2R sinB cosC ⇒ c = R sinB cosC.

Aplicand teorema sinusurilor in 4ABC, avem BC = 2R sinA ⇒ 2 (b+ c) = 2R sinA⇒ b+ c = R sinA. Deci avem de aratat ca

R2 > 4R2 sinA sinB cosC ⇔ sinA sinB cosC <1

4

⇔ sinA [sin (B + C)− sin (C −B)] <1

2⇔ sinA (sinA− sin (C −B)) <

1

2.

Cum C −B este unghi ascutit si C −B ≥ 30 ⇒ sin (C −B) ≥ 1

2⇒ − sin (C −B) ≤ −1

2.

Este suficient sa aratam ca

sinA

ÇsinA− 1

2

å<

1

2⇔ (sinA− 1) (2 sinA+ 1) < 0. (3)

Cum A < 90 ⇒ sinA− 1 < 0⇒ inegalitatea (3) este adevarata, deci concluzia problemeieste adevarata.

Aplicat, ia 3. Un 4ABC si un punct D din planul sau satisfac relat,iaBC

AD=CA

BD=AB

CD=√

3.

Demonstrat,i ca 4ABC este echilateral s, i D este centrul sau.

I.M.O. 1980-Luxembourg

Solut,ie. Alegem A (0, 1), B (−b, 0), C (c, 0) s, i D (x, y) .

Avem: BC2 = (b+ c)2, CA2 = 1+c2, AB2 = 1+b2, AD2 = x2+(y − 1)2, BD2 = (x+ b)2+y2

s, i CD2 = (x− c)2 + y2. Deci condit, iile din ipoteza devin:

b2 + 2bc+ c2 = 3x2 + 3y2 + 3− 6y, (4)

1 + c2 = 3x2 + 3y2 + 3b2 + 6bx (5)

s, i1 + b2 = 3x2 + 3y2 + 3c2 − 6cx. (6)

Scazand din relat, ia (5) relat, ia (6) obt, inem 4 (c2 − b2) = 6x (c+ b) ⇒ x =2 (c− b)

3. Scazand

din relat, ia (5) relat, ia (4), obt, inem y =2− 3bc

3. Inlocuind valorile x s, i y gasite ın relat, ia (4),

avem:

b2 + c2 + 2bc =4b2 + 4c2 + 9b2c2 + 4− 20bc

3+ 3− 4 + 6bc

30 L.M. Giugiuc, D. Trailescu Gogan

⇒ 3b2 + 3c2 + 6bc = 4b2 + 4c2 + 9b2c2 + 4− 20bc− 3 + 18bc

⇒ b2 + c2 − 2bc+ 9b2c2 − 6bc+ 1 = 0 ⇒ (b− c)2 + (3bc− 1)2 = 0⇒ b− c = 0

s, i 3bc− 1 = 0⇒ ctgB = ctgC ⇒ B = C s, i 3ctg 2B = 1⇒ ctgB =1√3

(deoarece triunghiul

nu poate avea doua unghiuri obtuze)⇒ B = C =π

3⇒4ABC este echilateral. D

Ç0,

1

3

å⇒ D

este centrul 4ABC.

In ıncheiere vom propune spre rezolvare trei probleme de geometria triunghiului prin metodade mai sus, ale caror enunt,uri pot fi gasite la adresele web urmatoare:

static.olimpiade.ro/uploads/attach data/32/51/5/2013 matematica internationala subiecte.pdf

ssmr.ro/files/onm2013/faza nationala/subiecte/subiecte juniori.pdf

Asupra unei probleme propuse la Olimpiada Nat, ionala

de Matematica

Marin Ionescu 1

La Olimpiada Nat, ionala de Matematica din 2017, domnul profesor Lucian Dragomir dinOt,elu Ros,u a propus elevilor de clasa a X-a urmatoarea problema:

Problema 1. Demonstrat, i inegalitatea:

sinπ

4n≥√

2

2n,

unde n este un numar natural nenul [1].

In cele ce urmeaza vom da o solut, ie diferita de cea propusa de Comisia Centrala.

Inegalitatea din enunt, este o consecint, a a urmatorului rezultat.

Propozit, ie. Pentru orice n ∈ N? s, i x ∈ R, exista inegalitatea

| sinnx| ≤ n| sinx|.

Demonstratie. Pentru n = 1, inegalitatea este verificata. Presupunem inegalitatea adevaratapentru n s, i demonstram ca este adevarata pentru n+ 1.

Avem:

| sin(n+ 1)x| = | sinnx cosx+ sinx cosnx| ≤ | sinnx cosx|+ | sinx cosnx|

= | sinnx| · | cosx|+ | sinx| · | cosnx| ≤ | sinnx|+ | sinx| ≤ n| sinx|+ | sinx| = (n+ 1) · | sinx|.

Cu propozit, ia de mai sus, inegalitatea data la concurs rezulta astfel:

√2

2= sin

π

4=∣∣∣∣sinÅn · π4nã∣∣∣∣ ≤ n ·

∣∣∣∣sin π

4n

∣∣∣∣ = n · sin π

4n

s, i de aici obt, inem

√2

2n≤ sin

π

4n.

In G.M. 10/2017 domnul profesor Traian Preda din Bucures,ti a propus la clasa a X-aurmatoarea problema:

Problema 2. (27437) Sa se demonstreze inegalitatea sinπ

mn≥ 1

msin

π

n, unde m s, i n sunt

numere naturale nenule.

1Profesor, Colegiul Nat,ional ,,Ion C. Bratianu”, Pites,ti, [email protected]

31

32 M. Ionescu

Cu propozit, ia de mai sus, solut, ia este imediata:

sinπ

n= sin

Åm · π

mn

ã=∣∣∣∣sinÅm · πmnã∣∣∣∣ ≤ m ·

∣∣∣∣sin π

mn

∣∣∣∣ = m · sin π

mn

s, i de aici obt, inem

sinπ

mn≥ 1

msin

π

n.

Bibliografie

[1] A 68-a Olimpiada Nat, ionala de Matematica, 19-23 aprilie 2017, Timis,oara.

[2] Colect, ia Gazetei Matematice, Seria B.

Probleme de calcul integral. Inegalitat, i integrale I

Florin Stanescu 1

Scopul acestei note este familiarizarea cititorului cu problemele de calcul integral, urmarindu-se ın special rezolvarea de aplicat, ii din sfera inegalitat, ilor integrale. In final, pentru pasionat, i,am propus un test.

Funct, ii pozitive. Funct, ii monotone. Funct, ii convexe

• Fie f : [a, b]→ R integrabila s, i f (x) ≥ 0, x ∈ [a, b] . Atunci∫ ba f (x) dx ≥ 0.

• Fie f, g : [a, b] → R doua funct, ii integrabile cu proprietatea ca f (x) ≤ g (x) x ∈ [a, b] .Atunci

∫ ba f (x) dx ≤

∫ ba g (x) dx.

• Daca f : [a, b]→ R este o funct, ie integrabila, atunci |f | este integrabila s, i∣∣∣∣∣∫ b

af (x) dx

∣∣∣∣∣ ≤∫ b

a|f (x)| dx.

• Inegalitatea lui Cebas, ev :

Fie f, g : [a, b]→ R doua funct, ii avand aceeas, i monotonie, iar p : [a, b]→ [0,∞) o funct, ieintegrabila. Atunci are loc inegalitatea:Ç∫ b

ap(x)dx

åÇ∫ b

ap(x)f(x)g(x)dx

å≥Ç∫ b

ap(x)f(x)dx

å·Ç∫ b

ap(x)g(x)dx

å.

In cazul cand f s, i g sunt de monotonii diferite, avem:Ç∫ b

ap(x)dx

åÇ∫ b

ap(x)f(x)g(x)dx

å≤Ç∫ b

ap(x)f(x)dx

å·Ç∫ b

ap(x)g(x)dx

å.

Punand p(x) = 1, (∀)x ∈ [a, b], obt, inem:

a) (b− a)∫ ba f(x)g(x)dx ≥

∫ ba f(x)dx

∫ ba g(x)dx, daca f s, i g au aceeas, i monotonie;

b) (b− a)∫ ba f(x)g(x)dx ≤

∫ ba f(x)dx

∫ ba g(x)dx, daca f s, i g sunt de monotonii diferite.

Egalitatea are loc daca una din funct, iile f, g este constanta (cu except, ia, eventual, a uneimult, imi numarabile).

• Daca f : I → R este o funct, ie convexa s, i x, y, z ∈ I astfel ıncat x < y < z, atunci are locinegalitatea:

f(y)− f(x)

y − x≤ f(z)− f(y)

z − y.

1Profesor, S, coala Gimnaziala ,,S, erban Cioculescu”, Gaes,ti, [email protected]

33

34 F. Stanescu

Aplicat, ii

1. Se considera f : [a, b]→ R o funct, ie derivabila, cu derivata continua, astfel ıncat f (a) =f (b) = 0. Aratat, i ca are loc inegalitatea:

maxa≤t≤b

f (t) ≤ 1

2·∫ b

a|f ′ (x)| dx.

Solut,ie. Avem |f (x)| = |∫ xa f′ (t) dt| s, i |f (x)| =

∣∣∣∫ bx f ′ (t) dt∣∣∣ , deci 2 |f (x)| = |∫ xa f′ (t) dt|+∣∣∣∫ bx f ′ (t) dt∣∣∣ ≤∫ xa |f ′ (t)| dt+

∫ bx |f ′ (t)| dt =

∫ ba |f ′ (x)| dx, (∀)x ∈ [a, b] . Astfel, max

a≤t≤bf (t) ≤

12·∫ ba |f ′ (x)| dx.

2. Daca f : [a, b]→ R este o funct, ie derivabila cu f ′ continua pe [a, b], demonstrat, i inegalita-tea:

0 ≤ 1

b− a·∫ b

a|f (x)| dx−

∣∣∣∣∣ 1

b− a·∫ b

af (x) dx

∣∣∣∣∣ ≤ b− a3· maxa≤x≤b

|f ′ (x)| .

Solut,ie. Avem∣∣∣∫ ba f (x) dx

∣∣∣ ≤ ∫ ba |f (x)| dx, deci 0 ≤ 1b−a ·

∫ ba |f (x)| dx−

∣∣∣ 1b−a ·

∫ ba f (x) dx

∣∣∣.Mai departe, vom calcula:∫ b

af (x) dx+

∫ x

a(t− a) f ′ (t) dt−

∫ b

x(b− t) f ′ (x) dt

=∫ b

af (x) dx+ (x− a) f (x)−

∫ x

af (x) dx+ (b− x) f (x)−

∫ b

xf (x) dx = (b− a) f (x) ,

de unde obt, inem ca

|(b− a) f (x)| =∣∣∣∣∣∫ b

af (x) dx+

∫ x

a(t− a) f ′ (t) dt−

∫ b

x(b− t) f ′ (x) dt

∣∣∣∣∣≤∣∣∣∣∣∫ b

af (x) dx

∣∣∣∣∣+∫ x

a(t− a) |f ′ (t)| dt+

∫ b

x(b− t) |f ′ (t)| dt.

Rezulta

(b− a) ·∫ b

a|f (x)| dx− (b− a)

∣∣∣∣∣∫ b

af (x) dx

∣∣∣∣∣≤∫ b

a

Å∫ x

a(t− a) |f ′ (t)| dt

ãdx+

∫ b

a

Ç∫ b

x(b− t) |f ′ (t)| dt

ådx

≤ (b− a)3

3· maxa≤x≤b

|f ′ (x)| ,

ceea ce ıncheie demonstrat, ia.3. Fie f : [a, b]→ R derivabila cu derivata continua. Sa se arate ca:∣∣∣∣∣

∫ a+b2

af (x) dx−

∫ b

a+b2

f (x) dx

∣∣∣∣∣ ≤ (b− a)2

4· maxx∈[0,1]

|f ′ (x)| .

Solut,ie. Avem:∫ ba f (x) dx =

∫ ba (x− a)

′f (x) dx = (b− a) f (b) −

∫ ba (x− a) f ′ (x) dx,

iar din teorema de medie exista c1 ∈ (a, b) astfel ıncat∫ ba (x− a) f ′ (x) dx = f ′ (c1) ·∫ b

a (x− a) dx = f ′ (c1)(b−a)2

2, de unde

∫ ba f (x) dx = (b− a) f (b) − f ′ (c1)

(b−a)22

. In mod

analog, exista c2 ∈ (a, b) astfel ıncat∫ ba f (x) dx =(b− a) f (a) + f ′ (c2)

(b−a)22

.

Inegalitat, i integrale 35

Astfel, inegalitatea de demonstrat rezulta din:∣∣∣∣∣∣Ça+ b

2− a

åf

Ça+ b

2

å− f ′ (c1)

(b− a)2

8−Çb− a+ b

2

åf

Ça+ b

2

å−f ′ (c2)

(b− a)2

8

∣∣∣∣∣∣ =(b− a)2

8|f ′ (c1) + f ′ (c2)| ≤

(b− a)2

4· maxx∈[0,1]

|f ′ (x)| .

4. Fie f : [0,∞)→ [0,∞) o funct, ie integrabila. Aratat, i ca:∫ 1

0f (x) dx ·

∫ 1

0f 2 (x) dx ≤

∫ 1

0f 3 (x) dx.

Solut,ie. Din inegalitatea lui Schur de gradul al III-lea, avem:

f 3 (x) + f 3 (y) + f 3 (z) + 3f (x) f (y) f (z) ≥ f 2 (x) f (y) + f 2 (y) f (x)

+f 2 (y) f (z) + f 2 (z) f (y) + f 2 (z) f (x) + f 2 (z) f (y) ,

(∀)x, y, z ∈ [0,∞) , de unde prin integrare ın funct, ie de cele trei variabile, obt, inem

3∫ 1

0f 3 (x) dx+ 3

Ç∫ 1

0f (x) dx

å3

≥ 6∫ 1

0f 2 (x) dx ·

∫ 1

0f (x) dx

⇒Jensen

6∫ 1

0f 2 (x) dx ·

∫ 1

0f (x) dx ≤ 6

∫ 1

0f 3 (x) dx,

ceea ce ıncheie demonstrat, ia.5. Fie f : [0,∞) → [0,∞) o funct, ie integrabila pe orice interval [0, a] , a > 0, astfel incatf (x) ≤ 1

x2s, i∫ x0

îfÄt2

ä− f (t)

ódt ≤ 1, (∀)x ∈ (0,∞) . Sa se arate ca∫ 1

0f (x) dx ≤ 1.

Solut,ie. Relat, ia din enunt, poate fi scrisa sub forma∫ x2

0f (t) dt ≤ 1

2

Å1 +

∫ x

0f (t) dt

ã, (∀)x > 0

⇒∫ x

0f (t) dt ≤ 1

2

Ç1 +

∫ 2x

0f (t) dt

å,

iar prin induct, ie obt, inem ca∫ x

0f (t) dt ≤ 1

2n

Ç2n − 1 +

∫ 2nx

0f (t) dt

å, (∀)n ≥ 1.

Acum, pentru a > 1, putem scrie:∫ a

0f (x) dx =

∫ 1

0f (x) dx+

∫ a

1f (x) dx ≤

∫ 1

0f (x) dx+

∫ a

1

1

x2dx

=∫ 1

0f (x) dx+ 1− 1

a< 1 +

∫ 1

0f (x) dx := A.

In continuare, ∫ x

0f (t) dt ≤ 1

2n

Ç2n − 1 +

∫ 2nx

0f (t) dt

å≤ A+ 2n − 1

2n→n→∞

1, (∀)x > 0⇒∫ 1

0f (t) dt ≤ 1.

36 F. Stanescu

6. Fie a, b ∈ R cu a > 0, 2a + 3b = 6 s, i f : [0, 1] → R descrescatoare. Sa se arate ca:∫ 10 f (x) dx ≥

∫ 10 (ax2 + bx) f (x) dx.

Solut,ie. Notam g : [0, 1]→ R, g(x) = ax2 + bx− 1. Ecuat, ia g (x) = 0 are o unica solut, ie αin intervalul (0, 1).

Rezulta g (x) ≤ 0, (∀)x ∈ [0, α] s, i g (x) ≥ 0, (∀)x ∈ [α, 1] . Cum f este descrescatoare,obt, inem

g (x) (f (x)− f (α)) ≤ 0, (∀)x ∈ [0, α]

s, i

g (x) (f (α)− f (x)) ≥ 0, (∀)x ∈ [α, 1]

⇒ g (x) (f (x)− f (α)) ≤ 0, (∀)x ∈ [0, 1]⇒∫ 1

0g (x) f (x) dx ≤ 0.

7. Daca f : [0, 2π]→ R este o funct, ie convexa s, i integrabila, atunci are loc inegalitatea:∫ 2π

0f (x) · cosxdx ≥ 0.

Solut,ie. Putem scrie:

∫ 2π

0f (x) · cosxdx =

∫ π

0f (x) cosxdx+

∫ 2π

πf (x) cosxdx.

Pentru ultima integrala facand substitut, ia x = t+ π, rezulta ca∫ 2π

πf (x) cosxdx = −

∫ π

0f (x+ π) cosxdx,

de unde ∫ 2π

0f (x) · cosxdx =

∫ π

0cosx (f (x)− f (x+ π)) dx.

Acum, pentru x, y ∈ [0, π] , x < y, deci x < y < x+ π < y + π, obt, inem ca

f (y + π)− f (x+ π)

y − x≥ f (y)− f (x)

y − x

⇒ f (y)− f (y + π) ≤ f (x)− f (x+ π) .

Astfel, funct, ia f (x)− f (x+ π) , x ∈ [0, π] este descrescatoare.

In final, cum s, i cosx este descrescatoare pe [0, π], aplicand inegalitatea lui Cebas,ev, avem∫ 2π

0f (x) · cosxdx =

∫ π

0cosx (f (x)− f (x+ π)) dx

≥ 1

π

Å∫ π

0cosxdx

ã·Å∫ π

0(f (x)− f (x+ π)) dx

ã= 0.

8. Presupunem ca f : [0, 1] → R este o funct, ie derivabila, cu f ′ continua, astfel ıncat∫ 10 f (x) dx = 0. Pentru α ∈ (0, 1) demonstrat, i inegalitatea:∣∣∣∣∫ α

0f (x) dx

∣∣∣∣ ≤ 1

8· maxx∈[0,1]

|f ′ (x)| .

Inegalitat, i integrale 37

Solut,ie. Consideram funct, ia derivabila de doua ori

g : [0, 1]→ R, g (x) =∫ x

0f (t) dt+M · x

2

2,

unde M = maxx∈[0,1]

|f ′ (x)| . Avem g′′ (x) = f ′ (x) +M ≥ 0, (∀)x ∈ [0, 1] , deci g este convexa

pe [0, 1] . Folosind ca g este convexa, putem scrie:

g (α) = g ((1− α) · 0 + α · 1) ≤ (1− α) g (0) + αg (1)

⇒∫ α

0f (x) dx+

α2

2M ≤ α

2M ⇒

∫ α

0f (x) dx =

M

2α (1− α) ≤ M

2

Ç1− α + α

2

å2

=1

8M.

In final, ınlocuind pe f cu −f , obt, inem ca −∫ α0 f (x) dx ≤ 1

8M , deci∣∣∣∣∫ α

0f (x) dx

∣∣∣∣ ≤ 1

8· maxx∈[0,1]

|f ′ (x)| .

9. Daca h este o funct, ie pozitiva s, i descrescatoare pe [0, 1], atunci:∫ 10 xh

2 (x) dx∫ 10 xh (x) dx

≤∫ 10 h

2 (x) dx∫ 10 h (x) dx

.

Solut,ie. In inegalitatea lui Cebas,ev, pentru f (x) = x, g (x) = 1h(x)

, p (x) = h2 (x) , obt, inem:Ç∫ 1

0h2(x)dx

å·Ç∫ 1

0h2 (x) · x · 1

h (x)dx

å≥Ç∫ b

ah2(x) · xdx

åÇ∫ b

ah2(x)

1

h (x)dx

å⇒Ç∫ 1

0h2(x)dx

å·∫ 1

0x · h (x) dx ≥

Ç∫ b

ah2(x) · xdx

åÇ∫ b

ah(x)dx

å.

10. Consideram P ∈ R[X] un polinom de grad n ≥ 2 cu coeficient, i strict pozitivi s, i 0 ≤ a < b.Daca P are numai radacini reale, demonstrat, i ca:

(b− a)∫ b

a

P ′′(x)

P (x)dx > ln

P (a)

P (b)· ln P

′(a)

P ′(b).

Solut,ie. Daca x1, x2, ..., xn sunt radacinile polinomului P , din enunt, deducem ca ele suntstrict negative. Mai avem ca

P ′(x)

P (x)=

1

x− x1+

1

x− x2+ ...+

1

x− xn, (∀)x ≥ 0.

Derivand, obt, inem:ÇP ′(x)

P (x)

å′= −

(1

(x− x1)2+

1

(x− x2)2+ ...+

1

(x− xn)2

)< 0.

In continuare, construind funct, ia φ : [0,∞) → R, φ(x) = P ′(x)P (x)

, rezulta ca φ este strict

descrescatoare. Deoarece P ′ are numai radacini reale strict negative, analog, obt, inem ca

funct, ia ψ : [0,∞) → R, ψ(x) = P ′′(x)P ′(x)

este strict descrescatoare, deci funct, iile φ s, i ψ auaceeas, i monotonie si nu sunt constante.

Daca 0 ≤ a < b, putem scrie:∫ b

a

P ′′(x)

P (x)dx =

∫ b

aφ(x) · ψ(x)dx >

1

b− a

∫ b

aφ(x)dx ·

∫ b

aψ(x)dx

=1

b− a

∫ b

a

P ′(x)

P (x)dx ·

∫ b

a

P ′′(x)

P ′(x)dx =

1

b− alnP (b)

P (a)· ln P

′(b)

P ′(a).

38 F. Stanescu

Test

1. Fie f : [0, 1]→ R. Daca f este derivabila s, i exista a ∈ (0, 1] astfel ıncat∫ a0 f (x) dx = 0, sa

se arate ca∣∣∣∫ 10 f (x) dx

∣∣∣ ≤ 1−a2

supx∈[0,1]

|f ′ (x)|.

2. Daca f : [0, 1]→ R este o funct, ie crescatoare, demonstrat, i inegalitat, ile:

∫ 1n

0f (x) dx ≤

∫ 1

0xn−1f (x) dx ≤

∫ 1

1− 1n

f (x) dx, (∀)n ∈ N∗.

3. Daca f : [0, 1]→ R este o funct, ie derivabila, cu f ′ integrabila s, i∫ 10 f (x) dx = 0, demonstrat, i

inegalitatea: ∫ 1

0

Ä1− x2

ä(f ′ (x))

2dx ≥ 24

Ç∫ 1

0xf (x) dx

å2

.

4. Daca f : [0, 1]→ R, este o funct, ie integrabila, cuf ′ integrabila s, i fÄ12

ä= 0, aratat, i ca are

loc inegalitatea: Ç∫ 1

0f (x) dx

å2

≤ 1

12·∫ 1

0(f ′ (x))

2dx.

5. Fie a, b > 0. Demonstrat, i ca:

1√2π

Ç1

a+

1

b

å≤∫ √π

2

0

Çsinx2

a+

cosx2

b

ådx ≤

√π

2

Ç1

a2+

1

b2

å.

Bibliografie

[1] M. Andronache, R. Gologan, D. Schwarz, D. S, erbanescu, Olimipiada de matematica 2006-2010, Ed. Sigma, Bucures,ti, 2010.

[2] M.O. Drimbe, Inegalitat,i, idei s, i metode, Ed. Gil, Zalau, 2003.

[3] L.G. Ladunca, Borne pentru matematicieni, Algebra-Analiza, clasele IX-XII, Ed. Taida, Ias, i,2010.

[4] C. Mortici, Bazele Matematicii. Teorie s, i exercit,ii, Ed. Paralela 45, Pites,ti, 2016.

[5] N. Mus,uroaia, Gh. Boroaica, Analiza Matemtica pentru concursuri, olimpiade s, i centre deexcelent,a, Clasa a XII-a, Ed. Paralela 45, Pites,ti, 2014.

[6] F. Stanescu, Inegalitat,i integrale:de la init,iere la performant,a, Ed. Paralela 45, Pites,ti, 2015.

[7] R.T. Rockafeller, Analiza convexa, Ed. Theta, Bucures,ti, 2002.

[8] Colect, ia Gazetei Matematice, Seria B.

ARTICOLE S, I NOTE DE INFORMATICA

Optimalitatea algoritmului de cautare binara

Costel Balcau 1

In acest articol vom demonstra ca algoritmul de cautare binara este optim. Pentru aceastavom prezenta s, i utiliza cateva not, iuni s, i rezultate de baza din teoria algoritmilor: ordine decomplexitate, Teorema Master, proprietat, i ale arborilor binari strict, i.

Ordine de complexitate

Timpul de execut, ie al unui algoritm depinde, ın general, de setul datelor de intrare, iar pentrufiecare astfel de set el este bine determinat de numarul de operat, ii executate s, i de tipul acestora.Astfel timpul de execut, ie al unui algoritm poate fi interpretat s, i analizat drept o funct, ie pozitivace are ca argument dimensiunea datelor de intrare.

Definit, ia 1. Pentru orice algoritm A, notam cu TA(n) timpul de execut,ie pentru algoritmulA corespunzator unui set de date de intrare avand dimensiunea totala n.

Definit, ia 2. Un algoritm A este considerat optim daca (se demonstreaza ca) nu exista unalgoritm avand un timp de execut,ie mai bun pentru rezolvarea problemei date, adica pentru oricealgoritm A′ care rezolva problema data avem TA(n) ≤ TA′(n), pentru orice n.

Obt, inerea de algoritmi pur optimi - ın sensul definit, iei anterioare - este posibila ın put, inesituat, ii, iar demonstrarea optimalitat, ii acestora este de obicei dificila. Mult mai des se ıntalnescalgoritmi cu o comportare apropiata de cea optima, pentru valori suficient de mari ale dimensiuniisetului datelor de intrare. Prezentam ın continuare cateva not, iuni prin care se cuantifica aceastaapropiere.

Definit, ia 3. Fie f, g : N \ A→ R+ doua funct,ii, unde A este o mult,ime finita, A ⊂ N.

a) f s, i g se numesc asimptotic echivalente s, i notam f(n) ∼ g(n) daca ∃ limn→∞

f(n)

g(n)= 1.

b) Spunem ca f este asimptotic marginita superior de g, iar g este asimptoticmarginita inferior de f s, i notam f(n) = O (g(n)) s, i g(n) = Ω(f(n)) daca ∃ c > 0, ∃ n0 ∈ Nastfel ıncat f(n) ≤ c · g(n), ∀ n ≥ n0.

c) Spunem ca f s,i g au acelas, i ordin de cres, tere s,i notam f(n) = Θ(g(n)) dacaf(n) = O (g(n)) s, i f(n) = Ω(g(n)).

Urmatorul rezultat este o consecint, a imediata a definit, iei anterioare.

1Conf. univ. dr., Universitatea din Pites,ti, [email protected]

39

40 C. Balcau

Propozit, ia 1. Fie f, g : N \ A → R+ doua funct,ii, unde A este o mult,ime finita, A ⊂ N.

Presupunem ca exista limn→∞

f(n)

g(n)= L. Atunci:

a) f(n) = O (g(n)) daca s, i numai daca L ∈ [0,+∞);

b) f(n) = Ω(g(n)) daca s, i numai daca L ∈ (0,+∞];

c) f(n) = Θ(g(n)) daca s, i numai daca L ∈ (0,+∞).

Corolarul 1. Fie f, g : N \ A→ R+ doua funct,ii, unde A este o mult,ime finita, A ⊂ N. Dacaf(n) ∼ g(n), atunci f(n) = Θ(g(n)).

Definit, ia 4. Fie A un algoritm s, i f : N\A→ R+ o funct,ie, unde A este o mult,ime finita, A ⊂ N.Spunem ca algoritmul A are ordinul de complexitate (complexitatea) O (f(n)), Ω(f(n)),respectiv Θ(f(n)) daca TA(n) = O (f(n)), TA(n) = Ω(f(n)), respectiv TA(n) = Θ(f(n)).

Observat,ia 1. Notat, ia O se utilizeaza pentru a exprima complexitatea unui algoritm cores-punzatoare timpului de execut,ie ın cazul cel mai defavorabil, fiind astfel cea mai adecvata analizeialgoritmilor. Notat, ia Ω este corespunzatoare timpului de execut,ie ın cazul cel mai favorabil, cazpractic irelevant, fiind astfel mai put, in utilizata. Notat, iile ∼ s, i Θ se utilizeaza atunci cand seconstata ca timpii de execut, ie corespunzatori cazurilor cel mai defavorabil s, i cel mai favorabilfie sunt chiar asimptotic echivalent, i (notat, ia ∼, deci s, i notat, ia Θ), cazul cel mai simplu fiindacela al algoritmilor a caror executare depinde doar de dimensiunea setului de date de intrare,nu s, i de valorile acestor date, fie au macar acelas, i ordin de cres,tere (notat, ia Θ). Tot acestenotat, ii se utilizeaza s, i atunci cand se poate determina timpul mediu de execut,ie al algoritmului,calculat ca medie aritmetica ponderata a timpilor de execut, ie pentru toate seturile de date deintrare posibile, ponderile fiind frecvent,ele de aparit, ie ale acestor seturi.

Definit, ia 5. Un algoritm A este considerat asimptotic-optim daca (se demonstreaza ca) nuexista un algoritm avand un ordin de complexitate mai bun pentru rezolvarea problemei date,adica pentru orice algoritm A′ care rezolva problema data avem TA(n) = O (TA′(n)).

Evident, orice algoritm optim este asimptotic-optim. Reciproca acestei afirmat, ii nu esteadevarata, ın continuare fiind prezentat un exemplu ın acest sens.

Exemplul 1. Consideram problema determinarii maximului s, i minimului dintre elementele unuivector dat A = (a1, a2, . . . , an), n ≥ 1, adica se cere sa se determine perechea (M,m), undeM = maxai | 1 ≤ i ≤ n, m = minai | 1 ≤ i ≤ n.

Un algoritm uzual de rezolvare este urmatorul.

MAX-MIN(A, n,M,m) :M ← a1; m← a1;for i = 2, n do

if ai > M thenM ← ai;

elseif ai < m then

m← ai;

Pentru evaluarea complexitat, ii algoritmilor care rezolva problema data, vom analiza numai com-parat, iile ın care intervin elemente ale vectorului sau valorile M s, i m, numite comparat,ii de chei.

Optimalitatea algoritmului de cautare binara 41

Evident, algoritmul MAX-MIN efectueaza cel put, in n− 1 s, i cel mult 2n− 2 astfel de comparat, ii(iar celelalte operat, ii nu depas,esc ordinul de cres,tere al acestora), deci are complexitatea Θ(n).

Se poate demonstra us,or prin induct, ie ca orice algoritm A care calculeaza maximul dintre nelemente, bazat pe comparat, ii de chei, necesita cel put, in n− 1 astfel de comparat, ii, deci arecomplexitatea Ω(n). Astfel TA(n) = Ω(TMAX-MIN(n)), sau, echivalent, TMAX-MIN(n) = O (TA(n)),deci algoritmul MAX-MIN este asimptotic-optim (ın clasa algoritmilor bazat, i pe comparat, ii dechei). Pe de alta parte, el nu este optim, din punct de vedere al timpului de execut, ie ın cazulcel mai defavorabil, deoarece exista algoritmi (bazat, i pe comparat, ii de chei) care ın cazul celmai defavorabil efectueaza mai put, in de 2n− 2 comparat, ii de chei, cat efectueaza algoritmulMAX-MIN. Un astfel de exemplu este urmatorul algoritm, ce compara a1 cu a2, a3 cu a4, . . . ,s, i calculeaza maximul dintre maximele acestor perechi s, i minimul dintre minimele perechilor.

MAX-MIN-PER(A, n,M,m) :M ← an; m← an;for i = 1, bn/2c do

if a2i−1 > a2i thenif a2i−1 > M then

M ← a2i−1;

if a2i < m thenm← a2i;

elseif a2i > M then

M ← a2i;

if a2i−1 < m thenm← a2i−1;

(bxc reprezinta partea ıntreaga a numarului real x). Evident, algoritmul MAX-MIN-PER

efectueaza exact 3 ·õn

2

ûcomparat, ii de chei, indiferent de ordinea dintre elementele vectorului A

(iar celelalte operat, ii nu depas,esc ordinul de cres,tere al acestora), deci are complexitatea Θ(n).

Teorema Master

Prezentam un rezultat celebru, deosebit de util ın determinarea complexitat, ii algoritmilorspecifici metodei de programare Divide et Impera s, i nu numai.

Consideram o relat, ie de recurent, a de forma

T (n) = T (n/b) + . . .+ T (n/b)︸ ︷︷ ︸a termeni

+f(n), ∀n > n0,

unde T, f : N → R+, n0 ∈ N, a ∈ N∗, b ∈ R, b > 1 s, i, prin convent, ie, fiecare termen n/b

reprezinta fieõnb

û, fie

°nb

§.

Ment, ionam ca dxe = mink | k ∈ Z, k ≥ x reprezinta aproximarea ıntreaga prin adaos anumarului real x, numita s, i partea ıntreaga superioara a lui x. In acest context, partea ıntreagauzuala, bxc (notata de obicei cu [x]), se numes,te s, i partea ıntreaga inferioara a lui x.

Prin abuz de notat, ia de ınmult, ire, aceasta relat, ie de recurent, a este convent, ional rescrisaprescurtat sub forma

T (n) = aT (n/b) + f(n), ∀n > n0. (1)

42 C. Balcau

Urmatorul rezultat prezinta o metoda generala pentru rezolvarea acestei relat, ii de recurent, a.

Teorema 1 (Teorema Master [3]). Fie T : N→ R+ o funct,ie ce verifica relat,ia de recurent,a(1), sub ipotezele s, i convent,iile de mai sus.

Cazul 1. Daca f(n) = O (nc), cu c < logb a, atunci T (n) = Θ(nlogb a).

Cazul 2. Daca f(n) = Θ(nc logk2 n), cu c = logb a s, i k ≥ 0, atunci T (n) = Θ(nlogb a logk+12 n).

Cazul 3. Daca f(n) = Ω(nc), cu c > logb a, s,i exista k < 1 s,i n1 ∈ N astfel ıncataf(n/b) ≤ kf(n) ∀n ≥ n1, atunci T (n) = Θ(f(n)).

Proprietat, i ale arborilor binari strict, i

Deoarece terminologia ın tematica arborilor nu este unificata, definim not, iunile pe care le vomutiliza ın continuare.

Definit, ia 6. Un arbore binar este un arbore cu radacina, reprezentat pe nivele, ın care fiecarenod x are cel mult doi descendent,i (fii, succesori direct,i), situat,i pe nivelul urmator, s, i anume(cel mult) un descendent stang, situat ın stanga lui x, s,i (cel mult) un descendent drept,situat ın dreapta lui x, contand ordinea dintre aces,tia. Nodurile cu cel put,in un descendent senumesc interne (interioare), iar nodurile fara descendent,i se numesc externe (exterioare,frunze).

Definit, ia 7. Fie T = (V, F ) un arbore binar avand radacina r, r ∈ V .

a) Notam cu I(T ) mult,imea nodurilor interne s,i cu E(T ) mult,imea nodurilor ex-terne ale arborelui binar T .

b) Pentru orice nod x ∈ V , distant,a de la nodul radacina r la nodul x, notata cuDT (x) = D(x), reprezinta lungimea lant,ului elementar unic de la r la x.

c) Numarul h(T ) = maxDT (x) | x ∈ V se numes,te ınalt,imea arborelui binar T .

Observat,ia 2. Fie T = (V, F ) un arbore binar. Evident, avem V = I(T )∪E(T ), I(T )∩E(T ) = Ø,E(T ) 6= Ø s, i h(T ) = maxDT (x) | x ∈ E(T ). Deci h(T ) = k − 1, unde k reprezinta numarulde nivele ale arborelui T .

Daca nivelul pe care este reprezentata radacina arborelui este numerotat cu 0, atunci pentruorice nod x ∈ V avem DT (x) = numarul nivelului pe care este situat nodul x, iar ınalt, imeaarborelui T este egala cu numarul ultimului nivel al arborelui.

Definit, ia 8. Un arbore binar strict este un arbore binar pentru care orice nod intern areexact doi descendent,i (s, i anume un descendent stang s, i un descendent drept).

Propozit, ia 2. Orice arbore binar strict cu n noduri interne, n ∈ N, are n+ 1 noduri externe.

Demonstratie. Demonstram afirmat, ia din enunt, prin induct, ie dupa n.

Pentru n = 0 este evident ca arborele este format doar din nodul radacina, deci are un singurnod s, i acesta este extern.

Presupunem afirmat, ia adevarata pentru n − 1 s, i o demonstram pentru n, n ≥ 1. FieT = (V, F ) un arbore binar strict cu n noduri interne. Fie x un nod intern al lui T situat pe

Optimalitatea algoritmului de cautare binara 43

penultimul nivel, deci x are ca descendent, i doua noduri externe y s, i z, situate pe ultimul nivel.Fie

T ′ = T \ y, z = (V \ y, z, F \ [x, y], [x, z])arborele obt, inut din T prin eliminarea nodurilor externe y s, i z. Evident, T ′ este arbore binarstrict s, i I(T ′) = I(T ) \ x, E(T ′) = E(T ) \ y, z ∪ x, deci card(I(T ′)) = card(I(T ))− 1 =n− 1, card(E(T ′)) = card(E(T ))− 1. Conform ipotezei de induct, ie, aplicata pentru T ′, avemcard(E(T ′)) = card(I(T ′)) + 1 = n, deci card(E(T )) = card(E(T ′)) + 1 = n+ 1, adica arboreleT are n+ 1 noduri externe. Demonstrat, ia prin induct, ie este ıncheiata.

Propozit, ia 3. Pentru orice arbore binar strict T cu n noduri interne, n ∈ N, avem∑v∈E(T )

DT (v) =∑

v∈I(T )DT (v) + 2n.

Demonstratie. Demonstram egalitatea din enunt, prin induct, ie dupa n.

Pentru n = 0 este evident ca arborele T este format doar din nodul radacina r, E(T ) = r,I(T ) = Ø,

∑v∈E(T )

DT (v) = DT (r) = 0,∑

v∈I(T )DT (v) + 2n = 0 + 2 · 0 = 0, deci egalitatea din

enunt, este verificata.

Presupunem egalitatea adevarata pentru n − 1 s, i o demonstram pentru n, n ≥ 1. FieT = (V, F ) un arbore binar strict cu n noduri interne. Fie x un nod intern al lui T situat pepenultimul nivel, deci x are ca descendent, i doua noduri externe y s, i z, situate pe ultimul nivel.Evident, DT (y) = DT (z) = 1+DT (x). Fie T ′ = T \y, z = (V \y, z, F \[x, y], [x, z]) arboreleobt, inut din T prin eliminarea nodurilor externe y s, i z. Evident, T ′ ramane tot arbore binarstrict s, i I(T ′) = I(T ) \ x, E(T ′) = E(T ) \ y, z∪ x, DT ′(v) = DT (v), ∀ v ∈ I(T ′)∪E(T ′).Utilizand s, i ipoteza de induct, ie, aplicata pentru T ′, avem

∑v∈E(T )

DT (v) =∑

v∈E(T ′)DT ′(v)−DT ′(x)+

DT (y)+DT (z) =∑

v∈I(T ′)DT ′(v)+2(n−1)−DT (x)+2(1+DT (x)) =

∑v∈I(T )\x

DT (v)+DT (x)+2n =∑v∈I(T )

DT (v) + 2n. Demonstrat, ia prin induct, ie este ıncheiata.

Propozit, ia 4. Pentru orice arbore binar strict T cu n noduri interne, n ∈ N, avem

h(T ) ≥ dlog2(n+ 1)e .

Demonstratie. Consideram ca nivelul pe care este reprezentata radacina arborelui este numerotatcu 0. T este un arbore binar strict, cu 2n+ 1 noduri (n interne s, i n+ 1 externe), iar pe fiecarenivel i ∈ 0, 1, . . . , h(T ) el are cel mult 2i noduri (induct, ie!), deci 2n+1 ≤ 1+2+22+ . . .+2h(T ).Obt, inem, succesiv: 2n + 1 ≤ 2h(T )+1 − 1, 2h(T ) ≥ n + 1, h(T ) ≥ log2(n + 1). Cum h(T ) ∈ N,deducem ca h(T ) ≥ dlog2(n+ 1)e .

Observat,ia 3. Pentru orice n ∈ N∗ avem dlog2(n+ 1)e = 1 + blog2 nc .

Intr-adevar, fie dlog2(n+ 1)e = k, k ∈ N∗. Avem, succesiv: k − 1 < log2(n + 1) ≤ k,2k−1 < n+1 ≤ 2k, 2k−1 ≤ n < 2k, k−1 ≤ log2 n < k. Deci blog2 nc = k−1 = dlog2(n+ 1)e−1.

Propozit, ia 5. Fie T un arbore binar strict cu n noduri interne, n ∈ N. Daca toate nodurileexterne sunt situate pe ultimul s, i, eventual, pe penultimul nivel, atunci avem

h(T ) = dlog2(n+ 1)e ,∑v∈E(T )

DT (v) = (n+ 1) dlog2(n+ 1)e+ n+ 1− 2dlog2(n+1)e.

44 C. Balcau

Demonstratie. Consideram din nou ca nivelul pe care este reprezentata radacina arborelui estenumerotat cu 0. Rezulta ca:

• arborele are h(T ) + 1 nivele, numerotate cu 0, 1, . . . , h(T );

• pe nivelele 0, 1, . . . , h(T )− 2 avem doar noduri interne, fiecare avand cate doi descendent, i;

• pe penultimul nivel, h(T )− 1, putem avea s, i noduri externe, iar nodurile interne de peacest nivel au de asemenea cate doi descendent, i;

• pe ultimul nivel, h(T ), avem doar noduri externe;

• pentru orice i ∈ 0, 1, . . . , h(T )− 1, pe nivelul i avem exact 2i noduri.

Notand cu a numarul de noduri interne de pe penultimul nivel s, i cu b numarul de noduri externede pe acelas, i nivel, obt, inem ca

a+ b = 2h(T )−1 (2)

(numarul total de noduri de pe penultimul nivel, nivelul h(T )− 1), iar a ≥ 1 (pe penultimulnivel exista cel put, in un nod intern).

Pe nivelele 0, 1, . . . , h(T )− 2 nu avem noduri externe, pe penultimul nivel, h(T )− 1, avem bnoduri externe, iar pe ultimul nivel, h(T ), avem 2a noduri externe (s, i anume descendent, ii celora noduri interne de pe penultimul nivel), deci numarul total de noduri externe ale arborelui esteegal cu b+ 2a. Rezulta ca

b+ 2a = n+ 1 (3)

s, i ∑v∈E(T )

DT (v) = b(h(T )− 1) + 2a · h(T ). (4)

Din relat, iile (2) s, i (3) obt, inem ca

a = n+ 1− 2h(T )−1. (5)

Dar 1 ≤ a ≤ 2h(T )−1 (a doua inegalitate rezulta din (2)), deci 1 ≤ n + 1 − 2h(T )−1 ≤ 2h(T )−1.Rezulta ca 2h(T )−1 ≤ n < n + 1 s, i 2h(T ) ≥ n + 1, deci h(T )− 1 < log2(n + 1) ≤ h(T ) s, i astfelh(T ) = dlog2(n+ 1)e .

Conform (4), (3) s, i (5) obt, inem ca∑v∈E(T )

DT (v) = (n+ 1− 2a)(h(T )− 1) + 2a · h(T ) = (n+ 1)h(T ) + 2a− n− 1

= (n+ 1)h(T ) + 2Än+ 1− 2h(T )−1

ä− n− 1 = (n+ 1)h(T ) + n+ 1− 2h(T ),

de unde rezulta a doua egalitate din enunt, .

Propozit, ia 6. Pentru orice arbore binar strict T cu n noduri interne, n ∈ N, avem∑v∈E(T )

DT (v) ≥ (n+ 1) dlog2(n+ 1)e+ n+ 1− 2dlog2(n+1)e.

Mai mult, egalitatea are loc daca s, i numai daca toate nodurile externe sunt situate pe ultimul s, i,eventual, pe penultimul nivel.

Optimalitatea algoritmului de cautare binara 45

Demonstratie. FieM = T | T = arbore binar strict, card(I(T )) = n s, i S(T ) =∑

v∈E(T )DT (v),

∀T ∈M. Fie T ∗ ∈M, T ∗ = (V, F ), astfel ıncat

S(T ∗) = minS(T ) | T ∈M.

Consideram din nou ca nivelul pe care este reprezentata radacina arborelui este numerotat cu 0.Demonstram ca arborele T ∗ nu are noduri externe pe nivelele 0,1, . . ., h(T ∗)− 2 prin reducere laabsurd. Sa presupunem ca arborele T ∗ ar avea un nod extern x pe un nivel i, cu i ≤ h(T ∗)− 2.Fie y un nod intern al lui T ∗ situat pe penultimul nivel, nivelul h(T ∗)− 1, s, i fie y1 s, i y2 cei doidescendent, i ai lui y, deci y1 s, i y2 sunt noduri externe situate pe ultimul nivel, nivelul h(T ∗).Evident, DT ∗(x) = i, DT ∗(y) = h(T ∗)− 1, DT ∗(y1) = DT ∗(y2) = h(T ∗).

Fie T ′ arborele obt, inut din T ∗ prin mutarea nodurilor y1 s, i y2 de la descendent, i ai nodului yla descendent, i ai nodului x, adica

T ′ = (V, F \ [y, y1], [y, y2] ∪ [x, y1], [x, y2]).

Evident, T ′ ramane un arbore binar strict, T ′ ∈ M, E(T ′) = E(T ∗) \ x ∪ y,DT ′(y1) = DT ′(y2) = i + 1, DT ′(v) = DT ∗(v), ∀ v ∈ E(T ′) \ y1, y2. Avem S(T ′) − S(T ∗) =∑v∈E(T ′)

DT ′(v)− ∑v∈E(T ∗)

DT ∗(v) = DT ′(y) +DT ′(y1) +DT ′(y2)−DT ∗(x)−DT ∗(y1)−DT ∗(y2) =

h(T ∗)− 1 + 2(i+ 1)− i− 2h(T ∗) = i+ 1− h(T ∗) < 0, deci S(T ′) < S(T ∗), ceea ce contrazicealegerea lui T ∗. Demonstrat, ia prin reducere la absurd este ıncheiata.

Deoarece arborele T ∗ nu are noduri externe pe nivelele 0,1, . . ., h(T ∗) − 2, rezulta catoate nodurile sale externe sunt situate pe ultimul s, i, eventual, pe penultimul nivel. AplicandPropozit, ia 5 rezulta ca

minS(T ) | T ∈M = s(T ∗) = (n+ 1) dlog2(n+ 1)e+ n+ 1− 2dlog2(n+1)e,

de unde rezulta inegalitatea din enunt, . Mai mult, daca toate nodurile externe sunt situate peultimul s, i, eventual, pe penultimul nivel atunci, conform Propozit, iei 5, inegalitatea din enunt,devine egalitate, iar ın caz contrar, conform demonstrat, iei prin reducere la absurd de mai sus,inegalitatea este stricta.

Cautarea binara

Consideram problema cautarii unei valori date x printre elementele unui vector sortat crescatordat A = (a1, a2, . . . , an), a1 ≤ a2 ≤ . . . ≤ an, n ∈ N∗.

Algoritmul de cautare binara rezolva aceasta problema prin urmatoarea strategie de tipDivide et Impera:

• Se compara x cu elementul median am, unde m =ön+12

ù;

• Avem trei variante posibile:

– Daca x = am, atunci cautarea se ıncheie cu succes;

– Daca x < am, atunci algoritmul continua cu cautarea lui x ın subvectorul (a1, . . . , am−1);

– Daca x > am, atunci algoritmul continua cu cautarea lui x ın subvectorul (am+1, . . . , an).

46 C. Balcau

Obt, inem astfel urmatoarea varianta iterativa a algoritmului:

CAUT-BIN(A, n, x) :p← 1; u← n; // (ap, . . . , au) reprezinta subvectorul curent

// la care s-a restrans cautareawhile p ≤ u do

m←õp+ u

2

û;

if x = am thenreturn m; // cautarea se ıncheie cu succes

elseif x < am then

u← m− 1;else

p← m+ 1;

return −1; // cautarea se ıncheie fara succes

Observat,ia 4. Pentru evaluarea complexitat, ii algoritmilor care rezolva problema considerata,vom analiza numai comparat, iile ın care intervin valoarea cautata x s, i un element al vectoruluiA, numite comparat,ii de chei. Astfel consideram ca aceste comparat, ii se efectueaza succesiv(comparat, iile simultane pot fi separate).

Prezentam cateva not, iuni ajutatoare.

Definit, ia 9. Un arbore binar de cautare (BST - Binary Search Tree) este un arborebinar cu proprietatea ca orice nod intern are valoarea (eticheta) mai mare decat orice nod dinsubarborele sau stang si mai mica decat orice nod din subarborele sau drept.

Observat,ia 5. Fie A un algoritm de cautare, bazat pe comparat, ii de chei, ce rezolva problemaconsiderata. Presupunem ca algoritmul nu cont, ine instruct, iuni redundante s, i este aplicabilpentru orice pozit, ionare posibila a valorii x ın raport cu componentele vectorului A, deci valoareax este comparata exact cate o data cu fiecare componemta ai, i = 1, n.

Atunci algoritmului A i se poate asocia un arbore binar de cautare T (A) avand n noduri,etichetate cu numerele 1, 2, . . . , n, construit recursiv astfel: daca prima comparat, ie efectuata dealgoritm este ıntre valoarea cautata x s, i un element ai al vectorului, atunci:

• radacina arborelui este etichetata cu indicele i;

• subarborele stang corespunde continuarii algoritmului ın cazul x < ai, daca acest caz esteposibil dupa prima comparat, ie (altfel este vid, adica nu exista descendent stang);

• subarborele drept corespunde continuarii algoritmului ın cazul x > ai, daca acest caz esteposibil dupa prima comparat, ie (altfel este vid, adica nu exista descendent drept).

Reciproc, oricarui arbore binar de cautare T avand n noduri, etichetate cu numerele1, 2, . . . , n, ıi corespunde un algoritm de cautare A(T ) a valorii x ın vectorul sortat

Optimalitatea algoritmului de cautare binara 47

A = (a1, a2, . . . , an), descris recursiv astfel:

A(T ) :ifîT este vid

óthen

return −1; // cautare fara succeselse

i← eticheta radacinii lui T ;if x = ai then

return i; // cautare cu succeselse

if x < ai thenTs ← subarborele stang al lui T ;return A(Ts);

elseTd ← subarborele drept al lui T ;return A(Td);

Corespondent,ele A → T (A) s, i T → A(T ) de mai sus sunt bine definite s, i inverse unaceleilalte, deci sunt biject, ii ıntre mult, imea algoritmilor de cautare (bazat, i pe comparat, ii dechei) a unei valori x ıntr-un vector sortat cu n componente (a1, a2, . . . , an) s, i mult, imea arborilorbinari de cautare avand n noduri etichetate cu numerele 1, 2, . . . , n.

Definit, ia 10. Arbore binar de cautare T (A) construit ın observat,ia anterioara se numes,te s,iarborele de decizie al algoritmului de cautare A.

Exemplul 2. Arborele binar de cautare asociat algoritmului cautarii binare pentru n = 8 estereprezentat ın figura urmatoare.

4

2 6

1 3 5 7

8

Observat,ia 6. Arborele de cautare asociat algoritmului de cautare binara este un arbore binar,nu neaparat strict.

Definit, ia 11. Fie T un arbore binar de cautare avand n noduri, etichetate cu numerele1, 2, . . . , n. Extindem arborele binar T la un arbore binar strict T astfel: pentru oricek ∈ 1, 2, . . . , n, nodul lui T avand eticheta k este reprezentat ıntr-un cerc s,i devine nodintern ın T prin aplicarea urmatoarelor doua reguli:

• daca nodul nu are descendent stang, atunci i se adauga un descendent stang etichetat cuk − 1 s, i reprezentat ıntr-un patrat;

• daca nodul nu are descendent drept, atunci i se adauga un descendent drept etichetat cu ks, i reprezentat ıntr-un patrat.

48 C. Balcau

Arborele T se numes,te arbore binar extins de cautare.

Exemplul 3. Arborele binar extins de cautare asociat algoritmului cautarii binare pentru n = 8este reprezentat ın figura urmatoare.

4

2 6

1 3 5 7

0 1 2 3 4 5 6 8

7 8

Observat,ia 7. Pentru orice algoritm de cautareA a valorii x ın vectorul sortat A = (a1, a2, . . . , an),arborele binar extins de cautare asociat T = T (A) este un arbore binar strict ce are n noduriinterne, etichetate cu numerele 1, 2, . . . , n reprezentate ın cercuri s, i n + 1 noduri externe,etichetate cu numerele 0, 1, 2, . . . , n reprezentate ın patrate, avand semnificat, ia urmatoare:

• pentru orice i ∈ 1, 2, . . . , n, nodul intern etichetat cu i reprezinta comparat, ia valoriicautate, x, cu elementul ai din vectorul sortat;

• pentru orice i ∈ 0, 1, 2, . . . , n, nodul extern etichetat cu i reprezinta ıncheierea farasucces a cautarii, corespunzatoare situat, iei:

x < a1, daca i = 0,ai < x < ai+1, daca 1 ≤ i ≤ n− 1,

x > an, daca i = n.

De asemenea, avem:

• la o cautare cu succes, x = ai, numarul de comparat, ii de chei este egal cu numarul denoduri (interne) situate pe lant,ul elementar dintre nodul radacina s, i nodul intern i, adicaeste mai mare cu 1 decat lungimea acestui lant, ;

• la o cautare fara succes numarul de comparat, ii de chei este egal cu numarul de noduriinterne situate pe lant,ul elementar dintre nodul radacina s, i nodul extern corespunzatorıncheierii fara succes a cautarii, adica cu lungimea acestui lant, .

Teorema 2 (complexitatea algoritmilor de cautare). Pentru orice algoritm de cautare A,bazat pe comparat,ii de chei, a valorii x ın vectorul sortat A = (a1, a2, . . . , an) avem:

1) numarul de comparat,ii de chei efectuate ın cazul cel mai defavorabil este mai mare sauegal cu dlog2(n+ 1)e ;

2) numarul mediu de comparat,ii de chei efectuate este mai mare sau egal cu

2n+ 2

2n+ 1· dlog2(n+ 1)e+

n+ 2− 2 · 2dlog2(n+1)e

2n+ 1.

Optimalitatea algoritmului de cautare binara 49

Demonstratie. 1) Analizam cazul cel mai defavorabil. Conform Observat, iei 7, numarul decomparat, ii de chei efectuate ın cazul cel mai defavorabil este egal cu ınalt, imea h a arboreluibinar extins de cautare asociat T = T (A). Arborele T este un arbore binar strict, cu n noduriinterne (s, i n+ 1 noduri externe), deci conform Propozit, iei 4 rezulta ca h ≥ dlog2(n+ 1)e .

2) Analizam acum cazul mediu (cazul timpului mediu de execut, ie). Conform Observat, iei 7,numarul mediu de comparat, ii de chei are valoarea

Nmed(n) =

∑k∈I

(1 +D(k)) +∑k∈E

D(k)

2n+ 1=n+

∑k∈I

D(k) +∑k∈E

D(k)

2n+ 1,

unde I este mult, imea nodurilor interne, E este mult, imea nodurilor externe, iar, pentru fiecarenod k, D(k) reprezinta distant,a de la radacina arborelui T = T (A) la nodul k. ConformPropozit, iilor 3 s, i 6 rezulta ca

Nmed(n) =2∑k∈E

D(k)− n

2n+ 1≥

2Ä(n+ 1) dlog2(n+ 1)e+ n+ 1− 2dlog2(n+1)e

ä− n

2n+ 1,

adica inegalitatea din enunt, .

Propozit, ia 7. Pentru algoritmul de cautare binara ıntr-un vector sortat cu n componente, ocautare cu succes necesita cel mult h comparat,ii de chei, iar o cautare fara succes necesita h− 1sau h comparat,ii de chei, unde h = dlog2(n+ 1)e = blog2 nc+ 1.

Demonstratie. Fie T (n) arbore binar de cautare asociat algoritmului de cautare binara ın-tr-un vector sortat cu n componente. Conform descrierii algoritmului de cautare binaras, i a construirii arborelui binar de cautare asociat, rezulta ca subarborele stang al lui T (n)corespunde cautarii binare ın subvectorul (a1, . . . , am−1), unde m =

ön+12

ù, deci este chiar

TÄö

n−12

ùä, iar subarborele drept corespunde cautarii binare ın subvectorul (am+1, . . . , an) de

lungime n − m = n −ön+12

ù=†n−12

£, deci are aceeas, i structura cu T

Ćn−12

£ä(doi arbori

binari au aceeas, i structura daca sunt identici, cu except, ia etichetelor, adica prin suprapunerearadacinilor ei se suprapun perfect). Aplicand metoda induct, iei matematice, rezulta ca arborelebinar T (n + 1) are aceeas, i structura cu arborele binar T (n) plus un nod extern, situat peultimul nivel (la pasul inductiv se analizeaza separat cazurile n par, respectiv impar s, i se aplicaipoteza de induct, ie pentru cei doi subarbori). Cum

†n−12

£=ön−12

ùpentru n impar, respectiv†

n−12

£=ön−12

ù+1 pentru n par, rezulta ca subarborele drept al lui T (n), avand aceeas, i structura

cu TĆ

n−12

£ä, are aceeas, i structura cu subarborele stang T

Äön−12

ùäplus un eventual nod extern

(cand n este par). Aplicand din nou metoda induct, iei matematice, rezulta ca ın arborele binarT (n) toate nodurile care nu au doi descendent, i (fii) sunt situate pe ultimul s, i, eventual, pepenultimul nivel (la pasul inductiv se foloses,te relat, ia de structura ment, ionata dintre cei doisubarbori s, i se aplica ipoteza de induct, ie pentru subarborele drept).

Rezulta ca arborele binar extins de cautare asociat T (n) are nodurile externe situate pecel mult doua nivele. Notand cu h ınalt, imea arborelui extins (adica lungimea maxima de laradacina la un nod extern) s, i considerand ca radacina arborelui este reprezentata pe nivelul 0,rezulta ca: arborele extins are h+ 1 nivele, n noduri interne s, i n+ 1 noduri externe; pe nivelele0, 1, . . . , h− 2 avem doar noduri interne, fiecare avand cate doi descendent, i; pe penultimul nivel,h− 1, putem avea s, i noduri externe, iar nodurile interne de pe acest nivel au de asemenea catedoi descendent, i; pe ultimul nivel, h, avem doar noduri externe.

50 C. Balcau

Conform Propozit, iei 5 s, i Observat, iei 3 rezulta ca h = dlog2(n+ 1)e = blog2 nc+ 1. ConformObservat, iei 7 rezulta ca o cautare cu succes necesita cel mult h comparat, ii de chei, iar o cautarefara succes necesita h− 1 sau h comparat, ii de chei.

Observat,ia 8. Conform propozit, iei anterioare, algoritmul de cautare binara are timpul de execut, ieın cazul cel mai defavorabil de ordinul Θ(log2 n) (celelalte operat, ii nu depas,esc ordinul de cres,tereal comparat, iilor de chei), deci timpul (mediu) de execut, ie are ordinul O (log2 n). Teoremaurmatoare ımbunatat,es,te acest rezultat.

Teorema 3. Fie T (n) timpul mediu de execut,ie al algoritmului de cautare binara ıntr-un vectorsortat cu n componente. Avem T (n) = Θ(log2 n).

Demonstratie. Consideram ca timpul mediu de execut, ie T (n) al algoritmului de cautare binaraeste dat de numarul de comparat, ii de chei efectuate (celelalte operat, ii nu depas,esc ordinul decres,tere al acestora). Vom demonstra ca T (n) = Θ(log2 n) prin doua metode.

Metoda 1) Vom aplica Teorema Master. Conform descrierii recursive a algoritmului decautare binara, timpul de execut, ie T (n) verifica relat, ia de recurent, a

T (n) = T (n/2) +O (1), ∀n > 0,

cu convent, iile de notat, ie din Teorema Master. Conform Cazului 2 al acelei teoreme, pentrua = 1, b = 2, c = 0 = logb a s, i k = 0, rezulta ca T (n) = Θ(log2 n).

Observat,ia 9. Ca o consecint, a imediata a Metodei 1, Propozit, iei 7 s, i Teoremei 2 rezulta caalgoritmul de cautare binara este asimptotic optim (ın clasa algoritmilor bazat, i pe comparat, iide chei, atat ın raport cu timpul de execut, ie ın cazul cel mai defavorabil, cat s, i ın raport cutimpul mediu de execut, ie). Pentru a justifica faptul ca algoritmul este chiar optim, nu estesuficienta evaluarea complexitat, ii cu Teorema Master. Aici intervine metoda urmatoare, princare se numara comparat, iile efectuate de algoritm.

Metoda 2) Vom calcula efectiv timpul mediu de execut, ie T (n). Conform demonstrat, ieiPropozit, iei 7, arborele binar extins de cautare asociat algoritmului cautarii binare este un arborebinar strict cu n noduri interne s, i n+ 1 noduri externe, iar toate nodurile externe sunt situatepe ultimul s, i, eventual, pe penultimul nivel. Notam cu I mult, imea nodurilor interne, cu Emult, imea nodurilor externe, iar pentru fiecare nod k notam cu D(k) distant,a de la radacinaarborelui la nodul k. Conform Observat, iei 7, timpul mediu de execut, ie are valoarea

T (n) =

∑k∈I

(1 +D(k)) +∑k∈E

D(k)

2n+ 1=n+

∑k∈I

D(k) +∑k∈E

D(k)

2n+ 1.

Conform Propozit, iilor 3 s, i 5 rezulta ca

T (n) =2∑k∈E

D(k)− n

2n+ 1=

2n+ 2

2n+ 1· dlog2(n+ 1)e+

n+ 2− 2 · 2dlog2(n+1)e

2n+ 1. (6)

Dar, conform Observat, iei 3, dlog2(n+ 1)e = 1 + blog2 nc , deci

T (n) =2n+ 2

2n+ 1· blog2 nc+

3n+ 4− 4 · 2blog2 nc

2n+ 1.

Optimalitatea algoritmului de cautare binara 51

Evident, log2 n− 1 < blog2 nc ≤ log2 n, decin

2< 2blog2 nb ≤ n. Rezulta ca

log2 n−3n− 2− log2 n

2n+ 1< T (n) < log2 n+

n+ 4 + log2 n

2n+ 1, deci

1− 3n− 2− log2 n

(2n+ 1) log2 n<

T (n)

log2 n< 1 +

n+ 4 + log2 n

(2n+ 1) log2 n, ∀n ≥ 2.

Aplicand Criteriul cles,telui rezulta ca

limn→∞

T (n)

log2 n= 1, (7)

deci conform Propozit, iei 1 obt, inem ca T (n) = Θ(log2 n).

Mai mult, conform relat, iei (7) am obt, inut urmatorul rezultat.

Corolarul 2. Numarul mediu de comparat,ii de chei efectuate de algoritmul de cautare binaraıntr-un vector sortat cu n componente este asimptotic echivalent cu log2 n.

Teorema 4 (optimalitatea algoritmului de cautare binara). Algoritmul de cautare binaraeste optim (ın clasa algoritmilor bazat,i pe comparat,ii de chei), atat ın raport cu timpul de execut,ieın cazul cel mai defavorabil, cat s, i ın raport cu timpul mediu de execut,ie.

Demonstratie. Optimalitatea ın raport cu timpul de execut, ie ın cazul cel mai defavorabil este oconsecint, a directa a Propozit, iei 7 s, i a primei part, i din Teorema 2. Optimalitatea ın raport cutimpul mediu de execut, ie este o consecint, a directa a relat, iei (6) din demonstrat, ia Teoremei 3 s, ia part, ii secunde din Teorema 2.

Bibliografie

[1] Gh. Barbu, I. Vaduva, M. Bolos,teanu, Bazele informaticii, Editura Tehnica, Bucures,ti, 1997.

[2] A. Carabineanu, Structuri de date, http://ebooks.unibuc.ro/informatica/carabineanu/CARA STR.pdf.

[3] T.H. Cormen, C.E. Leiserson, R.L. Rivest, C. Stein, Introduction to Algorithms, MIT Press,Cambridge, 2009.

[4] H. Georgescu, Tehnici de programare, Editura Universitat, ii din Bucures,ti, Bucures,ti, 2005.

[5] D.E. Knuth, The Art Of Computer Programming. Vol. 4A: Combinatorial Algorithms,Addison-Wesley, Massachusetts, 2011.

[6] R. Sedgewick, P. Flajolet, An Introduction to the Analysis of Algorithms, Addison-Wesley,New Jersey, 2013.

Perspective profesionale pentru pasionat, ii de programare

ROWEB Development 1

In contextul actual al ınvat, amantului superior din Romania numarul de absolvent, i deInformatica este de aproximativ 7000 pe an, iar necesarul de pe piat,a muncii este mult maimare. Comisia Europeana estimeaza ca, pana ın anul 2020, Europa s-ar putea confrunta cuun deficit de pana la 825.000 de profesionis,ti ın IT&C, dintre care aproximativ 300.000 vor finecesari pentru a acoperi cres,terea prognozata a domeniului de software din Romania.

Discrepant,a dintre cerere s, i oferta reprezinta deja o provocare, pe care atat companiileproducatoare de software, cat s, i universitat, ile ıncearca sa o rezolve, adesea ın colaborare. Unelefacultat, i de informatica ıs, i dubleaza deja de la un an la altul numarul de locuri, iar unelecompanii ofera training sau colaboreaza cu universitat, ile pentru a-s, i asigura viitori angajat, i.

O alta provocare o reprezinta potrivirea dintre limbajele de programare predate la nivelulliceelor s, i facultat, ilor, s, i tehnologiile cerute pe piat, a. Stagiile de vara, internship-urile ın cadrulcompaniilor cu experient, a, precum s, i concursurile de programare urmaresc ımbunatat, irea acestuiaspect. Infoarena reunes,te informat, ii despre cele mai importante concursuri, inclusiv:

a) .campion - un program de pregatire de performant, a ın informatica sust, inut de profesori deprestigiu din t,ara, membri ai Comisiei Nat, ionale de Informatica, s, i de student, i - fos,ti olimpiciinternat, ionali la informatica;

b) TopCoder - un concurs ce pune accent ın primul rand pe viteza s, i acuratet,e, apoi pedificultate;

c) IPSC - Internet Problem Solving Contest - cel mai prestigios concurs online pe echipes, i totodata primul concurs care nu pune accentul pe surse de solut, ii, ci pe output-uri pentrutestele publicate;

d) Waterloo Programming Contests - concursuri de pregatire pentru ACM organizate decatre Universitatea Waterloo din Canada;

e) ACM - University of Vallaloid, ACM - Peking University s, i ACM - Zhejiang Universityorganizeaza concursuri lunare;

f) ACM ICPC International Collegiate Programming Contest - cel mai prestigios concurs deprogramare/algoritmica pentru universitat, i;

g) CodeCup - Concurs anual organizat de catre The Netherlands Olympiad in Informatics;

h) Bubble Cup - Concurs organizat ın Serbia, cu sprijinul Microsoft Development CenterSerbia.

Concursurile stimuleaza student, ii sa ıs, i dezvolte un nivel ınalt al cunos,tint,elor s, i abilitat, ilorın programare, ınsa pentru a-i ajuta sa-s, i faca o idee asupra piet,ei muncii ın care urmeaza salucreze, este nevoie de implicarea companiilor.

[email protected], [email protected]

52

Perspective profesionale... 53

ROWEB este un exemplu de companie care investes,te ın astfel de eforturi, remarcandu-se ınPites,ti printr-o colaborare cu Universitatea ce dureaza de peste 12 ani. Fondatorii ROWEB auabsolvit Universitatea din Pites,ti s, i ınca din primii ani de la ınfiint,are au atras tineri student, is, i absolvent, i talentat, i, care s-au bucurat de aparit, ia unei firme care le permitea sa-s, i urmezepasiunea pentru programare fara sa fie nevoie sa paraseasca Pites,ti-ul ın favoarea unui centrude software mai mare din t,ara.

Colaborarea dintre ROWEB s, i UPIT a adus beneficii comunitat, ii universitare locale s, ia ajutat multe tinere talente sa aiba mai us,or acces la o cariera ca programatori. Aceastacolaborare s-a concretizat prin workshop-uri gratuite de Android, HTML s, i PHP, sust, inere ınjurizarea unor concursuri de programare, precum s, i stagii de vara de care au beneficiat sute destudent, i. Stagiile se desfas,oara cu succes ınca de acum 9 ani s, i ın fiecare an ajuta cateva zecide student, i sa se familiarizeze cu dezvoltarea de aplicat, ii ın .NET, PHP sau cu dezvoltarea deaplicat, ii mobile. Mai mult, i stagiari au ramas ulterior ca angajat, i full-time, aproximativ 90%din angajat, ii firmei fiind fos,ti stagiari.

Stagiile de vara permit student, ilor sa lucreze ımpreuna cu dezvoltatorii seniori, foarteprietenos, i, entuzias,ti sa-s, i ımpartas,easca experient,a s, i cunos,tint,ele, s, i sa puna ın practicacunos,tint,ele acumulate la facultate, capatand nu doar experient, a profesionala, ci s, i trainingintensiv din partea unor specialis,ti care au o reputat, ie foarte buna pentru expertiza lor ın celemai recente tehnologii s, i pentru aderarea la cele mai bune practici de lucru.

Un stagiu de practica la ROWEB reprezinta o oportunitate de pregatire a student, ilor pentruintrarea pe piat,a muncii, dar ın acelas, i timp le ofera acestora s,ansa de a pune ın practica toatecunos,tint,ele dobandite ın anii de studiu, ajutandu-i sa se familiarizeze cu cerint,ele concrete dinpiat,a de IT.

Insa ın IT nu este suficient sa dezvolt, i noi talente s, i sa atragi proaspet, ii absolvent, i. Retent, iaangajat, ilor a devenit o provocare ın ultimii ani, ın condit, iile disparitat, ii dintre cerere s, i ofertade munca. Salariul nu mai este de mult principalul diferent, iator pe care ıl are ın vedere uncandidat bun cand alege compania pentru care sa lucreze. Acesta ajunge acum sa se uite laalte aspecte, cum sunt cele legate de cultura organizat, ionala, dezvoltare de cariera s, i personala,de posibilitatea unui program de lucru flexibil, posibilitatea de a lucra de acasa etc. Tineriiprogramatori ıs, i doresc s, i au posibilitatea sa aleaga o companie care sa le ımpartas,easca valoriles, i sa le asigure transparent,a s, i un mediu de lucru placut, ın care ideile sa le fie ascultate s, iapreciate.

ROWEB are ın vedere aceste aspecte prin posibilitatea de a se realoca dintr-o linie de businessın alta prin intermediul programului de mobilitate interna, acces la cursuri de programare,de limba engleza, la o biblioteca de specialitate s, i prin dezvoltarea s, i pastrarea unei culturiorganizat, ionale etice s, i incluzive, care asigura angajat, ilor un echilibru ıntre viat,a personala s, iprofesionala s, i beneficii care le permit sa progreseze profesional s, i sa ıs, i ıntemeieze s, i sust, inafamilii ın acelas, i timp, ceea ce reprezinta o diferent, a importanta mai ales pentru programatoare,care ın alte companii sunt adesea nevoite sa aleaga ıntre evolut, ia profesionala s, i decizia de aavea copii.

Alte criterii importante sunt amploarea s, i complexitatea proiectelor, accentul pus maidegraba pe proiecte de dezvoltare, care implica tehnologii noi, decat pe cele de mentenant, a, ceeace permite angajat, ilor sa ınvet,e ın permanent, a s, i sa aiba de-a face cu provocari noi ın domeniu.

In condit, iile ın care, conform estimarilor ANIS, anul acesta industria de software s, i serviciiIT din Romania va reprezenta 3% din PIB, iar nevoia de a forma specialis,ti ıntr-un ritm care sa

54 ROWEB Development

t, ina pasul cu cel al cererii exprimate este din ce ın ce mai evidenta ın piat, a, tinerii care alegspecializarea informatica au provocari tot mai mari ın fat, a, trebuie sa se adapteze rapid s, i sa fiemereu la curent cu cele mai noi tehnologii pentru a fi competitivi, dar ın acelas, i timp beneficiazade posibilitat, i tot mai mari de dezvoltare, de compensare salariala s, i de a alege angajatorul carele ındeplines,te cel mai bine cerint,ele.

MATEMATICA PENTRU PROGRAMATORI S, I

PROGRAMARE PENTRU MATEMATICIENI

Rezolvarea sistemelor de ecuat, ii liniare

Laurent, iu Deaconu 1

Ne propunem sa realizam o aplicat, ie (un program) care sa rezolve un sistem de ecuat, ii liniare.Nu insistam asupra sistemelor de ecuat, ii liniare compatibile determinate (pentru care rangulmatricei sistemului este egal cu rangul matricei extinse s, i cu numarul de necunoscute) pentrucare exista multe metode simple de rezolvare (inclusiv metoda Cramer) care se pot transfera ınalgoritmi informatici, ci prezentam o metoda care permite atat stabilirea compatibilitat, ii cat s, irezolvarea sistemelor compatibile nedeterminate.

Argumentare matematica

Consideram sistemul de ecuat, ii liniarea11x1+ a12x2+ · · ·+ a1nxn =b1a21x1+ a22x2+ · · ·+ a2nxn =b2

...am1x1+am2x2+ · · ·+amnxn =bm

, (1)

cu aij, bi ∈ R, i ∈ 1,m, j ∈ 1, n, m,n ∈ N?.

Algoritmul folosit pentru rezolvarea sistemului are la baza lema substitut, iei. Pentru simplifi-carea calculelor vom organiza rezolvarea transferand sistemul ıntr-un tabel. In mod normal,daca sistemul este compatibil, algoritmul are m pas, i. La pasul k (0 ≤ k ≤ m) tabelul areurmatoarea structura:

x1 x2 . . . xl . . . xn

j[k]1 j

[k]2 . . . j

[k]l . . . j[k]n

i[k]1 a

[k]11 a

[k]12 . . . a

[k]1l . . . a

[k]1n b

[k]1

i[k]2 a

[k]21 a

[k]22 . . . a

[k]2l . . . a

[k]2n b

[k]2

......

.... . .

.... . .

......

i[k]k a

[k]k1 a

[k]k2 . . . a

[k]kl . . . a

[k]kn b

[k]k

0 a[k]k+1,1 a

[k]k+1,2 . . . a

[k]k+1,l . . . a

[k]k+1,n b

[k]k+1

......

.... . .

.... . .

......

0 a[k]m′1 a

[k]m′2 . . . a

[k]m′l . . . a

[k]m′n b

[k]m′

1Lect. univ. dr., Universitatea din Pitesti, [email protected]

55

56 L. Deaconu

Elementele tabelului vor fi precizate ın cadrul algoritmului.

Pasul 1 Generarea tabelului. Primul tabel se completeaza astfel:

1. Prima linie x1, x2, . . . , xn este doar o notat, ie pentru coloanele asociate necunoscutelorsistemului. Ea nu se modifica pe parcursul algoritmului.

2. A doua linie j1, j2, . . . , jn se completeaza cu 0, jl = 0, ∀l ∈ 1, n. Ea se completeaza cuelemente nenule la fiecare pas.

3. Coloana din stanga cu elementele i1, i2, . . . , im′ are init, ial toate elementele nule, ik = 0,∀k ∈ 1,m′. La primul pas avem m′ = m. Se completeaza cu elemente nenule la fiecarepas al algoritmului.

4. Zona centrala cont, ine elementele matricei sistemului. La primul tabel (k = 0) avem

a[0]ij = aij, ∀i ∈ 1,m, ∀j ∈ 1, n.

5. Coloana din dreapta se completeaza cu termenii liberi ai sistemului. Avem b[0]i = bi,

∀i ∈ 1,m.

Pasul 2 Prelucrarea tabelului. La pasul k, (k ∈ 1,m′), parcurgem urmatoarele etape:

1. Cautam, ın linia a[k−1]k1 , a

[k−1]k2 , . . . , a

[k−1]kl , . . . , a

[k−1]kn din zona centrala, un element nenul.

Daca gasim, ıl numim pivot, identificam coloana ın care se afla (notata xl ın tabelul de

mai sus) s, i trecem la pasul urmator. Daca avem a[k−1]kq = 0, ∀q ∈ 1, n, atunci:

(a) daca b[k−1]k = 0 eliminam linia k deoarece corespunde unei ecuat, ii care este o

combinat, ie liniara a ecuat, iilor precedente s, i care nu conteaza ın rezolvarea sistemului.In acest caz numarul ecuat, iilor se mics,oreaza cu 1, m′ ← m′ − 1. Daca avem k ≤ m′,reluam etapa 1, altfel trecem la Pasul 3.

(b) daca b[k−1]k 6= 0 algoritmul se opres,te. Sistemul este incompatibil.

2. In coloana din stanga, punem i[k]k = l, unde xl este coloana ın care a fost identificat pivotul.

Celelalte elemente raman ca la pasul precedent, i[k]p = i[k−1]p , ∀p ∈ 1, 2, . . . ,m′ \ k.3. In linia a doua, punem j

[k]l = k s, i j[k]q = j[k−1]q , ∀q ∈ 1, 2, . . . , n \ l.

4. Elementele din linia care cont, ine pivotul se ımpart la pivot:

a[k]kq =

a[k−1]kq

a[k−1]kl

, ∀q ∈ 1, n, b[k]k =

b[k−1]k

a[k−1]kl

.

Evident a[k]kl = 1.

5. Celelalte elemente din coloana xl, ın care se afla pivotul, vor fi egale cu 0, a[k]pl = 0,

∀p ∈ 1, 2, . . . ,m′ \ k.6. Toate elementele care nu au fost ınlocuite ın etapele 2-5 se determina folosind regula

dreptunghiului:

a[k]pq =a[k−1]pq a

[k−1]kl − a[k−1]kq a

[k−1]pl

a[k−1]kl

= a[k−1]pq −a[k−1]kq a

[k−1]pl

a[k−1]kl

,

∀p ∈ 1, 2, . . . ,m′ \ k, ∀q ∈ 1, 2, . . . , n \ l.7. Daca k < m′, cres,tem valoarea lui k cu 1 s, i reluam Pasul 2, altfel trecem la Pasul 3.

Pasul 3 Recuperarea solut, iei. Sistemul este compatibil:

1. determinat, daca m′ = n. Solut, ia este xl = b[m′]

j[m′]l

, 1 ≤ l ≤ n.

Rezolvarea sistemelor de ecuat, ii liniare 57

2. nedeterminat, daca m′ < n. Construim mult, imea S = l ∈ N | j[m′]

l = 0, 1 ≤ l ≤ n.Parcurgem a doua linie, j

[m′]1 , j

[m′]2 , . . . , j[m

′]n :

(a) daca j[m′]l = 0, 1 ≤ l ≤ n, atunci xl este necunoscuta secundara s, i avem xl = αl, unde

αl ∈ R este un parametru.

(b) daca j[m′]l 6= 0, 1 ≤ l ≤ n, atunci xl = b

[m′]

j[m′]l

−∑q∈S

a[m′]

j[m′]l

qαq.

Exemplu de calcul

Consideram sistemul

x1+2x2+3x3+ 4x4+ 5x5 = 6

2x1+ x2+3x3+ 2x4+ x5 = 8

x1+5x2+6x3+10x4+14x5 =10

2x1+ x2+6x3+ 5x4+ 4x5 = 3

, (2)

Pentru rezolvare utilizam algoritmul prezentat mai sus. Mai ıntai scriem tabelul init, ial.

x1 x2 x3 x4 x5

0 0 0 0 0

0 1 2 3 4 5 6

0 2 1 3 2 1 8

0 1 5 6 10 14 10

0 2 1 6 5 4 3

In linia corespunzatoare primei ecuat, ii toate elementele sunt nenule, deci oricare elementpoate fi ales ca pivot. Pentru a evita calculele complicate, alegem ca pivot primul element, pe 1,elementul ıncadrat ıntr-un patrat ın tabelul anterior. Prelucram primul tabel:– ın coloana din stanga, ın linia pivotului, punem 1 deoarece am ales pivotul ın coloana lui x1;restul elementelor se transcriu din primul tabel;– ın linia de deasupra zonei centrale, ın coloana pivotului, sub x1, punem 1 deoarece pivotul segases,te ın linia 1 din zona centrala, linia corespunzatoare primei ecuat, ii; restul elementelor setranscriu din primul tabel;– linia pivotului se ımparte la pivot, adica se transcrie deoarece pivotul este egal cu 1;– coloana pivotului se completeaza cu zero-uri;– restul elementelor se calculeaza cu regula dreptunghiului; de exemplu, elementul din linia 4 s, icoloana 3 din zona centrala (care este 6 ın primul tabel) se calculeaza astfel:

a[1]43 =

6 · 1− 2 · 31

= 0,

6 este elementul corespunzator din tabelul anterior, 1 este pivotul, 2 s, i 3 sunt ”proiect, iile”elementului ınlocuit pe coloana, respectiv linia, pivotului. Termenii liberi se calculeaza ın acelas, imod: elementul din linia 3 din coloana termenilor liberi se calculeaza astfel

b[1]3 =

10 · 1− (1) · 61

= 4.

Obt, inem tabelul urmator:

58 L. Deaconu

x1 x2 x3 x4 x5

1 0 0 0 0

1 1 2 3 4 5 6

0 0 −3 −3 −6 −9 −4

0 0 3 3 6 9 4

0 0 −3 0 −3 −6 −9

In linia corespunzatoare ecuat, iei 2 alegem pivotul −3 din coloana lui x2. In coloana dinstanga, ın linia pivotului punem 2 deoarece pivotul este ın coloana lui x2, iar ın linia de deasuprazonei centrale, sub x2, punem 2 deoarece pivotul se gases,te ın linia 2 din zona centrala. Restulelementelor se completeaza ca la pasul precedent. Obt, inem tabelul:

x1 x2 x3 x4 x5

1 2 0 0 0

1 1 0 1 0 −1 103

2 0 1 1 2 3 43

0 0 0 0 0 0 0

0 0 0 3 3 3 −5

Observam ca ın linia 3 din zona centrala, ın coloanele corepunzatoare necunoscutelor toateelementele sunt egale cu 0, deci nu putem alege un pivot. Elementul corespunzator din coloanatermenilor liberi este tot 0, deci putem renunt,a la ecuat, ia 3. Eliminam aceasta linie s, i obt, inemtabelul:

x1 x2 x3 x4 x5

1 2 0 0 0

1 1 0 1 0 −1 103

2 0 1 1 2 3 43

0 0 0 3 3 3 −5

In tabelul anterior alegem pivotul 3 din ultima linie s, i coloana lui x3.

x1 x2 x3 x4 x5

1 2 3 0 0

1 1 0 0 −1 −2 5

2 0 1 0 1 2 3

3 0 0 1 1 1 −53

Analizand ultimul tabel deducem ca sistemul este compatibil nedeterminat. Necunoscu-tele principale sunt x1, x2, x3, iar necunoscutele secundare x4 s, i x5. Asociem paramatri realinecunoscutelor secundare: x4 = α, x5 = β.

Pentru simplificarea identificarii solut, iei putem completa tabelul anterior astfel:

α β

x1 x2 x3 x4 x5

1 2 3 0 0

1 1 0 0 −1 −2 5

2 0 1 0 1 2 3

3 0 0 1 1 1 −53

Rezolvarea sistemelor de ecuat, ii liniare 59

Pentru x1, ın linia de deasupra zonei centrale gasim 1. Din linia corespunzatoare, adicalinia 1 recuperam x1: din elementul din coloana termenilor liberi, 5, se scad produsele dintreelementele din coloanele corespunzatoare parametrilor s, i respectivii parametri, adica −1 · α s, i−2 · β. Procedam similar pentru necunoscutele x2 s, i x3. Obt, inem astfel solut, ia:

x1 = 5+α+2β

x2 = 3−α−2β

x3 =−53−α −β

x4 = α

x5 = β

, (3)

Implementare ın limbajul C++

Algorimul prezentat ın acest articol permite realizarea unui program care

• determina daca un sistem de ecuat, ii liniare este compatibil sau incompatibil

• identifica daca un sistem compatibil este compatibil determinat sau compatibil nedetermi-nat

• pentru sistemele compatibile determinate calculeaza solut, ia

• pentru sistemele compatibile nedeterminate determina solut, ia generala, ın funct, ie deparametri.

Programul prezentat mai jos, realizat ın Code::Blocks, cites,te datele din fis, ierul textsistem.txt s, i scrie rezultatele ın fis, ierul text solutie.txt.

Fis, ierul prezentat mai jos (sistem.txt) cont, ine datele corespunzatoare sistemului (2). Primalinie cont, ine m,n, numarul ecuat, iilor, respectiv numarul necunoscutelor. In continuare se scriuelementele matricei extinse de dimensiune m× (n+ 1), ultima coloana cont, inand termenii liberi.

4 51 2 3 4 5 62 1 3 2 1 81 5 6 10 14 102 1 6 5 4 3

Programul respecta algoritmul prezentat, cu urmatoarele precizari:

– Pentru alegerea pivotului se determina elementul din linia curenta care are valoarea absolutamaxima. Daca acesta este nul, deducem ca toate elementele liniei sunt nule. Se analizeaza Pasul 2,etapa 1 (a) s, i (b). Daca elementul determinat este nenul, acesta va fi pivotul (se alege elementul cuvaloare absoluta maxima pentru reducerea erorilor de rotunjire).

– Pentru a verifica daca un numar real este nul, nu ıl comparam cu 0 ci verificam daca valoareaabsoluta a lui este mai mica decat ε, unde ε este un numar real pozitiv foarte mic. In programε = 10−20.

– Prelucrarea tabelului din Pasul 2 se face parcurgand etapele ın ordinea 6, 4, 5, 3, 2.

60 L. Deaconu

#include <iostream>#include <fstream>#include <iomanip>#include <math.h>using namespace std;

ifstream f1("sistem.txt");ofstream f2("solutie.txt");

int m,n,io,jo;double a[20][40],eps=1.0e-20;int o[40],v[20];

void citire()

int i,j;f1>>m>>n;for(i=1;i<=m;i++)for(j=1;j<=n+1;j++)

f1>>a[i][j];f1.close();

void generare_tabel()

int i,j;for(i=1;i<=m;i++)

v[i]=0;for(j=1;j<=n+1;j++)

o[j]=0;

void prelucrare()

int i,j;for(i=1;i<=m;i++)if(i!=io)

for(j=1;j<=n+1;j++)if(j!=jo)

a[i][j]=(a[i][j]*a[io][jo]-a[i][jo]*a[io][j])/a[io][jo];for(j=1;j<=n+1;j++)if(j!=jo)

a[io][j]/=a[io][jo];for(i=1;i<=m;i++)

a[i][jo]=0.0;a[io][jo]=1.0;o[jo]=io;v[io]=jo;

int pivot()

int i,j;double max;max=fabs(a[io][1]);jo=1;for(j=2;j<=n;j++)if(max<=fabs(a[io][j]))

max=fabs(a[io][j]);jo=j;

if(max<eps)

if(fabs(a[io][n+1])<eps)

Rezolvarea sistemelor de ecuat, ii liniare 61

for(i=io;i<m;i++)for(j=1;j<=n+1;j++)

a[i][j]=a[i+1][j];m--;return -1;

else

f2<<"Sistem incompatibil!";io=m+2;return 0;

else

return 1;

void recuperare_solutie()

int j;char c[n];if(io==m+1)

if(m==n)f2<<"Sistemul este compatibil determinat."<<endl;

else

f2<<"Sistemul este compatibil nedeterminat."<<endl;int k=96;for (j=1;j<=n;j++)

if(o[j]==0)c[j]=char(++k);

f2<<"Solutia sistemului este:"<<endl;for(jo=1;jo<=n;jo++)

f2<<"x"<<noshowpos<<jo<<"="<<showpos;if(o[jo]==0)

f2<<c[jo]<<endl;else

io=o[jo];f2<<setw(8)<<fixed<<setprecision(4)<<a[io][n+1];for(j=1;j<=n;j++)if((o[j]==0)&&(fabs(a[io][j]))>eps)

f2<<setw(8)<<fixed<<setprecision(4)<<-a[io][j]<<"*"<<c[j];f2<<endl;

void scrie_tabel()

int i,j;f2<<"======";for(j=1;j<=n+1;j++)

62 L. Deaconu

f2<<"========";f2<<endl;f2<<" |";for(j=1;j<=n;j++)

f2<<setw(5)<<o[j]<<" ";f2<<"|"<<endl;f2<<"------";for(j=1;j<=n+1;j++)

f2<<"--------";f2<<endl;for(i=1;i<=m;i++)

f2<<setw(4)<<v[i]<<"|";for(j=1;j<=n;j++)

f2<<setw(8)<<fixed<<setprecision(3)<<a[i][j];f2<<"|"<<setw(8)<<fixed<<setprecision(3)<<a[i][n+1]<<endl;

f2<<"======";for(j=1;j<=n+1;j++)

f2<<"========";f2<<endl;

int main(void)

int p;citire();if (m==0||n==0)

cout<<"Sistemul nu exista!";return 1;

f2<<"Tabelul initial:"<<endl;generare_tabel();scrie_tabel();io=1;do

p=pivot();if(p==1)

prelucrare();f2<<"Pozitia pivotului: linia "<<io<<" coloana "<<jo<<endl;scrie_tabel();io++;

else

if(p==-1)

f2<<"Eliminare ecuatie - linia "<<io<<endl;scrie_tabel();

while(io<=m);recuperare_solutie();f2.close();return 0;

Pentru sistemul (2) se obt, ine rezultatul din fis, ierul solutie.txt:

Tabelul initial:

Rezolvarea sistemelor de ecuat, ii liniare 63

======================================================| 0 0 0 0 0 |

------------------------------------------------------0| 1.000 2.000 3.000 4.000 5.000| 6.0000| 2.000 1.000 3.000 2.000 1.000| 8.0000| 1.000 5.000 6.000 10.000 14.000| 10.0000| 2.000 1.000 6.000 5.000 4.000| 3.000

======================================================Pozitia pivotului: linia 1 coloana 5======================================================

| 0 0 0 0 1 |------------------------------------------------------

5| 0.200 0.400 0.600 0.800 1.000| 1.2000| 1.800 0.600 2.400 1.200 0.000| 6.8000| -1.800 -0.600 -2.400 -1.200 0.000| -6.8000| 1.200 -0.600 3.600 1.800 0.000| -1.800

======================================================Pozitia pivotului: linia 2 coloana 3======================================================

| 0 0 2 0 1 |------------------------------------------------------

5| -0.250 0.250 0.000 0.500 1.000| -0.5003| 0.750 0.250 1.000 0.500 0.000| 2.8330| 0.000 0.000 0.000 0.000 0.000| 0.0000| -1.500 -1.500 0.000 0.000 0.000| -12.000

======================================================Eliminare ecuatie - linia 3======================================================

| 0 0 2 0 1 |------------------------------------------------------

5| -0.250 0.250 0.000 0.500 1.000| -0.5003| 0.750 0.250 1.000 0.500 0.000| 2.8330| -1.500 -1.500 0.000 0.000 0.000| -12.000

======================================================Pozitia pivotului: linia 3 coloana 1======================================================

| 3 0 2 0 1 |------------------------------------------------------

5| 0.000 0.500 -0.000 0.500 1.000| 1.5003| 0.000 -0.500 1.000 0.500 0.000| -3.1671| 1.000 1.000 -0.000 -0.000 -0.000| 8.000

======================================================Sistemul este compatibil nedeterminat.Solutia sistemului este:x1= +8.0000 -1.0000*ax2=ax3= -3.1667 +0.5000*a -0.5000*bx4=bx5= +1.5000 -0.5000*a -0.5000*b

As,a cum se poate observa din fis, ierul cu rezultate, solut, ia generala obt, inuta prin programpentru sistemul (2) este

x1 = 8 −ax2 = a

x3 =−196

+12a−1

2b

x4 = b

x5 = 32−1

2a−1

2b

, (4)

64 L. Deaconu

In aceasta forma ea difera de solut, ia generala (3) obt, inuta ın exemplul de mai sus. Cele douasolut, ii generale genereaza aceeas, i mult, ime de solut, ii.

Daca identificam necunoscutele x4 s, i x5 din cele doua solut, ii

x4 = α = b s, i x5 = β =3

2− 1

2a− 1

2b,

adica, daca folosim substitut, ia

α = b s, i β =3

2− 1

2a− 1

2b

ın solut, ia (3), regasim solut, ia (4):

x1 =5 + b+ 3− a− b = 8− ax2 =3− b− 3 + a+ b = a

x3 =−55− b− 3

2+ 1

2a+ 1

2b = −19

6+ 1

2a− 1

2b

x4 =b

x5 = 32− 1

2a− 1

2b

.

Determinarea pozit, iilor relative a mai multor drepte ın

plan cu ajutorul mediului Maple

Raluca Mihaela Georgescu 1

Stabilirea pozit, iilor relative a dreptelor determinate de perechi de puncte din plan este oproblema frecvent ıntalnita ın geometria analitica plana.

Rezolvarea acestei probleme necesita destul de mult timp, deoarece presupune aplicareametodelor de determinare a pozit, iilor relative a doua drepte pentru fiecare pereche de drepte.Pentru n puncte distincte din plan avem N = C2

n perechi de puncte care identifica acelas, i numarde drepte. Stabilirea pozit, iilor relative presupune astfel analizarea unui numar destul de mare,C2N , de perechi de drepte.

Mediul de programare Maple ofera posibilitatea realizarii unui program care, pentru fiecarepereche ın parte, determina pozit, ia relativa a dreptelor componente s, i, pentru dreptele concurente,precizeaza masura unghiului dintre ele.

Consideram punctele Pi(xi, yi), cu i ∈ 1, n. Pentru a defini aceste puncte folosim comanda

point(Pi, xi, yi), (1)

pentru fiecare punct Pi cu i ∈ 1, n.

Folosind instruct, iunea for s, i obiectul geometric line construim toate dreptele dij ce trecprin punctele Pi s, i Pj, pentru orice i ∈ 1, n− 1 s, i j ∈ 2, n, j > i:

for i to n-1 dofor j from i+1 to n do

line(d||i||j, [P||i, P||j]) (2)end doend do;

Pentru a verifica daca trei puncte sunt coliniare, sau daca doua drepte sunt paralele sauperpendiculare, se foloses,te una din funct, iile:

• AreCollinear, pentru a verifica daca trei puncte sunt coliniare

• AreParallel, pentru a verifica daca doua drepte sunt paralele

• ArePerpendicular, pentru a verifica daca doua drepte sunt perpendiculare

Aceste trei funct, ii au ca rezultat ”adevarat” sau ”fals”.Masura unghiului dintre doua drepte neparalele se determina cu ajutorul funct, iei FindAngle.Daca unghiul dintre cele doua drepte nu este nul sau drept, atunci rezultatul va fi de forma”arctan(x)”. In aceasta situat, ie, pentru a converti ın grade valoarea obt, inuta se folosesccomenzile:

1Lect. univ. dr., Universitatea din Pites,ti, [email protected]

65

66 R.M. Georgescu

Round := proc (x, n) -> parse(sprintf("%.*f", n, x));u := FindAngle(d||i||j, d||k||l);a := 180*convert(u, float, 6)/(3.1415926535); (3)b := Round(a, 4); c := trunc(b); d := trunc(60*(b-c));e := Round(3600*(b-c-(1/60)*d), 0)

unde c reprezinta gradele, d minutele, iar e secundele corespunzatoare masurii unghiului u dintredreptele dij s, i dkl.

Pentru determinarea s, i afis,area perechilor de drepte paralele folosim secvent,a

print(Dreptele paralele sunt):for i to n-1 dofor j from i+1 to n dofor k from i to n-1 dofor l from k+1 to n doif (i <> k or j < l) thenif AreParallel(d||i||j, d||k||l) then (4)print(d||i||j si d||k||l)

end ifend ifend doend doend doend do;

Pentru determinarea s, i afis,area perechilor de drepte confundate folosim secvent,a

print(Dreptele confundate sunt):for i to n-1 dofor j from i+1 to n dofor k from i to n-1 dofor l from k+1 to n doif (i <> k or j < l) thenif (AreCollinear(P||i, P||j, P||k) and

AreCollinear(P||i, P||j, P||l)) then (5)print(d||i||j si d||k||l)end ifend ifend doend doend doend do

Pentru determinarea s, i afis,area perechilor de drepte perpendiculare folosim secvent,a

print(Dreptele perpendiculare sunt):for i to n-1 dofor j from i+1 to n dofor k from i to n-1 dofor l from k+1 to n doif (i <> k or j < l) thenif ArePerpendicular(d||i||j, d||k||l) then (6)print(d||i||j si d||k||l)end if

Determinarea pozit, iilor relative ... 67

end ifend doend doend doend do

Pentru a determina s, i afis,a masurile unghiurilor dintre perechile de drepte secante neperpen-diculare folosim secvent,a

print(Masurile unghiurilor dintre dreptele secante neperpendiculare sunt):for i to n-1 dofor j from i+1 to n dofor k from i to n-1 dofor l from k+1 to n doif (i <> k or j < l) thenif ((((AreCollinear(P||i, P||j, P||k) andAreCollinear(P||i, P||j, P||l)) or AreParallel(d||i||j, d||k||l)) orArePerpendicular(d||i||j, d||k||l)) = false thenRound := proc (x, n) -> parse(sprintf("%.*f", n, x));u := FindAngle(d||i||j, d||k||l);a := 180*convert(u, float, 6)/(3.1415926535); (7)b := Round(a, 4); c := trunc(b); d := trunc(60*(b-c));e := Round(3600*(b-c-(1/60)*d), 0);printf("Masura unghiului dintre dreptele %3.3s si %3.3s este%d grade %d minute %d secunde \n", d||i||j, d||k||l, c, d, e)end ifend ifend doend doend doend do

Observat, ie Pentru stabilirea si afis,area pozit, iilor relative a perechilor de drepte determinatede cele n puncte, secvent,ele (4)-(7) pot fi combinate ıntr-una singura, iar programul ın Mapleeste

for i to n-1 dofor j from i+1 to n dofor k from i to n-1 dofor l from k+1 to n doif (i <> k or j <> l) thenif AreCollinear(P||i, P||j, P||k)

and AreCollinear(P||i, P||j, P||l) thenprint(Dreptele d||i||j si d||k||l sunt confundate)

elif AreParallel(d||i||j, d||k||l) thenprint(Dreptele d||i||j si d||k||l sunt paralele) (8)

elif ArePerpendicular(d||i||j, d||k||l) thenprint(Dreptele d||i||j si d||k||l sunt perpendiculare)

else Round := proc (x, n) -> parse(sprintf("%.*f", n, x));u := FindAngle(d||i||j, d||k||l);a := 180*convert(u, float, 6)/(3.1415926535);b := Round(a, 4); c := trunc(b); d := trunc(60*(b-c));e := Round(3600*(b-c-(1/60)*d), 0);print(Dreptele d||i||j si d||k||l sunt secante neperpendiculare)printf("Masura unghiului dintre cele doua drepte este

68 R.M. Georgescu

%d grade %d minute %d secunde", c, d, e)end ifend ifend doend doend doend do

In continuare vom exemplifica not, iunile prezentate pentru n = 5 puncte din plan.

Exemplu. Se dau punctele P1(1, 1), P2(4,−2), P3(5, 5), P4(6, 4), P5(0, 2).

a) Sa se precizeze perechile de drepte paralele;

b) Sa se precizeze perechile de drepte confundate;

c) Sa se precizeze perechile de drepte perpendiculare;

d) Sa se calculeze masurile unghiurilor dintre perechile de drepte secante neperpendiculare.

Cele 5 puncte determina N = 10 drepte, deci trebuie analizate C210 = 45 perechi de drepte.

Mai ıntai definim punctele Pi, i ∈ 1, 5, folosind comanda (1) pentru fiecare punct ın parte, adica

point(P1, 1, 1), point(P2, 4, -2), point(P3, 5, 5),point(P4, 6, 4), point(P5, 0, 2)

Apoi construim dreptele dij, i, j ∈ 1, 5, i 6= j, folosindu-ne de (2), considerand n = 5.

Putem reprezenta ın sistemul de axe ortogonale punctele (Fig. 1 a)) s, i dreptele (Fig. 1 b))obt, inute, cu ajutorul comenzilor

draw(P1,P2,P3,P4,P5)draw(d12,d13,d14,d15,d23,d24,d25,d34,d35,d45)

a) b)Fig. 1

a) Pentru determinarea perechilor de drepte paralele folosim secvent,a (4) pentru n = 5 s, iobt, inem rezultatul

Dreptele paralele suntd12 si d15

Determinarea pozit, iilor relative ... 69

d12 si d25d12 si d34d14 si d35d15 si d25d15 si d34d25 si d34

b) Pentru determinarea perechilor de drepte confundate folosim secvent,a (5) pentru n = 5 s, iobt, inem rezultatul

Dreptele confundate suntd12 si d15d12 si d25d15 si d25

c) Pentru determinarea perechilor de drepte perpendiculare folosim secvent,a (6) pentrun = 5 s, i obt, inem rezultatul

Dreptele perpendiculare suntd12 si d13d13 si d15d13 si d25d13 si d34

d) Pentru a determina masurile unghiurilor dintre perechile de drepte secante neperpendicu-lare folosim secvent,a (7) pentru n = 5 s, i obt, inem valorile pentru masurile unghiurilor dintredrepte.

Daca unim cele patru cerint,e ıntr-o singura problema, folosindu-ne de (8) pentru n = 5,obt, inem pozit, iile relative pentru fiecare din cele 45 de perechi. Folosind notat, iile

= pentru dreptele confundate,

⊥ pentru dreptele perpendiculare,

|| pentru dreptele paralele,iar pentru dreptele neperpendiculare masura unghiului dintre ele, putem sintetiza rezultateleobt, inute ın urmatorul tabel:

d13 d14 d15 d23 d24 d25 d34 d35 d45

d12 ⊥ 7557′50′′ = 537′48′′ 6326′6′′ = || 7557′50′′ 6326′6′′

d13 142′11′′ ⊥ 3652′12′′ 2633′54′′ ⊥ ⊥ 142′11′′ 2633′54′′

d14 7557′50′′ 5054′22′′ 4036′5′′ 7557′50′′ 7557′50′′ || 1231′44′′

d15 537′48′′ 6326′6′′ = || 7557′50′′ 6326′6′′

d23 1018′17′′ 537′48′′ 537′48′′ 5054′22′′ 6326′6′′

d24 6330′′ 6326′6′′ 4036′5′′ 537′48′′

d25 || 7557′50′′ 6326′6′′

d34 7557′50′′ 6326′6′′

d35 1231′44′′

RUBRICA DE ROBOTICA

Programarea robot, ilor LEGO folosind mediul

Mindstorms EV3

Doru Anastasiu Popescu 1

Robot, ii sunt folosit, i pentru a realiza anumite activitat, i. Pentru a crea s, i programa robot, is-au realizat kit-uri speciale s, i medii de programare. Un asemenea robot este cel obt, inut dinpiese LEGO care pentru deplasare foloses,te mediul interactiv de programare Mindstorms EV3.Kit-ul de baza numit ın engleza LEGO Mindstorm Education EV3 Core Set cont, ine pieselenecesare pentru realizarea unui robot, care sa se deplaseze pe o suprafat, a plana (acesta cont, inedoua motoare pentru deplasare, senzori, unitate de comanda – numita caramida inteligenta,rot, i s, i alte componente LEGO necesare construct, iei robotului).

In Figura 1 este prezentat un robot simplu construit cu aceste piese. Pachetul de bazacont, ine descrierea necesara construirii unui robot, cu difert, i senzori, astfel ıncat sa poata fiprogramat pentru deplasarea cu anumite restrict, ii. Un senzor poate fi de mai multe feluri:de culoare – pentru determinarea unei culori, tactil – pentru a detecta aparit, ia unui obiect,infraros,u – pentru detectarea obiectelor din apropierea robotului, etc. Robotul pe care ıl vomfolosi (Figura 1) pentru verificarea programelor va avea doua motoare mari, cate unul pentrufiecare roata laterala s, i o caramida inteligenta (intelligent brick) pentru a controla act, iunile sale.Aceasta caramida are rolul de a memora proiectele s, i de a lansa ın execut, ie programele din ele.

Fig. 1: Elementele constructive ale unui robot LEGO Minstorms EV3

Pentru a crea programe ce deplaseaza robotul avem nevoie de instalarea mediului MindstormsEducation EV3, care nu necesita costuri de licent, a. Kit-ul de instalare se gases,te la adresa

1Conf. univ. dr., Universitatea din Pites,ti, [email protected]

70

Programarea robot, ilor LEGO 71

https://education.lego.com/en-us/downloads. Dupa lansarea ın execut, ie a acestui mediu graficinteractiv de programare se obt, ine o fereastra simpla ın care apare un buton cu semnul plus. Laactivarea acestui buton se va crea un proiect nou, ın care se vor scrie programele necesare pentrudeplasarea robotului. Un proiect se salveaza ıntr-un fis, ier cu extensia ev3, iar un programeste o succesiune de blocuri ce ıncepe cu blocul de Start s, i se termina cu blocul Stop Program.Blocurile sunt grupate pe categorii s, i se afla ın partea de jos a ferestrei zonei de lucru (Figura2). Un bloc este adus ın program prin select, ie s, i glisat la locul lui cu mouse-ul. Fiecare bloctrebuie configurat cu parametrii necesari.

Fig. 2: Interfat,a mediului de programare Minstorms EV3

Primul program!Programul din Figura 3 deplaseaza robotul cu trei rotat, ii complete ale unei rot, i (aproximativ 55cm pentru robotul din imagine). Acesta este alcatuit din patru blocuri: blocul de start, bloculMove Stering – configurat cu On for Rotations, direct, ia ınainte, viteza 50 s, i 3 rotat, ii pentrufiecare roata, blocul Move Stering – configurat cu Off pentru oprirea motoarelor, respectivblocul de oprire Stop Program.

Fig. 3: Deplasarea robotului in linie dreapta cu 3 rotat, ii

Dupa scrierea programului se conecteaza robotul la calculator s, i se ıncarca proiectul cu programulın caramida robotului, apoi se poate lansa ın execut, ie (Figura 4).

72 D.A. Popescu

Fig. 4: Lansarea ın execut, ie a unui program

Din acest exemplu se observa ca programul este ca o schema logica, cu specificarea faptului cablocurile sunt specializate pe o anumita operat, ie s, i conectate ıntre ele.Pentru programele pe care le vom scrie avem nevoie de doua blocuri motor din Grupul Action.

Fig. 5: Blocurile Move Steering s, i Move Tank

Blocurile Move Steering s, i Move Tank sunt folosite pentru cele doua motoare (ın Figura 5motoarele sunt conectate la porturile B s, i C). Primul buton din fiecare bloc permite setareaunitat, ii de masurare a deplasarii: secunde, grade, numar de rotat, ii (On for Rotations - pe careo vom utiliza ın continuare). Pentru oprirea motoarelor se foloses,te Off, iar pentru funct, ionareafara oprire On. La blocul Move Steering celelalte butoane sunt pentru configurarea direct, iei,vitezei s, i numarului de rotat, ii la ambele motoare. La blocul Move Tank apar doua butoanepentru configurarea separata a vitezei la fiecare motor, celelalte butoane au aceeas, i semnificat, ie.Blocul Move Tank este folosit atunci cand vrem sa schimbam direct, ia de deplasare (de exempluo ıntoarcere).

Fig. 6: Blocul Sound

In grupul de blocuri Action mai apar blocurile Sound pentru sunet (Figura 6) s, i Display pentruafis,area datelor pe ecranul caramizii (Figura 7). Blocul Sound permite alegerea fis, ierului cecont, ine sunetul dorit. Mindstorm Education EV3 cont, ine o biblioteca cu fis, iere sunet, acesteafiind grupate pe tipuri de sunete (mecanice, animale, culori, numere, etc.).

Fig. 7: Blocul Display

Programarea robot, ilor LEGO 73

Blocul Display permite afis,area textelor, figurilor geometrice, imaginilor din fis, iere, cu s,tergereasau nu a ecranului. Afis,area se va realiza pornind din pozit, ia specificata prin coordonatele x s, i y.Sistemul de coordonate are originea ın colt,ul din stanga - sus al ecranului, axa Ox pe orizontalacu sensul pozitiv spre dreapta, iar axa Oy pe verticala cu sensul pozitiv ın jos.

Problema deplasarii cu sunete

Dorim sa deplasam robotul cu doua rotat, ii s, i dupa fiecare rotat, ie completa a rot, ilor acesta saredea latratul unui caine s, i apoi sa pronunt,e cuvantul Mindstorms. Creat, i un proiect s, i salvat, i-lcu numele p2.ev3.

Solut, ie

Pentru realizarea programului se creeaza un proiect nou, apoi ın fereastra programului seaduc din zona blocurilor Action: doua blocuri Move Steering, doua blocuri Sound s, i un blocStop Program. Configurarea s, i ordinea acestor blocuri este cea din Figura 8.

Fig. 8: Program pentru deplasarea robotului cu redarea unor sunete

Problema deplasarii cu afis,are pe ecran

Deplasat, i robotul pe o linie dus-ıntors, redat, i repetat cuvantul Hello, iar pe ecranul caramiziiscriet, i cuvintele Dus-Intors. Distant,a parcursa pe linie va fi de aproximativ 55 cm. Creat, i unproiect s, i salvat, i-l cu numele p3.ev3.

Solut, ie

Blocul Start se pune ın legatura cu un bloc Display, care se configureaza cu Text – Pixels,s,tergere ecran, cordonatele x=10, y=20 (x este pe orizontala – spre dreapta, y este pe verticala –ın jos) s, i textul de afis,at Dus-Intors. Urmatorul bloc care trebuie introdus este Sound configuratcu sunetul MINDSTORMS din fis, ierul aflat ın grupul Communication, pentru redare repetata.Urmeaza apoi trei blocuri Move pentru deplasarea ınainte (Move Steering), rotire (Move Tank)s, i iar ınainte (Move Steering). Inainte de blocul Stop Program se reface ecranul caramiziifolosind blocul Display cu opt, iunea Reset Screen. In Figura 9 este programul descris anterior.

Fig. 9: Program pentru deplasarea robotului cu sunete s, i afis, ari pe ecran

Observat, ii

In mediul grafic interactiv Mindstorms Education EV3 proiectele pot cont, ine unul sau maimulte programe, care pot fi lansate ın execut, ie pe rand. Proiectele ce cont, in programele suntscrise pe calculator s, i apoi copiate pe caramida.

74 D.A. Popescu

Programe care folosesc senzori s, i variabile

In continuare vom prezenta modul de folosire a senzorilor s, i a variabilelor pentru a realizaprograme cat mai diverse.

Fig. 10: Senzori pentru robot, i LEGO Minstorms EV3

Problema detectarii culorii unui obiect

In multe situat, ii este nevoie sa cunoas,tem ce culoare are un obiect. Spre exemplu, dacarobotul se deplaseaza pe o suprafat, a cu diverse culori este util sa s,tim la fiecare moment pe ceculoare se afla, pentru a corecta deplasarea.

Programul din Figura 11 determina afis,area pe ecranul caramizii a valorii asociate culorii pecare se afla senzorul de culoare (Colour Sensor) al robotului.

Fig. 11: Program cu senzor de culoare

Problema detectarii distant,ei pana la un obiect

Senzorul care ne permite sa determinam distant,a de la robot pana la un obiect este celinfraros,u (Infrared Sensor). Pentru masurarea distant,ei foloses,te percent-ul notat cu pct, 100pct = 70 cm. Programul din Figura 12 afis,eaza pe ecranul caramizii distant,a de la senzor la uncub.

Programarea robot, ilor LEGO 75

Fig. 12: Program cu senzor de distant, a

Problema detectarii obiectelor prin atingere

Senzorul tactil (Touch Sensor) ne permite detectarea obiectelor atinse de varful acestuia. Infunct, ie de modul de atingere acesta transmite una din valorile 0, 1, 2. Programul din Figura 13afis,eaza una dintre aceste valori.

Fig. 13: Program cu senzor tactil

Grupul Sensor cont, ine blocuri pentru mai mult, i senzori, trei dintre ele fiind folosite maifrecvent (aces,tia sunt ın kit-ul de baza):

1. Touch Sensor (senzor tactil – folosit la masurarea gradului de atingere a obiectelor: 1, 2,3)

2. Infrared Sensor (senzor infraros,u – folosind la identificarea obiectelor de la o distant, aprecizata, unitatea de masura este pct, 100pct=70cm)

3. Colour Sensor (senzor de culoare – folosit la identificarea culorilor, de cele mai multeori pe suprafat,a pe care se deplaseaza robotul, culorile au asociate numere 0 - culoarenedefinita, 1 - negru, 2 – albastru, 3 – verde, 4 – galben, 5 – ros,u, 6 – alb, 7 - maro).

La tot, i senzorii, ın casut,a din dreapta-sus trebuie introdus numarul portului unde esteconectat la caramida. In cazul ın care caramida este conectata la calculator, atunci numarulportului este completat automat.

76 D.A. Popescu

Problema numarului aleator

Folosind generatorul de numere aleatoare vrem sa obt, inem un numar de doua cifre s, i saafis, am pe ecranul caramizii, la un interval de 10 secunde, numarul urmat de patratul sau. Inprogramul urmator sunt prezentate blocurile utilizate.

Fig. 14: Program cu variabile init, ializate cu valori generate aleator

Programul cont, ine cateva blocuri noi: Random – pentru generarea unui numar naturalcuprins ıntre doua valori date, Variable – pentru a realiza operat, ia de creare, atribuire s, i folosirea valorii unei variabile, Math – pentru operat, ii aritmetice, din grupul Data Operations.

Grupul Data Operations cont, ine blocuri pentru lucrul cu variabile, operatori s, i expresii.

Fig. 15: Crearea unei variabile

Blocurile cele mai utilizate din grupul Data Operations sunt:

1. Variable – cu doua stari: Read, pentru folosirea valorii s, i Write, pentru modificarea valorii(Figura 15).

2. Constant – pentru a crea date constante, ce pot fi folosite ın program numai cu valoareadata la definirea lor.

3. Random – permite generarea unui numar aleator dintr-un interval de valori specificat saua valorilor true sau false (de tip logic).

4. Logic Operations – pentru operat, iile logice (s, i, sau, negat, ie). Valorile pentru care serealizeaza operat, ia se vor introduce ın bloc.

5. Math – pentru operat, ii aritmetice (adunare, scadere, ınmult, ire, ımpart, ire, etc.).6. Compare – pentru operat, iile de comparare (folosind operatorii relat, ionali). Rezultatul

acestui bloc este o valoare logica.

Atat pentru variabile, cat s, i pentru constante, se utilizeaza trei tipuri de date: numerice,text s, i logic.

Programarea robot, ilor LEGO 77

Problema deplasarii pe un traseu ın forma de dreptunghi

Generat, i aleator numarul de rotat, ii pe care sa le parcurga un robot pe laturile unui drept-unghi (adica lungimile laturilor masurate ın rotat, ii de motor), pentru lungime din mult, imea4,5,6, iar pentru lat, ime din mult, imea 2,3,4, apoi deplasat, i robotul pornind dintr-un colt, s, ila final revenit, i ın aceeas, i pozit, ie. Toate operat, iile specificate se vor realiza cu acelas, i program.

Solut, ie

Pentru rezolvarea problemei vom folosi doua variabile numerice, cu numele a – pentrulungime, b - pentru lat, ime. Crearea lor se va realiza cu doua blocuri Variable, iar pentruinit, ializarea lor se vor utiliza numerele generate de blocul Random folosit de doua ori, cu limitelede valori 4 s, i 6 pentru a, respectiv 2 s, i 4 pentru b. Apoi sunt folosite blocurile Move Stering s, iVariable, cu opt, iunea Read, pentru numarul de rotat, ii, ca sa se deplaseze robotul. In Figura 16este prezentat programul.

Fig. 16: Program pentru deplasarea robotului pe un contur dreptunghiular

Probleme propuse

1. Se considera programul urmator:

Fig. 17: Program pentru deplasarea unui robot LEGO Minstorms EV3

Cum se deplaseaza robotul, daca se foloses,te acest program? Explicat, i care este rolul fiecaruibloc.

2. Se considera programul din Figura 18. Care este efectul acestui program, daca senzorulde culoare se afla deasupra unei plans,e de culoare alba?

78 D.A. Popescu

Fig. 18: Program cu senzor de culoare

Pentru fiecare din problemele urmatoare scriet,i cate un program folosind mediul interactivde programare Mindstorm Education EV3.

3. Generat, i aleator trei numere naturale din mult, imea 1, 2, ..., 10 s, i apoi afis,at, i pe ecranulcaramizii, pentru 5 secunde, suma lor.

4. Modificat, i programul de la Problema deplasarii pe un traseu ın forma de drept-unghi astfel ıncat deplasarea sa se realizeze pe conturul unui patrat.

Bibliografie

[1] D.A. Popescu, S. Profeanu, S. Dobrescu, Manual de informatica pentru clasa a V-a, EdituraCD-Press, 2017.

[2] Minstorms EV3 – Ghid de Utilizare, LEGO Group, 2013.

[3] L. Negrescu, L. Negrescu, Construirea s,i programarea robot,ilor LEGO Minstorms EV3,Editura Albastra, 2015.

[4] J. Olayvar, E. Lindberg, LEGO Mindstorms EV3 Programming Basics, Washington StateLibrary, 2016.

PROBLEME DE MATEMATICA PENTRU EXAMENE 79

PROBLEME DE MATEMATICA PENTRU

EXAMENE

Teste pentru examenul de Evaluare Nat, ionala

Testul 1

Ana Maria Iordache 1 s, i Madalina Enache 2

SUBIECTUL I

1. Rezultatul calculului 2−1 · 0, (3)− 1 : 6 este . . .2. Daca 2x

7= 10

4y, atunci 10% din xy este . . .

3. Un unghi al unui triunghi are masura egala cu 3312”. Suma celorlalte unghiuri aletriunghiului este egala cu . . .

4. Cardinalul mult, imii Z ∩ [−2, 3) este . . .5. Cubul PROBLEMA are muchia de 6 cm. Aria patrulaterului POML este . . . cm2.6. Un elev are evident,a mesajelor primite pe telefon de la prietenii sai ıntr-o saptamana,

redata ın tabelul de mai jos:

Ziua Luni Mart, i Miercuri Joi Vineri Sambata Duminica

Numarul mesajelor 17 12 18 6 14 20 18

primite

Numarul mediu de apeluri primite pe zi este . . .

SUBIECTUL al II-lea

1. Desenat, i o piramida triunghiulara regulata SABC.2. a) Demonstrat, i ca numarul A = (x2 + x)(x2 + x− 20) + 100 este patrat perfect, oricare

ar fi x ∈ N.b) Determinat, i o valoare naturala a lui x pentru care A este cub perfect.

3. Calculat, i opusul numarului∣∣∣2√3− 1

∣∣∣− 4

|1−√3| .4. Ana are o suma de bani. Dupa ce cheltuies,te o zecime din suma, restul ıl ımparte ın mod

egal cu fratele ei, Dan. In vacant, a, Ana ıs, i tripleaza suma, iar Dan s, i-o ınjumatat,es,te pea sa, ajungand sa aiba ımpreuna 315 lei. Cat, i bani a avut Ana init, ial?

5. Daca x > 0 s, i√x− 1√

x= 2, calculat, i x+ 1

x.

SUBIECTUL al III-lea

1. Figura urmatoare reprezinta curtea unei pensiuni formata din parcul ABCD s, i rondurile cuflori ın forma de semicerc. Stim ca AB = 20 m, DC = 10 m s, i AD = BC = 16 m, AB‖CD.

1 Profesor, Liceul tehnologic ,,Virgil Madgearu”, Ros, iorii de Vede, [email protected] Profesor, Liceul tehnologic ,,Virgil Madgearu”, Ros, iorii de Vede, [email protected]

80 PROBLEME DE MATEMATICA PENTRU EXAMENE

A B

CD

O

Figura 1

a) Aflat, i suprafat,a rondurilor cu flori.b) Determinat, i perimetrul curt, ii.c) Aflat, i cu cat la suta este mai mare aria triunghiului AOB fat, a de aria triunghiului

DOC, unde O = AC ∩BD.2. O cutie sub forma unui paralelipiped dreptunghic ABCDA′B′C ′D′ are dimensiunile

proport, ionale cu primele trei numere naturale prime s, i suma tuturor muchiilor 120 cm.

a) Aflat, i cat, i cm2 de carton au fost necesari pentru confect, ionarea cutiei.b) Determinat, i o funct, ie trigonometrica a unghiului format de dreptele D′B s, i AD.c) Este posibila introducerea ın cutie a unei rigle avand lungimea de 20 cm?

Testul 2

Florin Stanescu 3

SUBIECTUL I

1. Rezultatul calculului 5− 22 · (42 − 15) este . . .2. Daca 4n = 64, atunci n este egal cu . . .3. La aruncarea unui zar probabilitatea de a obt, ine un numar prim este egala cu . . .4. Diagonala unui patrat cu aria de 16 cm2 este egala cu . . . cm5. Se considera cubul ABCDMNPQ. Masura unghiului format de dreptele AB s, i DQ este

egala cu . . ..6. In tabelul de mai jos este reprezentata situat, ia notelor obt, inute de elevii unei clase la un

test de evaluare:

Nota 3 4 5 6 7 8 9 10

Numar elevi 2 5 7 4 5 4 2 1

Media clasei este egala cu . . .

SUBIECTUL al II-lea

1. Desenat, i pe foaia de examen piramida patrulatera regulata V ABCD, cu varful V s, i bazaABCD.

2. Fie a, b, c trei numere astfel ıncat 2a+ b+ 2c = 11 s, i a+ 2b+ c = 4. Calculat, i (a+ c)2− b2.3. Un casier, fiind ıntrebat cat a ıncasat ıntr-o zi, a raspuns: daca as, fi ıncasat ınca 25%

din cat am ıncasat s, i ınca 500 lei, as, fi obt, inut 5500 lei. Cat a ıncasat ın ziua respectivacasierul?

3 Profesor, S, coala Gimnaziala ,,S, erban Cioculescu”, Gaes,ti, [email protected]

PROBLEME DE MATEMATICA PENTRU EXAMENE 81

4. Se considera funct, ia f : R→ R, f(x) = ax+ 2, unde a ∈ R.

a) Determinat, i a, astfel ıncat punctul A(1, 3) apart, ine graficului funct, iei f .b) Pentru a = 1, determinat, i sinusul unghiului format de axa Ox cu graficul funct, iei f .

5. Se considera expresia

E(x) =

Ç3

2(x− 1)− 1

2(x+ 1)− 4

x2 − 1

å:

x− 2

x2 − 2x+ 1, x ∈ R \ −1, 1, 2.

Determinat, i a ∈ Z \ −1, 1, 2 pentru care E(a) ∈ Z.

SUBIECTUL al III-lea

1. In Figura 1 este ilustrat schematic un teren de fotbal. Cele doua careuri sunt reprezen-tate prin doua dreptunghiuri EFIJ , respectiv GHKL, care au lungimea dublul lat, imii,suprafat,a din mijlocul terenului este un cerc de centrul O s, i diametrul de 15 m, iar ABNMeste dreptunghi. Avem: PQ = EF = GH = AE = FB = DH = GC = NP = MQ s, iAM = MD = 50 m.

a) Stabilit, i daca suprafat,a terenului de fotbal, fara cele doua careuri s, i cercul de lamijlocul terenului, este mai mare decat 3390 m2. (3, 14 < π < 3, 15)

b) Doi jucatori, plecand ın acelas, i timp, alearga ın linie dreapta cu aceeas, i viteza peurmatoarele trasee: primul jucator: D −N − A, al doilea jucator: F −N −M −H.Care dintre cei doi jucatori termina primul alergarea?

c) Aflat, i sinusul unghiului EOF .

A

B

P

Q

O

C

D

E HK

J

F GLI

M

N

Figura 1

2. In Figura 2, este reprezentata o cutie sub forma unei prisme drepte ABCDA′B′C ′D′ cubaza patrat. Prisma are aria bazei egala cu 100 dm2, iar suma ariilor fet,elor laterale aleprismei este de 200 dm2.

a) Aratat, i ca ınalt, imea cutiei este BB′ = 5 dm.b) Aflat, i distant,a de la punctul A′ la dreapta AC ′.c) Aflat, i sinusul unghiului format de diagonalele prismei AC ′ s, i A′C.

82 PROBLEME DE MATEMATICA PENTRU EXAMENE

B’

A

D’ C’

B

C

A’

D

Figura 2

Testul 3

Adina Florina Militaru 4

SUBIECTUL I

1. Rezultatul calculului 20180 + 02018 + 2018 : 2 este . . .2. Daca a

b= 2

3, atunci 2a

3beste egal cu . . .

3. Lungimea razei cercului circumscris unui patrat este egala cu 4√

2 cm. Aria patratuluieste . . . cm2.

4. Doua unghiuri complementare congruente au masurile egale cu . . .5. Aria laterala a unui con circular drept cu raza de 3 cm s, i ınalt, imea de 4 cm este egala cu. . . π cm2.

6. In tabelul de mai jos sunt reprezentate notele obt, inute de elevii unei clase la teza de lamatematica:

Note sub 5 5− 5,99 6− 6,99 7− 7,99 8− 8,99 9− 9,99 10

Numar elevi 1 3 4 3 5 7 2

Numarul elevilor care au obt, inut cel put, in nota 7 este egal cu . . .

SUBIECTUL al II-lea

1. Desenat, i pe foaia de examen un paralelipiped dreptunghic PROMQV AT .2. Calculat, i media aritmetica a numerelor a = (3 +

√2)2 s, i b = (3−

√2)2.

3. Alina ıs, i propune sa citeasca o carte ın trei zile. In prima zi vrea sa citeasca jumatatedin ea, a doua zi un sfert s, i ınca 20 de pagini, iar a treia zi cu 5 pagini mai put, in decatjumatate din a doua zi. Cate pagini are cartea?

4. Se considera funct, ia f : R→ R, f(x) = 2x+ 3.

a) Reprezentat, i grafic funct, ia ın sistemul de coordonate xOy.b) Determinat, i aria triunghiului format de graficul funct, iei s, i axele de coordonate.

5. Se considera expresia

E(x) =

ñ1

x+

x− 3

(x− 1)(x+ 1)− x− 1

x(x+ 1)

ô· x− 1

x− 2, x ∈ R \ −1, 0, 1, 2.

Aratat, i ca E(x) = 1x.

4 Profesor, Liceul Tehnologic Construct, ii de Mas, ini, Mioveni, [email protected]

PROBLEME DE MATEMATICA PENTRU EXAMENE 83

SUBIECTUL al III-lea

1. Fie ABCD un trapez dreptunghic ın A, avand bazele AB = 14 cm s, i CD = 10 cm, iarm(^B) = 30.

a) Determinat, i lungimea liniei mijlocii a trapezului.b) Determinat, i aria trapezului.c) Calculat, i distant,a de la varful C la diagonala BD.

2. Se considera un paralelipiped dreptunghic ABCDA′B′C ′D′ cu dimensiunile de 6 cm, 4cm, respectiv 3 cm.

a) Calculat, i lungimea diagonalei paralelipipedului.b) Stiind ca pentru acoperirea unui cm2 se folosesc 10 g vopsea, calculat, i cate kg de

vopsea sunt necesare pentru acoperirea ıntregului corp.c) Cat, i litri de apa ıncap ın paralelipiped?

Testul 4

Mugurel Simion 5

SUBIECTUL I

1. Rezultatul calculului 18 · 6− 18 : 6 este egal cu . . . .2. Numerele ıntregi din intervalul (−2, 6] sunt ın numar de . . . .3. Sase robinete de acelas, i debit pot umple un bazin ın 8 ore. Doua robinete vor umple

acelas, i bazin ın . . . ore.4. Aria unui patrat este 81 cm2. Perimetrul patratului este egal cu . . . cm.5. Lungimea ınalt, imii unui con circular drept cu raza bazei egala cu 8 cm s, i generatoarea de

16 cm este egala cu . . . .6. Numarul elevilor dintr-un lot de handbal s, i varstele lor sunt reprezentate ın tabelul de

mai jos:

Varsta (ani) 11 12 13

Numarul elevilor 5 10 5

Media de varsta a lotului de elevi este egala cu . . . ani.

SUBIECTUL al II-lea

1. Desenat, i un cilindru circular drept cu sect, iunea axiala dreptunghiul ABB′A′.2. Aflat, i cel mai mic numar natural de trei cifre care prin ımpart, ire la 14 s, i 21 da de fiecare

data restul 3.

3. Un turist a parcurs un traseu ın trei zile. In prima zi a parcurs2

7din lungimea traseului.

A doua zi a parcurs 20% din rest, iar a treia zi a parcurs ultimii 24 de km. Aflat, i lungimeatraseului.

4. Se considera funct, ia f : R→ R, f(x) = −2x+ 6.

a) Aratat, i ca: f(√

3)− f(√

3− 5) = −10.

b) Determinat, i punctele M(x, y) situate pe graficul funct, iei f cu proprietatea∣∣∣x∣∣∣ =

∣∣∣y∣∣∣.5 Profesor, S, coala Gimnaziala ,,Constantin Brancoveanu”, S, tefanes,ti, simion [email protected]

84 PROBLEME DE MATEMATICA PENTRU EXAMENE

5. Se considera expresia:E(x) = (x+ 1)(x2 + 2x− 24).

Aratat, i ca 6|E(x), pentru oricare x numar ıntreg.

SUBIECTUL al III-lea

1. Figura urmatoare reprezinta schit,a unui teren. Triunghiul ABC este isoscel cu AB =AC = 20 cm s, i m(^BAC) = 120. Triunghiul BDC este dreptunghic cu m(^BDC) = 90

s, i DC = 10√

3 cm.

A

BQ

C

D

a) Aratat, i ca BC = 20√

3 cm.b) Aratat, i ca A4ACD = 100

√3 cm2.

c) Daca Q ∈ (BC) astfel ıncat BQ = 12√

3 cm, verificat, i daca punctele A, Q s, i D suntcoliniare.

2. Fie ABCDA′B′C ′D′ o prisma patrulatera regulata cu AB = 6 cm, AA′ = 8 cm, BD ∩AC = P s, i BC ′ ∩B′C = Q.

a) Aratat, i ca volumul prismei este egal cu 288 cm3.

b) Aratat, i ca sinusul unghiului format de dreptele PQ s, i B′D′ este egal cu

√82

10.

c) Aflat, i distant,a de la punctul A′ la planul (AB′D′).

Testul 5

Adrian Turcanu 6

SUBIECTUL I

1. Inversul numarului rat, ional 0,25 este numarul natural . . .2. Aria unui romb cu diagonalele de 12 cm s, i 7 cm este . . . cm2.3. Solut, ia negativa a ecuat, ie (x+ 5)2 = 121 este . . .4. Cel mai mare numar natural din intervalul [−3, 8) este . . .5. Fie ABCDA′B′C ′D′ un cub. Masura unghiului dintre dreptele B′C s, i AD′ este . . .6. Notele obt, inute de elevii clasei a VIII-a A la teza la matematica din semestrul I sunt date

ın tabelul de mai jos:

Nota 4 5 6 7 8 9 10

Numar elevi 1 2 3 3 5 5 11

6 Lect. univ. dr., Universitatea din Pites,ti, [email protected]

PROBLEME DE MATEMATICA PENTRU EXAMENE 85

Conform tabelului, probabilitatea ca alegand un elev din Clasa a VIII-a A, acesta sa fiobt, inut la teza o nota mai mare sau egala cu 8 este . . .

SUBIECTUL al II-lea

1. Desenat, i prisma patrulatera regulata MATEINFO.2. Aratat, i ca numarul N = (3−

√2)2 + 2(

√2 + 5)(3−

√2) + (

√2 + 5)2 este natural.

3. Un turist a parcurs un traseu ın trei zile astfel: ın prima zi o cincime din drum, a doua zijumatate din rest, iar a treia zi ultimii 10 km. Aflat, i lungimea traseului.

4. Fie funct, ia f : R→ R, f(x) = 2x− 6.

a) Reprezentat, i grafic funct, ia ın sistemul de coordonate xOy.b) Determinat, i numarul natural nenul n care verifica egalitatea f(1)+f(2)+. . . +f(n) =

150.5. Se considera mult, imile A = x ∈ Z | |2x− 1| ≤ 5 s, i B = x ∈ Z | 10

x+1∈ Z. Determinat, i

A ∩B.

SUBIECTUL al III-lea

1. O gradina are forma unui trapez isoscel ABCD cu AB = 100 m, CD = 40 m s, iBC = AD = 50 m.

a) Calculat, i lungimea gardului care ımprejmuies,te gradina.b) Calculat, i aria suprafet,ei gradinii.c) Daca AD ∩BC = M, determinat, i aria triunghiului MDC.

2. Fie V ABC o piramida triunghiulara regulata cu latura bazei BC = 12 cm, M mijlocullui [BC], VM = 6 cm s, i V O ınalt, imea acesteia.

a) Aflat, i aria laterala s, i volumul piramidei.b) Daca N este mijlocul laturii [CV ] aratat, i ca MN ‖ (V OB).c) Aratat, i ca V A ⊥ VM .

86 PROBLEME DE MATEMATICA PENTRU EXAMENE

Teste pentru examenul de Bacalaureat, specializarea S, tiint, e ale naturii

Testul 1

Marius Macarie 1

SUBIECTUL I

1. Determinat, i partea imaginara a numarului complex z =5− 4i

2 + 3i, unde i2 = −1.

2. Sa se determine m ∈ R astfel ıncat valoarea minima a funct, iei f : R → R, f(x) =

x2 − 3x+m− 2 sa fie −1

4.

3. Rezolvat, i ın R ecuat, ia: log3 x+ logx 3 =5

2.

4. Sa se calculeze probabilitatea ca alegand o mult, ime din mult, imea submult, imilor nevideale mult, imii A = 1, 2, 3, 4, 5, aceasta sa aiba trei elemente.

5. Sa se determine ecuat, ia mediatoarei segmentului AB, unde A(2,−1), B(−4, 1).6. Triunghiul ABC are lungimile laturilor AB = 2, BC = 3 s, i AC = 4. Sa se calculeze cosA.

SUBIECTUL al II-lea

1. Se considera matricea A(X) =

Ñ1 + 3x 9x

x 1 + 3x

é, unde x este numar real.

a) Calculat, i detA(−1).b) Demonstrat, i ca A(x) · A(y) = A(x+ y + 6xy), pentru orice x, y ∈ R.c) Calculat, i A−1(1).

2. Pe G = (1,∞) se defines,te legea de compozit, ie x y =√x2y2 − x2 − y2 + 2.

a) Demonstrat, i ca x y =»

(x2 − 1)(y2 − 1) + 1.b) Demonstrat, i ca legea ,,” este asociativa.c) Rezolvat, i ın G ecuat, ia x x . . . x︸ ︷︷ ︸

de 2018 ori x

= x.

SUBIECTUL al III-lea

1. Fie f : R→ R, f(x) = 2x3 · e−x.a) Determinat, i ecuat, ia asimptotei la Gf spre +∞.b) Determinat, i punctele de extrem ale funct, iei f .

c) Calculat, i limx→∞

x

Çf ′′(x)

f ′(x)+ 1

å.

2. Fie f : (0,∞)→ R, f(x) =x− 1√x

.

a) Sa se determine a, b ∈ R astfel ıncat funct, ia g : (0,∞)→ R, g(x) = (ax+ b) ·√x sa

fie o primitiva a lui f .

b) Calculat, i∫ 4

1f(x)g2(x)dx, unde a s, i b au valorile determinate la punctul a).

c) Determinat, i volumul corpului obt, inut prin rotat, ia ın jurul axei Ox a graficului funct, iei

h : [2, 3]→ R, h(x) =1

f(x).

1 Lect. univ. dr., Universitatea din Pites,ti, [email protected]

PROBLEME DE MATEMATICA PENTRU EXAMENE 87

Testul 2

Raluca Mihaela Georgescu 2

SUBIECTUL I

1. Sa se determine x ∈ R astfel ıncat 2, x2 + 3x s, i 8 sunt termeni consecutivi ai unei progresiigeometrice.

2. Sa se determine inversa funct, iei bijective f : R→ (4,∞), f(x) = 2x + 4.3. Rezolvat, i ın mult, imea numerelor reale ecuat, ia

√5x− 6− x = 0.

4. Cate numere naturale de 4 cifre distincte se pot forma cu elemente din A = 0, 1, 2, 3, 4, 5, 6.5. Fie punctele A(2, 3), B(−1, 1) s, i C(0,−2). Aflat, i lungimea vectorului

−→AB +

−→AC +

−−→BC.

6. Fie x ∈Å

0,π

2

ãs, i sin x =

5

13. Sa se calculeze tg 2x.

SUBIECTUL al II-lea

1. Se considera matricea A(a) =

Ü1 a 2

1 1 2a

1 2 a

ê, unde a este numar real.

a) Calculat, i detA(0).b) Sa se afle a ∈ R astfel ıncat A(a) sa fie inversabila.

c) Sa se determine X ∈M3,1(R) astfel ıncat A(3) ·X =

Ü−2

0

−1

ê.

2. Pe mult, imea G = (−2, 2) se defines,te legea de compozit, ie x y =4x+ 4y

4 + xy. Fie f : G→ R,

f(x) = log5

2 + x

2− x.

a) Demonstrat, i ca legea ,,” este asociativa.b) Demonstrat, i ca f(x y) = f(x) + f(y), ∀x, y ∈ G.c) Sa se determine simetricul lui 1 ∈ G fat, a de legea ,,”.

SUBIECTUL al III-lea

1. Fie f : (0,∞)→ R, f(x) =1 + ln x

x.

a) Determinat, i ecuat, ia tangentei la graficul funct, iei f ın punctul de abscisa x = e, situatpe graficul funct, iei f .

b) Demonstrat, i ca lnx ≤ x− 1, oricare ar fi x ∈ (0,∞).c) Determinat, i intervalele de convexitate s, i concavitate ale funct, iei f .

2. Fie f : R→ R, f(x) = (x+ 2) · e−x.

a) Aratat, i ca∫ 1

−1ex · f(x)dx = 4.

b) Calculat, i∫ 1

0f ′′(x)dx.

c) Sa se determine numarul real a > 1 astfel ıncat aria suprafet,ei plane delimitate degraficul funct, iei g : (0,∞)→ R, g(x) = f(lnx), axa Ox s, i dreptele de ecuat, ii x = 1

s, i x = a sa fie egala cu5

2.

2 Lect. univ. dr., Universitatea din Pites,ti, [email protected]

88 PROBLEME DE MATEMATICA PENTRU EXAMENE

Testul 3

Mihai Florea Dumitrescu 3

SUBIECTUL I

1. Aflat, i suma elementelor mult, imii A = x ∈ [−1; 1] | [x] · x = 1, unde [x] reprezinta parteaıntreaga a numarului real x.

2. Se considera funct, iile f : R→ R, f(x) = x2 − 2x+ 1 s, i g : R→ R, g(x) = −x2 + 4x− 4.Aflat, i distant,a dintre varfurile parabolelor asociate celor doua funct, ii.

3. Rezolvat, i ın mult, imea numerelor reale inecuat, ia 9log3 x ≤ 2 · 3log3 x − 1 .4. Cate numere pare de trei cifre se pot forma cu cifrele 0, 1, 2, 3 ?5. In reperul cartezian xOy se considera punctele A(3,−5) s, i B(−3, 1). Determinat, i lungimea

vectorului−→AO +

−−→OB.

6. Se considera expresia E (x) = sin2 2x+ cos2 3x+ tg 24x+ ctg 26x, x ∈ R. Calculat, i EÄπ12

ä.

SUBIECTUL al II-lea

1. Se considera matricele A (x, y) =

Üx

1

y

ê, B (x, y) =

(1 y x

), unde x s, i y sunt

numere reale.

a) Aratat, i ca det [A (x, y) ·B (x, y)] = 0 pentru orice numere reale x s, i y.b) Rezolvat, i ın mult, imea numerelor reale ecuat, ia det [A (x, x) ·B (x, x) + xI3] = 4x.c) Rezolvat, i ın Z× Z ecuat, ia det [B (x, y) · A (x, y)] = 1.

2. Pe mult, imea numerelor reale se defines,te legea de compozit, ie asociativa x∗y = xy−x−y+2.

a) Aratat, i ca x ∗ y = (x− 1) (y − 1) + 1 pentru orice numere reale x s, i y.b) Aflat, i numarul n ∈ N care verifica egalitatea C1

n ∗ C2n = 1.

c) Calculat, i 2 ∗ 3 ∗ 4 ∗ ... ∗ 2018.

SUBIECTUL al III-lea

1. Se considera funct, ia f : R→ R, f (x) = x+ cosx.

a) Calculat, i limx→+∞

f (x).

b) Determinat, i ecuat, ia tangentei la graficul funct, iei f ın punctul de abscisa x = 0, situatpe graficul funct, iei f .

c) Aratat, i ca 2 cos 1 ≤ π − 2.2. Se considera funct, ia f : R→ R, f (x) =

√x2 + 1.

a) Aratat, i ca∫ 1

0

√x2 + 1 · f (x) dx =

4

3.

b) Calculat, i∫ 2

1f(x) dx.

c) Aflat, i volumul corpului obt, inut prin rotat, ia ın jurul axei Ox a graficului funct, ieig : [1, 2]→ R, g (x) = x

f(x).

3 Profesor, Liceul ,,S, tefan Diaconescu”, Potcoava, [email protected]

PROBLEME DE MATEMATICA PENTRU EXAMENE 89

Testul 4

Gheorghe Nistor 4

SUBIECTUL I

1. Fie (an)n≥1 o progresie aritmetica. S, tiind ca a3 = 10 s, i a5 = 16, sa se calculeze a100.2. Rezolvat, i ın R ecuat, ia: (f f)(x) = 7, unde f : R→ R, f(x) = −4x+ 3.3. Rezolvat, i ın R inecuat, ia: 2 log2 x+ 1 ≤ 5.4. Aflat, i probabilitatea ca alegand o funct, ie f : 1, 2, 3 → 7, 8, 9, aceasta sa fie bijectiva.5. Sa se determine ecuat, ia dreptei care trece prin A(0, 3) s, i e perpendiculara pe dreapta

determinata de punctele B(0, 2) s, i C(−1, 4).6. Sa se afle raza cercului circumscris triunghiului ABC cu lungimile laturilor AB = 7,BC = 8 s, i AC = 9.

SUBIECTUL al II-lea

1. Se considera sistemul de ecuat, ii

mx+ y + 2z = 4

2x+my + z = 3

x+ 2y +mz = 1

s, i matriceaA(m) =

Üm 1 2

2 m 1

1 2 m

ê,

unde m este un numar real.

a) Calculat, i detA(0).b) Sa se afle m ∈ R pentru care matricea A(m) nu este inversabila.c) Rezolvat, i sistemul pentru m = 1.

2. Se considera polinomul f = X4 −X3 +mX2 + 2X + n unde m,n ∈ R.

a) Sa se determine m,n ∈ R astfel ıncat f sa fie divizibil cu g = X2 − 3X + 2.b) Sa se determine m,n ∈ R astfel ıncat x1 = 1 + i sa fie radacina pentru f .c) Pentru m = n = 0 sa se descompuna polinomul f ın produs de factori ireductibili ın

R[X].

SUBIECTUL al III-lea

1. Fie f : R \ 1 → R, f(x) =x2 + x+ 1

x− 1.

a) Calculat, i limx→3

f(x)− f(3)

x− 3.

b) Determinat, i ecuat, ia asimptotei oblice spre +∞ la graficul funct, iei f .c) Determinat, i intervalele de monotonie ale funct, iei f .

2. Fie f : (0,∞)→ R, f(x) = (x+ 1) lnx.

a) Sa se determine o primitiva F a lui f pentru care F (1) =3

4.

b) Calculat, i∫ 3

2

lnx

xf(x)dx.

c) Determinat, i volumul corpului obt, inut prin rotat, ia ın jurul axei Ox a graficului funct, iei

g : [1, e]→ R, g(x) =f(x)

x+ 1.

4 Lect. univ. dr., Universitatea din Pites,ti, [email protected]

90 PROBLEME DE MATEMATICA PENTRU EXAMENE

Testul 5

Alina S, tefan 5

SUBIECTUL I

1. Aratat, i ca numarul n = log3

Ä9 3√

+ 8log2 3 − 8

3este numar natural.

2. Aflat, i distant,a dintre punctele de intersect, ie ale graficului funct, iei f : R → R, f(x) =x2 + 4x+ 3 cu axa absciselor.

3. Rezolvat, i ın R inecuat, ia:

Ç4

9

åx+1

·Ç

27

8

åx=

2

3.

4. Calculat, i probabilitatea ca, alegand un numar din mult, imea numerelor naturale de 3 cifre,acesta sa fie cub perfect.

5. Fie vectorii −→u = 2−→i − 3

−→j s, i

−→v = m−→i − (m− 1)

−→j . Sa se determine m ∈ R astfel ıncat

−→u · −→v = 5.6. Calculat, i lungimea razei cercului circumscris triunghiului MNP , s,tiind ca M =

π

6, N =

π

3s, i MN = 4.

SUBIECTUL al II-lea

1. Se considera matricele A =

(3 6

−1 −2

), I2 =

(1 0

0 1

)s, i X(a) = I2 + a ·A, unde a ∈ R.

a) Calculat, i det(A2).b) Verificat, i daca X(a) ·X(b) = X(a+ b+ ab) pentru orice a, b ∈ R.c) Determinat, i X2018(1).

2. Se considera polinomul f = X3 − 3X2 + aX − 2, a ∈ R.

a) Sa se determine a ∈ R astfel ıncat restul ımpart, irii lui f la X − 2 sa fie 4.

b) Sa se demonstreze ca daca a ∈Ç

9

2,∞

å, atunci f nu are toate radacinile reale.

c) Sa se determine a ∈ R astfel ıncat

∣∣∣∣∣∣∣∣x1 x2 x3x3 x1 x2x2 x3 x1

∣∣∣∣∣∣∣∣ = 9, unde x1, x2, x3 sunt radacinile

polinomului f .

SUBIECTUL al III-lea

1. Fie f : (0,∞)→ R, f(x) = x3 · lnx.

a) Calculat, i limx→e

f(x)− f(e)

x− e.

b) Aratat, i ca graficul funct, iei f nu admite asimptote.

c) Demonstrat, i ca f(x) ≥ − 1

3e, pentru orice x ∈ (0,∞).

2. Pentru fiecare n ∈ N? se considera funct, ia fn : R→ R, fn(x) =xn

x2 + 4.

a) Aratat, i ca∫ 2

1f2Ä√xädx = 1− 4 ln

6

5.

b) Demonstrat, i ca∫ 1

0fn+2(x)dx+ 4

∫ 1

0fn(x)dx =

1

n+ 1.

c) Sa se determine a > 1 astfel ıncat aria suprafet,ei plane delimitate de graficul funct, ieif1, axa Ox s, i dreptele de ecuat, ii x = 1 s, i x = a sa fie egala cu ln 2.

5 Lect. univ. dr., Universitatea din Pites,ti, [email protected]

PROBLEME DE MATEMATICA PENTRU EXAMENE 91

Teste pentru examenul de Bacalaureat, specializareaMatematica-Informatica

Testul 1

Daniel Jinga 1

SUBIECTUL I (30p)

1. Determinat, i numerele reale a, b s,tiind ca (a+ bi)(b+ ai) = (a− bi)(b− ai), unde i2 = −1.(5p)

2. Determinat, i m ∈ R pentru care funct, ia f : R → R, f(x) = x2 − (m− 3)x + m− 4 arevaloarea minima egala cu 0. (5p)

3. Rezolvat, i ecuat, ia 6x − 2x − 2 · 3x + 2 = 0. (5p)

4. Aflat, i n ∈ Z cu proprietatea Cn2

n+2 = 1. (5p)

5. In triunghiul ABC, AB = 3, BC = 6, AC = 4. Calculat, i−→AB ·

−→AC. (5p)

6. Fie x ∈ R astfel ıncat tg x =1

2. Calculat, i sin 2x. (5p)

SUBIECTUL al II-lea (30p)

1. Se considera sistemul de ecuat, ii liniare

x+my − z = 1

x+ y − z = 2

−x+ y + z = 0

, unde m ∈ R.

a) Aratat, i ca matricea sistemului are rangul 2 pentru orice m ∈ R. (5p)

b) Determinat, i m ∈ R astfel ıncat sistemul sa fie compatibil. (5p)

c) Aflat, i solut, iile (x, y, z) ∈ Z× Z× Z cu proprietatea ca x2 + y2 + z2 = 42. (5p)

2. Fie G = (−1, 1) s, i x ∗ y =x+ y

1− xy, ∀x, y ∈ G.

a) Aratat, i ca tg 27 ∗ tg 18 = 1. (5p)

b) Aratat, i ca (x ∗ y) ∗ z = x ∗ (y ∗ z), ∀x, y, z ∈ G. (5p)

c) Rezolvat, i ın G ecuat, ia x ∗ x ∗ x = 2x ∗ 2x ∗ 2x. (5p)

SUBIECTUL al III-lea (30p)

1. Se considera funct, ia f : R \ −1, 0 → R, f(x) =3x2 + 3x+ 1

x3(x+ 1)3.

a) Aratat, i ca f(x) =1

x3− 1

(x+ 1)3, ∀x ∈ R \ −1, 0. (5p)

b) Determinat, i coordonatele punctului situat pe graficul functiei f , ın care tangenta lagraficul funct, iei f este perpendiculara pe axa ordonatelor. (5p)

c) Calculat, i limn→∞

(f(1) + f(2) + . . .+ f(n))1

nf(n) . (5p)

1 Profesor, Colegiul Nat, ional ,,Ion C. Bratianu”, Pites,ti, [email protected]

92 PROBLEME DE MATEMATICA PENTRU EXAMENE

2. Se considera funct, ia f : R→ R, f(x) = 2x− x2 s, i In =∫ 2

0fn(x)dx, unde n ∈ N∗.

a) Aratat, i ca I1 = 43. (5p)

b) Aratat, i ca s, irul (In)n≥1 este convergent. (5p)

c) Aratat, i ca (2n+ 1)In = 2nIn−1, ∀n ≥ 2. (5p)

Testul 2

Sorin Ulmeanu 2

SUBIECTUL I (30p)

1. Sa se rezolve ın mult, imea numerelor reale ecuat, ia |x− 1| = 2x− 5. (5p)

2. Fie ecuat, ia x2−2x+m = 0, m ∈ R, care are solutiile reale x1 s, i x2. Stiind ca |x1−x2| = 1,sa se determine m. (5p)

3. Sa se rezolve ecuat, ia sinÅx+

π

6

ã+ cos

Åπ3− x

ã= 1, x ∈ R. (5p)

4. Sa se arate ca funct, ia f : (−1, 1)→ R, f(x) = ln1− x1 + x

, ∀x ∈ (−1, 1) este impara. (5p)

5. Fie G centrul de greutate al triunghiului ABC. Sa se determine a, b ∈ R astfel ıncat sa

aiba loc egalitatea a−→GA+ b

−−→GB =

−→GC. (5p)

6. Sa se determine ecuat, ia medianei din A a triunghiului ABC, s,tiind ca A(2, 2) s, i caecuat, iile medianelor duse din B s, i C sunt 2x+ y − 2 = 0, respectiv x− y + 2 = 0. (5p)

SUBIECTUL al II-lea (30p)

1. Se considera sistemul (S):

ax+ y + z = 1

x+ ay + z = 0

x+ y + az = −1

, a ∈ R.

a) Aratat, i ca determinantul matricei sistemului este egal cu (a+ 2)(a− 1)2. (5p)

b) Determinat, i a ∈ R astfel ıncat (S) sa nu fie compatibil. (5p)

c) Aflat, i a ∈ R astfel ıncat solut, ia unica (x0, y0, z0) a sistemului (S) sa formeze o progresie

aritmetica de rat, ie r = −2

5. (5p)

2. Se considera mult, imea de numere complexe G = cos(qπ) + i sin(qπ) | q ∈ Q.

a) Sa se arate ca z = 0 nu se afla ın G s, i ca ε =1

2+ i

√3

2se afla ın G. (5p)

b) Sa se arate ca ınmult, irea numerelor complexe determina pe G o structura algebrica degrup comutativ. (5p)

c) Sa se arate ca polinomul f = X6 − 1 are toate radacinile ın G. (5p)

2 Profesor, Colegiul Nat, ional ,,Ion C. Bratianu”, Pites,ti, [email protected]

PROBLEME DE MATEMATICA PENTRU EXAMENE 93

SUBIECTUL al III-lea (30p)

1. Se consdera funct, ia f : R→ R, data prin f(x) = x− sinx, pentru orice x ∈ R.

a) Sa se arate ca funct, ia f este strict crescatoare. (5p)

b) Sa se arate ca graficul funct, iei f nu are asimptote. (5p)

c) Sa se arate ca g : R→ R, g(x) = 3»f(x), oricare ar fi x ∈ R, este derivabila pe R. (5p)

2. Fie s, irul (In)n∈N∗ , dat prin In =∫ 1

−1(1− x2)ndx, ∀n ∈ N∗.

a) Sa se calculeze I2. (5p)

b) Sa se demonstreze ca In+1 =2n+ 2

2n+ 3In, ∀n ∈ N∗. (5p)

c) Sa se demonstreze ca s, irul (an)n∈N∗ , definit prin an =n∑k=0

(−1)kCkn

2k + 1, ∀n ∈ N∗, are limita 0.

(5p)

Testul 3

Ileana Tarca 3

SUBIECTUL I (30p)

1. Calculat, i partea reala a numarului complex (1− 5i)2. (5p)

2. Rezolvat, i ecuat, ia3»

15−√x− 4 = 2. (5p)

3. Se considera mult, imea A = 1, 2, 3, 4, 5, 6. Determinat, i probabilitatea ca alegand laıntamplare una dintre submult, imile lui A, aceasta sa cont, ina exact 3 elemente. (5p)

4. Fie fm, g : R → R, fm(x) = x2 + mx + m s, i g(x) = x + 2. Determinat, i m ∈ R pentrucare Gfm ∩Gg are un singur element. (5p)

5. Fie dreptele d1: mx+ 2y+ 1 = 0, d2 : x+ (m+ 1)y+ 5 = 0. Sa se determine m ∈ R astfelıncat d1 ‖ d2. (5p)

6. Sa se calculeze sin2 1480 + cos2 2200. (5p)

SUBIECTUL al II-lea (30p)

1. Se considera matricele A =

Ü1 0 0

0 1 0

0 0 0

ê, B =

Ü0 1 0

0 0 0

1 0 0

ês, i O3 =

Ü0 0 0

0 0 0

0 0 0

ê.

a) Sa se calculeze A2. (5p)

b) Sa se calculeze rangul matricei A+B. (5p)

c) Sa se calculeze m·n, unde m este cel mai mic numar natural nenul pentru care (A·B)m = O3

s, i n este cel mai mic numar natural nenul pentru care (B · A)n = O3. (5p)

2. Se considera polinomul f = X4 − 3X + 1, cu radacinile x1, x2, x3, x4 ∈ C.

3 Profesor, Liceul de Arte ,,Dinu Lipatti”, Pites,ti, ileana [email protected]

94 PROBLEME DE MATEMATICA PENTRU EXAMENE

a) Sa se calculeze x1 · x2 · x3 · x4. (5p)

b) Sa se calculeze x41 + x42 + x43 + x44. (5p)

c) Sa se arate ca polinomul f nu are radacini rationale. (5p)

SUBIECTUL al III-lea (30p)

1. Se considera funct, ia f : (−2, 2)→ R, f(x) =ln(2− x)

x+ 2.

a) Sa se calculeze f ′(x), x ∈ (−2, 2). (5p)

b) Verificat, i daca funct, ia f este descrescatoare pe intervalul (−2, 2). (5p)

c) Determinat, i punctele de inflexiune ale funct, iei f . (5p)

2. Se considera funct, ia f : R→ R, f(x) =1√

x2 + 9.

a) Calculat, i∫ 1

0f(x)dx. (5p)

b) Calculat, i∫ 1

0xf(x)dx. (5p)

c) Determinat, i volumul corpului obt, inut prin rotat, ia ın jurul axei Ox a graficului funct, ieig : [0, 1]→ R, g(x) = f(x). (5p)

Testul 4

Mihai Florea Dumitrescu 4

SUBIECTUL I (30p)

1. Aflat, i suma elementelor mult, imii A =

®ñ1

x

ô|x ∈ Z∗

´, unde [a] reprezinta partea ıntreaga

a numarului real a. (5p)

2. Se considera funct, ia f : R→ R, f(x) = x2 − ax+ 1. Aflat, i a ∈ R pentru care distant,a dela axa de simetrie a parabolei asociate funct, iei f la axa Oy este egala cu 2. (5p)

3. Rezolvat, i ın mult, imea numerelor reale ecuatia |x− 1|3 = (1− x)2. (5p)

4. Calculat, i probabilitatea ca, alegand la ıntamplare un numar din mult, imea numerelornaturale de doua cifre, cu cifre nenule s, i diferite, acesta sa aiba cifra zecilor patrat perfect s, icifra unitat, ilor cub perfect. (5p)

5. In reperul cartezian xOy se considera punctele A(1,−5) s, i B(−3, 3). Determinat, isimetricul punctului O fat, a de dreapta AB. (5p)

6. Se considera expresia, E(x) = sinÅπ

2− x

ã+ cos

Åx− π

2

ã+ tg

Åx− π

2

ã+

1

tg xunde

x ∈Å

0,π

2

ã. Stiind ca sin 2x =

1

4, aratat, i ca E(x) =

√5

2. (5p)

4 Profesor, Liceul ,,S, tefan Diaconescu”, Potcoava, [email protected]

PROBLEME DE MATEMATICA PENTRU EXAMENE 95

SUBIECTUL al II-lea (30p)

1. Se considera matricea A(x) =

Üx −1 1

−1 x −1

1 −1 x

ê, unde x este numar real.

a) Calculat, i detA(0). (5p)

b) Rezolvat, i ın mult, imea numerelor reale ecuat, ia detA(log2 x) = 0. (5p)

c) Aratat, i ca A(a) ·A(b) = A(ab+ 2) + (a+ b) ·A(0), pentru orice numere reale a s, i b. (5p)

2. Se considera polinomul f = X3 + aX2 + aX + b, unde a, b ∈ R s, i fie x1, x2, x3 radacinilepolinomului.

a) Aflat, i a s, i b s,tiind ca polinomul X2 + 1 divide polinomul f . (5p)

b) Pentru b = 1, exista a ∈ Z pentru care are loc egalitatea (5p)

1

x31+

1

x32+

1

x33= 2

Ç1

x21+

1

x22+

1

x23

å+

1

x1+

1

x2+

1

x3?

c) Aratat, i ca, pentru b = 1 s, i a ∈ (−∞,−1] ∪ [3,∞) polinomul f are toate radacinile reale.(5p)

SUBIECTUL al III-lea (30p)

1. Se considera funct, ia f : R→ R, f(x) = sinx− x.

a) Aratat, i ca funct, ia f este inversabila. (5p)

b) Aratat, i ca 2(2−√

3)(3 sin 1 + π) > 3. (5p)

c) Aratat, i ca fÅx+ y

2

ã≥ f(x) + f(y)

2, pentru orice x, y ∈ [0, π]. (5p)

2. Pentru fiecare numar natural nenul n, se considera integrala In =∫ 1

0

xn

x4 + 1dx.

a) Calculat, i I3. (5p)

b) Calculat, i8∑

k=1Ik. (5p)

c) Calculat, i limn→∞

nIn. (5p)

Testul 5

Antonio Nuica 5

SUBIECTUL I (30p)

1. Sa se determine

∣∣∣∣∣1− 3i

2 + i

∣∣∣∣∣. (5p)

2. Sa se rezolve ın R ecuat, ia 4x − 3 · 2x + 2 = 0. (5p)

5 Lect. univ. dr., Universitatea din Pites,ti, antonio [email protected]

96 PROBLEME DE MATEMATICA PENTRU EXAMENE

3. Sa se determine m ∈ R pentru care mx2 −mx+ 1 > 0, ∀x ∈ R. (5p)

4. Sa se determine numarul funct, iilor injective f : 1, 2, 3, 4, 5 → 1, 2, 3, 4, 5 cu proprieta-tea f(1) = 1. (5p)

5. Sa se rezolve ın R ecuat, ia sin 2x = sinx. (5p)

6. Sa se calculeze raza cercului circumscris triunghiului ABC, pentru care AB = 3, BC = 5,CA = 4. (5p)

SUBIECTUL al II-lea (30p)

1. Fie G = (a,∞), a > 0 s, i x y = xy − ax− ay + a2 + a, x, y ∈ G.

a) Sa se arate ca G este parte stabila relativ la legea de compozit, ie . (5p)

b) Sa se arate ca (G, ) este grup comutativ. (5p)

c) Sa se determine inversul elementului a+ 1 ın grupul G. (5p)

2. Se considera sistemul (S):

x− y + z = 1

x+ y + z = 3

mx+ y + z = 3m

, m ∈ R.

a) Sa se determine valorile lui m pentru care sistemul are solut, ie unica. (5p)

b) Sa se rezolve sistemul pentru m = 2. (5p)

c) Sa se rezolve sistemul pentru m = 1. (5p)

SUBIECTUL al III-lea (30p)

1. Se considera funct, ia f : R→ R, f(x) =x2 − 1

x2 + 1.

a) Sa se determine asimptotele la graficul lui f . (5p)

b) Sa se studieze monotonia lui f . (5p)

c) Sa se determine ecuat, ia tangentei la graficul lui f ın punctul (0,−1). (5p)

2. Fie In =∫ 1

0

xn

7x+ 8dx, n ≥ 1.

a) Sa se calculeze I1. (5p)

b) Sa se arate ca 7In+1 + 8In =1

n+ 1, pentru orice n ≥ 1. (5p)

c) Sa se calculeze limn→∞

In.(5p)

PROBLEME DE MATEMATICA PENTRU EXAMENE 97

Teste pentru admiterea la facultate

Testul 1

Eduard Asadurian 1

SUBIECTUL I

1. Fie mult, imea

M =

A(x) =

Üx 0 x

0 2 0

x 0 x

ê| x ∈ R∗

⊂M3(R).

a) Aratat, i ca (M, ∗), unde A(x) ∗ A(y) = 12A(x)A(y), este un grup comutativ, izomorf

cu grupul multiplicativ R∗.b) Rezolvat, i ecuat, ia

A(x) ∗ A(x) ∗ · · · ∗ A(x)︸ ︷︷ ︸de n ori

= A(x) + A(x) + · · ·+ A(x)︸ ︷︷ ︸de n ori

ın R∗ × N.

2. Sa se rezolve ın C ecuat, ia x4 − x3 + 2x2 + x+ 3 = 0, s,tiind ca are doua radacini a carorsuma este egala cu 2.

SUBIECTUL al II-lea

1. Se considera s, irul (xn)n≥1 dat prin x1 = 4, xn+1 = x2n+2x2n−1

, ∀n ≥ 1. Sa se studieze convergent,a

s, irului (xn).2. Calculat, i

limn→∞

2∫1

enx + 2

enx − 1dx.

SUBIECTUL al III-lea

Fie M s, i N doua puncte pe laturile BC, respectiv AC ale triunghiului ascut, itunghic

ABC astfel ıncat−−→MC = k1

−−→MB s, i

−−→NC = k2

−−→NA. Fie I = AM

⋂BN s, i P = CI

⋂AB.

Presupunem cunoscute lungimile laturilor triunghiului ABC.

1. Aflat, i raportulIC

IP;

2. Deducet, i valorile raportuluiIC

IPın cazurile ın care AM s, i BN sunt:

a) mediane; b) bisectoare; c) ınalt, imi.

1 Lect. univ. dr., Universitatea din Pites,ti, [email protected]

98 PROBLEME DE MATEMATICA PENTRU EXAMENE

Testul 2

Raluca Mihaela Georgescu 2

SUBIECTUL I

Fie mult, imea A =¶a+ b

√2| a, b ∈ Z

©.

1. Sa se arate ca (A,+) este grup abelian.2. Sa se arate ca polinomul f ∈ R[X], f(X) = X3 − 3X2 +X + 1 are o radacina x0 ∈ A, cux0 < 3.

3. Sa se arate ca toate radacinile ecuat, iei x4 − 6x2 + 1 = 0 se afla ın mult, imea A.

SUBIECTUL al II-lea

Fie funct, iile fn : R→ R, fn(x) =xn

x2 + 2x+ 5, n ∈ N?.

1. Sa se determine ecuat, iile asimptotelor la graficul funct, iei f3.2. Sa se determine numarul punctelor de extrem ale funct, iei g = f3 − f2.

3. Sa se calculeze limn→∞

nIn, unde In =1∫0fn(x)dx.

SUBIECTUL al III-lea

In planul de coordonate xOy se considera punctele A(1, 0), B(3, 1) s, i C(1, 3).

1. Daca M este mijlocul segmentului [BC], sa se determine coordonatele unui punct Ddin plan, D 6= A, astfel ıncat ariile triunghiurilor ABC s, i BCD sa fie egale s, i puncteleA, M, D sa fie coliniare.

2. Sa se determine coordonatele centrului cercului circumscris triunghiului ABC.3. Sa se gaseasca doua puncte E s, i F ın cadranul IV astfel ıncat AEFB sa fie patrat.

Testul 3

D.M.I. 3

Algebra

1. Sa se rezolve ecuat, ia 2√x−2 + 5

√x2−3x+2 = 2.

2. Sa se calculeze C1n − 2C2

n + 3C3n + . . .+ (−1)n−1nCn

n .3. Fie En o matrice patratica de ordinul n cu toate elementele egale cu 1. Sa se arate ca:

a) E2n = nEn;

b) In − En este inversabila s, i sa se arate ca inversa ei este egala cu

In −1

n− 1En.

4. Pe mult, imea G = (0,+∞)− 1 se defines,te legea ,,∗” astfel x ∗ y = xln y. Sa se arate ca,,∗” este o lege de compozit, ie pe G s, i sa se demonstreze ca (G, ∗) este grup comutativ.

5. a) Combinari (definit, ie).b) Aratat, i ca numarul submult, imilor unei mult, imi cu n elemente este 2n.c) Aratat, i ca: Ck

n = Cn−kn ; Ck

n = Ckn−1 + Ck−1

n−1.

2 Lect. univ. dr., Universitatea din Pites,ti, [email protected] Universitatea din Pites,ti, [email protected]

PROBLEME DE MATEMATICA PENTRU EXAMENE 99

Analiza Matematica

1. Sa se calculeze limita s, irului an =12 + 22 + 32 + . . .+ n2

n3.

2. Fie funct, ia f : R→ R, f(x) =

ex + x− α , x ≤ 1

x1

x−1 , x > 1, α ∈ R. Sa se studieze continuita-

tea funct, iei.3. Fie funct, ia f : R → R, f(x) = A cosωx + B sinωx, unde ω ∈ R. Sa se demonstreze

egalitateaf ′′(x) + ω2f(x) = 0, ∀ x ∈ R.

4. Se da funct, ia f : R→ R, f(x) =

ex + a , x ≤ 0√

4 + x−√b

x, x > 0

, a, b ∈ R, b > 0.

a) Sa se determine a s, i b astfel ıncat f sa fie continua.b) Sa se calculeze primitivele funct, iei f pentru a s, i b determinat, i la punctul anterior.

5. Sa se calculeze∫ π/40

2tg x+3sin2 x+2 cos2 x

dx.

Geometrie

1. Fie ABC un triunghi s, i D,E, F picioarele ınat, imilor din A, B, respectiv C. Sa se arateca:

a) tiunghiurile AEF , DEC, DBF sunt asemenea cu triunghiul ABC;b) bisectoarele triunghiului DEF coincid cu ınalt, imile triunghiului ABC.

2. Se da un cerc s, i o coarda AB. Prin mijlocul C al arcului AB se duc doua coarde oarecarece intersecteaza cercul ın D s, i E, iar coarda AB respectiv ın F s, i G. Demonstrat, i capatrulaterul DEGF este inscriptibil.

3. a) Fie ABC un triunghi cu lungimile laturilor egale cu 2,√

2, 1 +√

3. Determinat, iunghiurile triunghiului.

b) In orice triunghi avem R(b+ c) ≥ a√bc, notat, iile fiind cele uzuale.

4. Fie ABC un triunghi s, i AD, perpendiculara pe planul sau. Ducem AE ⊥ BD (E ∈ BD)s, i AF ⊥ DC (F ∈ CD). Aratat, i ca patrulaterul BCEF este inscriptibil.

5. Intr-un triunghi ABC fie D s, i E mijloacele laturilor BC, respectiv CA. Stiind ca A(1,−1),D(1, 2), E(2, 0), determinat, i:

a) sa se scrie ecuat, iile laturilor triunghiului ABC;b) ecuat, ia cercului circumscris triunghiului ABC.

(Admiterea la Universitatea din Pites,ti, specializarile Matematica s, i Matematica-Informatica, 1995)

100 PROBLEME DE MATEMATICA PENTRU EXAMENE

Teste grila pentru admiterea la facultate

Testul 1

Sorin Ulmeanu 1

1. Sa se determine mult, imea valorilor funct, iei f : R→ R, f(x) =x2 + x+ 1

x2 − 1.

a) [−3, 3]; b)(−∞,−

√32

] ⋃ [√32,∞

); c)

[−√32,√32

]; d) R; e) (−∞,−1)

⋃(1,∞).

2. Mult, imea solut, iilor inecuat, iei arccos2x

x+ 1>π

3este:

a) (-1,1); b)Ø; c) [-1/3,1]; d) [-1/3,1/3); e) (-1,1/3]3. Determinat, i cardinalul mult, imii

A =

®x ∈ Q | x =

n− 1

n2 + n;n ∈ −3,−2, 1, 2, 3, ..., 12, 13

´.

a) 14; b) 12; c) 10; d) 15; e) 13.4. Sa se determine pentru ce valori reale s, i pozitive x, y pentru care x + y = 50, produsulP = x3y2 ia valoarea maxima.

a) (25,25); b) (10,40); c) (30,20); d) (40,10); e) (20,30).

5. Sa se determine cea mai mica valoare n ∈ N∗ pentru care σn = e, unde σ =

Ñ1 2 3 4 5 6 7 8

6 1 4 3 8 2 5 7

é,

iar e este permutarea identica din S8.

a) n = 8; b) n = 12; c) n = 9; d) n = 8!; e) n = 6.

6. Sa se calculeze limn→∞

[n(a

1n − 1

)]= l, a > 0, a 6= 1.

a) l = ln a; b) l = 1; c) l = e; d) l =∞; e) l = 0.7. Determinat, i funct, iile f : R → R∗ care verifica relat, iile

1f(x)

+ 1f ′(x)

= 1ex

, ∀x ∈ R s, i

f(0) = 1 +√

2.

a)f(x) = ex −√ex + 1; b)f(x) = ex +

√e2x + 1; c)f(x) = −ex +

√ex + 1;

d)f(x) = −ex −√ex + 1; e)f(x) = ex.

8. Fie matricea A =

Ü2 0 0

1 1 0

0 0 1

êcu elemente ın Z3. Atunci inversa acesteia este:

a) A−1 =

Ü2 0 0

2 2 0

0 0 2

ê; b) A−1 =

Ü1 0 0

1 1 0

0 0 1

ê; c) A−1 =

Ü1 0 0

2 2 0

0 0 2

êd) A−1 =

Ü2 0 0

1 1 0

0 0 1

ê; e) A nu este inversabila.

9. Determinat, i valoarea lui I =1∫−1

dx

(ex + 1)(x2 + 1).

a) I = eπ/4; b) I = π/4; c) I = e2π/2; d) I = 1; e) I = e2.

1 Profesor, Colegiul Nat, ional ,,Ion C. Bratianu”, Pites,ti, [email protected]

PROBLEME DE MATEMATICA PENTRU EXAMENE 101

10. Determinat, i o relat, ie de recurent, a ın cadrul s, irului (In)n∈N∗ , unde In =1∫−1

(1 − x2)ndx,

n ∈ N∗.a) In = n

n+1In−1; b) In = 2n

n+1In−1; c) In = 2n

2n+1In−1; d) In = n

2n+1In−1; e) In = 1

n+1In−1.

11. Mult, imea S a solut, iilor inecuat, ieix+ 1

x+ 2≥ 0 este:

a) (−∞,−2); b) (−∞,−2)⋃

(−1,+∞); c) (−∞,−2)⋃

[−1,∞);

d) (−∞,−2]⋃

[−1,∞); e) (−∞,−2]⋃

(−1,∞).12. Mult, imea valorilor funct, iei f : R→ R, f(x) = x2 + x+ 1, ∀ x ∈ R este:

a) R; b)î34,+∞

ä; c)

Ä34,+∞

ä; d)

î−3

4,+∞

ä; e) (0,+∞).

13. Valoarea limitei limx→2

(x− 2) · sin 1

x− 2este:

a) 1; b) 0; c) -1; d)+∞; e) −∞.14. Suma patratelor solut, iilor ecuat, iei 4x − 6 · 2x + 8 = 0 este:

a) 20; b) 1/4; c) 3/2; d) 5; e) 5/4.

15. Daca A =

Ñ1 1

0 1

é, atunci A2 + A+ I2 este:

a)

Ñ1 2

0 1

é; b)

Ñ2 3

0 2

é; c)

Ñ1 3

0 1

é; d)

Ñ3 3

0 3

é; e)

Ñ3 2

0 3

é.

Testul 2

D.M.I. 2

1. Fie functia f : R→ R,

f(x) =

3x− 5 daca x ∈ (−∞, 1)

(m− 2)x− 5 daca x ∈ [1, 3]

x+ 7 daca x ∈ (3,+∞).

Valoarea parametrului m ∈ R pentru care f este strict crescatoare este

a) m ∈ [5, 7]; b) m ∈ [−5, 5]; c) m ∈ (−∞, 7]; d) m ∈ (−2, 4]; e) m ∈ [5,+∞).

2. Fie sistemul

ñ3x+ 1

2

ô+

ñ2y − 1

3

ô= x,ñ

3x− 1

3

ô+

ñ2y + 1

2

ô= y,

unde [a] reprezinta partea ıntreaga a numarului real a. Daca

A =x2 + y2

∣∣∣ (x, y) solutie a sistemului dat,

2 Universitatea din Pites,ti, [email protected]

102 PROBLEME DE MATEMATICA PENTRU EXAMENE

atunci

a) A = 1, 2, 3; b) A = 1, 2; c) A = 0, 3; d) A = 0, 1; e) A = 0, 4.

3. Fie f : R→ R, f(x) = mx2 − (m+ 1)x+ 1,m ∈ R. Daca f(x) ≥ 0,∀x ∈ R, atunci

a) m ∈ (−1,∞); b) m ∈ (0, 1); c) m ∈ (0,∞); d) m = 1; e) m = −1.

4. Solutiile ecuatiei√

3− x+√

2 + x = 3 sunt

a)

x1 = −2

x2 = 2; b)

x1 = −2

x2 = −1; c)

x1 = 1

x2 = 2; d)

x1 = −1

x2 = 3; e)

x1 = −1

x2 = 2.

5. Solutia inecuatiei√x− 5−

√9− x ≥ 1 este

a)[14+√7

2, 9]; b)

î6, 17

2

ó; c) Ø; d) [7, 9]; e) [5, 9].

6. Daca x este solutia ecuatiei

9x − 2x+12 = 2x+

72 − 32x−1,

atunci 4x2 + 2x =

a) 9; b) 12; c) 2; d) 0; e) −12.

7. Solutia ecuatiei C3n + C4

n = n(n− 2) este

a) n = 5; b) n = 4; c) n = 6; d) n = 7; e) n = 1.

8. Fie (bn)n≥1 o progresie geometrica ce satisface relatiile b5 − b1 = 15 si b4 − b2 = 6. Dacaq > 1 este ratia progresiei geometrice, atunci

a) q = 2, 5; b) q = 3; c) q = 8; d) q = 4; e) q = 2.

9. Daca limn→∞

nÄan+

√2 + bn+ cn2

ä= 1, atunci a+ b+ c =

a) 1; b) 2; c) −1; d) 0; e) −2.

10.

limn→∞

12 + 22 + 32 + · · ·+ n2

n3 + n2 + n+ 1=

a) ∞; b) 1; c)1

6; d)

1

3; e) 0.

11.

limx→∞

Çx2 + x+ 1

x2 − x+ 1

å√x2+1

=

a) e2; b) 0; c) 1; d) e−2; e) e.

PROBLEME DE MATEMATICA PENTRU EXAMENE 103

12. Se considera functia

f : R→ R, f(x) =|x| − 3

1 + 2|x|.

Multimea asimptotelor la graficul functiei este

a) y = x+ 1; b) y = x; c) x = −12; d) y = 1

2; e) y = 1

2, x = −1

2

13. Daca F este o primitiva a functiei

f : R→ R, f(x) =xearctg x»(1 + x2)3

, si limx→∞

F (x) =1

2eπ2 ,

atunci F (x) =

a) x1−2xe

arctg x; b) 12

(earctg x + e

arctg x1+x2

); c) x−1

2√1+x2

earctg x; d) 2x+14√1+x2

earctg x;

e) x2

1−2x

(earctg x + e

arctg x1+x2

).

14. Daca1∫α

Ä3x2 + 2x− 1

ädx = 3,

atunci

a) α = 0; b) α =1

2; c) α = −1; d) α = −2; e) α = −3.

15. Aria multimii marginita de graficele functiilor

f, g : R→ R, f(x) = x3, g(x) = x2

si dreptele x = 0 si x = 1 este

a) x+14

; b) 92; c) 1

12; d) 10

3; e) 32

3.

Testul 3

D.M.I. 3

1. Suma solutiilor reale ale ecuatiei ||x| − 1| = 13 este

a) 6; b) 0; c) 1; d) −1; e) −2.

2. Solutiile inecuatieix− x2

x2 + x− 6≥ 0 sunt

a) x ∈ (−1, 4); b) x ∈ [−3,−1) ∪ (1, 2); c) x ∈ (−3, 2); d) x ∈ (−3, 0] ∪ [1, 2);

e) x ∈ (−3, 0] ∪ [2, 3).

3 Universitatea din Pites,ti, [email protected]

104 PROBLEME DE MATEMATICA PENTRU EXAMENE

3. Solutiile sistemului de inecuatii |x+ 1| − x > 2

x2 + 2x− 80 < 0

sunt

a) x ∈ (−1, 5]; b) x ∈ (−8, 10); c) x ∈Ç−10,

3

2

å; d) x ∈ (−10, 8); e) x ∈ (−10,−4).

4. Valoarea numarului n =3»

45 + 29√

2 +3»

45− 29√

2 este

a) n = 3; b) n = 4; c) n = 5; d) n = 6; e) n = 2.

5. Numarul solutiilor ıntregi ale ecuatiei

x2 + 3−√

2x2 − 3x+ 2 =3

2(x+ 1)

este egal cu

a) 1; b) 3; c) 0; d) 4; e) 2.

6. Solutia inecuatieilog 1

3

√x+ 3

log 13(x+ 1)

< 1

este

a) (−1, 0] ∪ (1,∞); b) (−1, 1); c) (−1, 0) ∪ (1,∞); d) [0,∞); e) (−1, 0) ∪ (2,∞).

7. Solutia sistemului

Cyx = Cy+2

x

C2x = 153

este

a)

x = 12

y = 8; b)

x = 18

y = 8; c)

x = 18

y = 16; d)

x = 18

y = 10; e)

x = 16

y = 12.

8. Termenii unei progresii geometrice satisfac relatiile

a1 + a5 = 51 si a2 + a6 = 102.

Daca Sn = 3069, atunci n =

a) 8; b) 11; c) 10; d) 12; e) 9.

9. Limita sirului (an)n>0 cu termenul general

an =n√n!

n+ 1

este

PROBLEME DE MATEMATICA PENTRU EXAMENE 105

a) ∞; b) 1; c) e; d) 0; e) e−1.

10.

limn→∞

√12 + 22 + 32 + · · ·+ n2

n√

3n=

a)1

2; b) 1; c)

1

6; d) ∞; e)

1

3.

11. Numerele reale a si b pentru care

limx→−∞

Ä√x2 + 1 + ax+ b

ä= 2

sunt:

a)

a = 1

b = −2; b)

a = −1

b = −2; c)

a = 1

b = 2; d)

a = 0

b = −1; e)

a = 3

b = −2.

12. Functia

f : R→ R, f(x) =

2x−√a2x2 + ax+ 1 daca x ≤ 1√

x− 1 + |a|√x daca x > 1

este continua pe R daca si numai daca

a) a ∈¶−1, 3

5

©; b) a ∈

¶−2, 5

3

©; c) a ∈

¶−1,−3

5

©; d) a ∈

¶−2, 2

5

©; e) a ∈

¶−1

3, 15

©.

13. Primitivele functiei

f : R→ R, f(x) =1

a2 + x2, (a ∈ R?)

sunt

a) 1aarctg x

a+ C; b) arctg x

a+ C; c) arctg x+ C; d) tg x

a+ C; e) ln

Äax+

√a2 + x2

ä+ C.

14. Se considera functia f : R\−1 → R,

f(x) = limn→∞

1− x+ x2 − x3 + · · ·+ x2n

1 + x+ x2 + x3 + · · ·+ x2n.

Atunci

2∫0

f(x)dx =

a) 2 lne

3; b) 2 ln

2

3; c) 2 ln

3

e; d) 2 ln

4

3; e) ln

4

3.

15. Volumul corpului de rotatie determinat de functia

f : [0, a]→ R, f(x) =Ä√a−√xä2, a > 0,

este egal cu

a) πa3

7; b) πa3

14; c) πa3

4; d) πa3

15; e) πa3

5.

106 PROBLEME DE MATEMATICA PENTRU EXAMENE

Testul 4

D.M.I. 4

1. Se considera functiile

f, g : R→ R, f(x) = |x− 1|+ 2, g(x) = |x− 2|+ 1.

Atunci functia f g are legea de corespondenta

a) (f g)(x) =

x− 4 daca x ≥ 4

x+ 4 daca x < 4. b) (f g)(x) =

−4 daca x ≥ 2

x daca x < 2.

c) (f g)(x) =

−x daca x ≥ 2

4 daca x < 2. d) (f g)(x) =

4 daca x ≥ 2

x daca x < 2.

e) (f g)(x) =

x daca x ≥ 2

4− x daca x < 2.

2. Se considera ecuatia 2x2 − 2(2m− 1)x+ 2m2 + 2m+ 3 = 0, unde m este un parametru real.Radacinile x1, x2 sunt reale si

2−√

3 ≤ x1x2≤ 2 +

√3,

daca si numai daca

a) m ∈ [−4,−1],; b) m ∈ (−∞,−4] ∪ñ−1,−5

8

ô; c) m ∈ (−∞,−4] ∪ [−1,∞);

d) m ∈Ç−∞,−5

8

ô; e) m ∈

ñ−5

8,∞

å.

3. Suma solutiilor reale ale ecuatiei√x− 3 = x− 3 este

a) 8; b) 6; c) 7; d) 10; e) 9.

4. Se considera numerele reale x =3»

5√

2 + 7, y =3»

5√

2− 7, A = x+y si B = x−y. Atunci

a) A ∈ Q, B ∈ R\Q; b) A ∈ R\Q, B ∈ Q; c) A,B ∈ R\Q; d) x, y ∈ Q; e) A,B ∈ Q.

5. Ecuatia√x+ 1 +

√1− x2 = 3

√x2 + 7− 2

a) are solutie unica. b) nu are solutii. c) are doua solutii in intervalul [−2, 0]. d) are treisolutii. e) are doua solutii in intervalul [0, 2].

6. Multimea solutiilor ecuatieilog3 x»log√3x x

= −1

este

4 Universitatea din Pites,ti, [email protected]

PROBLEME DE MATEMATICA PENTRU EXAMENE 107

a)¶19, 13

©; b)

1√3, 19

; c)

13, 1√

3

; d)

¶19

©; e)

1√3

.

7. Termenul din mijloc al dezvoltariiÄ√x+√yä8

este

a) T5 = C48x

2 · y2; b) T7 = C68x

3 · y3; c) T4 = C38x

2 · y√xy; d) T6 = C58x · y2

√xy;

e) T3 = C28x

3 · y.

8. Fie ecuatia x3 + x+m = 0, m ∈ R cu radacinile x1, x2, x3. Daca x51 + x52 + x53 = 10, atuncim =

a) 1; b) 0; c) -1; d) 2; e) −2.

9. Limita sirului (an)n>0 cu termenul general

an =1! + 2! + 3! + · · ·+ n!

(n+ 1)!

este

a) 1; b) 0; c) ∞; d) e; e) e−1.

10.

limn→∞

n4

13 + 23 + 33 + · · ·+ n3=

a) 4; b) 0; c) 1; d)1

2; e) ∞.

11. Daca

L = limx→0

sin 5x− sin 3x

5x

atunci

a) L = 25; b) L = 1

5; c) L = −1

5; d) L = −2

5; e) L = 0.

12. Valoarea parametrului real a, pentru care functia

f : R→ R, f(x) =

(sinx+ ex)1x daca x 6= 0

a daca x = 0

este continua pe R, este

a) a = e3; b) a = 1; c) a = e−5; d) a = e2; e) a = 0.

13. ∫sinx

1 + cos2 xdx =

a) ln√

1 + cos2 x+ C; b) ln(1 + cos2 x) + C; c) arctg (cosx) + C; d) ln√

1− cos2 x+ C;

108 PROBLEME DE MATEMATICA PENTRU EXAMENE

e) −arctg (cosx) + C.

14. Daca

f : [0, 2]→ R, f(x) = max1, x2,

atunci ∫ 2

0exf(x)dx =

a) 4e+ 5; b) 1− 3e2; c) 2e2; d) 2e2 − 1; e) e2.

15. Aria domeniului plan marginit de graficele functiilor

f, g : [0, 1]→ R, f(x) = 1− x, g(x) =√

1− x2

este

a) e− 1; b) 12; c) 2; d) π−2

4; e)

√22.

Testul 5

Raluca Mihaela Georgescu 5

1. Sa se determine x ∈ R astfel ıncat

∣∣∣∣∣∣∣∣x2 − 3 −2

x+ 2 1

∣∣∣∣∣∣∣∣ = 0.

a) x ∈ −1, 1; b) x = −1; c) x = 1; d) x ∈ −1, 0; e) x = 0

2. Sa se calculeze limx→0

tg 22x

sin2 5x.

a) 2/5; b) 2/25; c) 4/5; d) 4/25; e) 1

3. Sa se determine m ∈ R astfel ıncat parabola asociata funct, iei f(x) = x2 − (m+ 3)x+ 1 sa

fie tangenta axei Ox.

a) m ∈ −1,−5; b) m = −1; c) m = 5; d) m ∈ 1,−5; e) m = 0

4. Sa se calculeze limx→∞

(x+ 2)2 − (x− 1)2

2x3 + 3x− 1.

a) ∞; b) 0; c) 1/2; d) 1; e) 5/2

5. Sa se determine termenul liber al polinomului f = (x+ 1)10 + (3x− 1)10.

a) 0; b) 2; c) -1; d) -2; e) 1

6. Sa se calculezee∫1

ln2 x

xdx.

a) 2/3; b) 1/2; c) 1/3; d) 0; e) 1

5 Lect. univ. dr., Universitatea din Pites,ti, [email protected]

PROBLEME DE MATEMATICA PENTRU EXAMENE 109

7. Fie A =

Ü1 i

−i 0

ê. Sa se calculeze A4.

a)

Ü3 2i

−2i 1

ê; b)

Ü5 3i

3i 1

ê; c)

Ü−5 3i

1 i

ê; d)

Ü5 3i

−3i 2

ê; e)

Ü−5 3i

3i 2

ê8. Fie F primitiva funct, iei f : R→ R, f(x) =

√x2 + 1, cu proprietatea ca F (0) = 0. Sa se

determine F (1).

a) 12

Ä√2 + ln(1 +

√2)ä; b)

√3 + ln(1 +

√3); c) 1

2

Ä√2 + ln

√2ä;

d) 12

Ä1 + ln(1 +

√3)ä; e) 1

2

Ä√2 + ln(1− 2

√2)ä

9. Valorile lui k ∈ R pentru care inecuat, ia x2 − (k − 1)x− k + 1 < 0 nu are solut, ie sunt:

a) [-1,3]; b) (2,4]; c) [-3,1]; d) (-3,1); e) Ø

10. Mult, imea solut, iilor ecuat, iei x2 + 3x+ 2 = 0 peste corpul Z5 este

a) 2; b) 3; c) 3, 4; d) 4; e) 2, 411. Aria subgraficului determinat de graficul funct, iei f : R → R, f(x) = x lnx, axa Ox s, i

dreptele de ecuat, ii x = 1, x = e este:

a)e2 − 1

4; b)

e+ 1

4; c)

e2 + 1

4; d)

e2 + 1

2; e)

e+ 1

2

12. Daca x este solut, ia ecuat, iei

Ç2

5

åx·Ç

25

4

åx=

125

8, atunci 3x + 6x este:

a) 9; b) 45; c) 243; d) 241; e) 120

13. Restul ımpart, irii polinomului f = (x+ 1)10 + (x− 1)10 la g = x2 − 3x+ 2 este:

a) (310 − 210 + 1)x+ 211 − 310 − 1; b) (310 + 210 − 1)x+ 211 + 310 − 1;

c) (310 − 210)x+ 211 − 310; d)(310 + 1)x+ 211 − 1; e) (310 + 210 + 1)x+ 211 + 310 + 1

14. Suma solut, iilor reale ale ecuat, iei |||x− 1| − 2| − 1| = 2 este:

a) 3; b) 2; c) 24; d) 6; e) 4

15. Daca ıntr-o progresie aritmetica a1 = 3, a5 − a3 = 10, atunci suma primilor 20 de termeni

ai progresiei este:

a) 1000; b) 1200; c) 1010; d) 1020; e) 1100

110 PROBLEME DE INFORMATICA PENTRU EXAMENE

PROBLEME DE INFORMATICA PENTRU

EXAMENE

Teste pentru examenul de Bacalaureat, specializarea S, tiint, e ale naturii

Testul 1

Marina Luncan 1

Limbajul C/C++

SUBIECTUL I (30 de puncte)

Pentru itemul 1, scriet, i pe foaia de examen litera corespunzatoare raspunsuluicorect.

1. Care este valoarea expresiei C/C++: (3 + 17/2) ∗ 2? (4p.)

a) 23 b) 22 c) 20 d) 21

2. Se considera algoritmul de mai jos, reprezentat ın pseudocod. S-a notat cu x%y restulımpart, irii lui x la y.

citeste n,k (numere naturale)pentru i←2,n executa| daca i%k=0 sau k%i=0 atunci|_ |_ s←s+1daca s>0 atunci scrie s|_ altfel scrie "NU"

a) Scriet, i pe foaia de examen ce se va afis,a daca se citesc ın aceasta ordine numerele 10s, i 3. (6p.)

b) Scriet, i o pereche formata din numere de doua cifre care sa determine afis,area mesajuluiNU. (4p.)

c) Scriet, i ın pseudocod un algoritm echivalent cu cel dat care sa ınlocuiasca structurapentru cu o structura repetitiva cu test init, ial. (6p.)

d) Scriet, i programul C/C++ corespunzator algoritmului dat. (10p.)

SUBIECTUL al II-lea (30 de puncte)

Pentru fiecare dintre itemii 1 s, i 2 scriet, i pe foaia de examen litera corespunzatoareraspunsului corect.

1. Care dintre urmatoarele expresii aritmetice determina patratul sumei dintre a s, i b? (4p.)

a) pow(a,2)+pow(b,2)b) (a+b)*2

c) sqrt(a+b)d) pow(a,2)+pow(b,2)+2*a*b

2. Se considera citit caracterul c (litera mica a alfabetului englez). Care dintre urmatoarelesecvent,e de program determina transformarea caracterului c ın litera mare? (4p.)

1 Profesor, Colegiul Nat, ional ,,Alexandru Odobescu”, Pites,ti, [email protected]

PROBLEME DE INFORMATICA PENTRU EXAMENE 111

S1: c=c+32;S2: c=c-32;

S3: for(i=1;i<=16;i++)c=c-2;

S4: for(i=1;i<=32;i++)c++;

a) S2 s, i S3; b) doar S1; c) doar S2; d) S1 s, i S4.

Scriet, i pe foaia de examen raspunsul pentru fiecare dintre cerint,ele urmatoare.3. O fract, ie este data prin doua numere ıntregi a s, i b, reprezentand numaratorul, respectiv

numitorul acesteia. Scriet, i o secvent, a de instruct, iuni care sa verifice daca fract, ia se poatesimplifica prin numarul d s, i afis,at, i numaratorul s, i numitorul fract, iei simplificate saumesajul NU, daca aceasta nu se poate simplifica prin d. (6p.)

4. Un numar se numes,te ”rar” daca mai mult de jumatate din cifrele sale sunt egale cu 0.

a) Scriet, i un algoritm pseudocod care cites,te repetat numere, pana la ıntalnirea numarului0 s, i afis,eaza pe ecran numerele rare din care s-au extras cifrele egale cu 0. (10p.)

b) Ment, ionat, i rolul fiecarei variabile s, i specificat, i care sunt datele de intrare s, i care suntdatele de ies, ire ale problemei. (6p.)

SUBIECTUL al III-lea (30 de puncte)

Pentru itemul 1, scriet, i pe foaia de examen litera corespunzatoare raspunsuluicorect.

1. Se considera s, irul de numere ıntregi 20, 4, 3, 1, 10, 5. Se ordoneaza s, irul descrescator,folosind metoda bulelor. Dupa cate parcurgeri de s, ir se va obt, ine ordonarea. (4p.)

a) 3 b) 4 c) 2 d) 6

Scriet, i pe foaia de examen raspunsul pentru fiecare dintre cerint,ele urmatoare.2. Inlocuit, i punctele de suspensie, ın secvent,a urmatoare astfel ıncat sa se afis,eze secvent,a

de numere din dreapta. (6p.)

for (i=1;i<=4;i++)for(j=3;j>=1;j--).....................

1 1 1

2 2 2

3 3 3

4 4 4

3. Se cites,te de la tastatura un s, ir cu n elemente, numere reale (n<100). Scriet, i un programC/C++ care determina modificarea s, irului init, ial astfel ca numerele divizibile cu 10 sase afle la ınceputul s, irului, ordinea celorlalte elemente nefiind semnificativa. Elementeles, irului vor fi afis,ate pe ecran, pe o singura linie, despart, ite prin cate un spat, iu. Daca ıns, ir nu exista numere divizibile cu 10, se va afis,a pe ecran mesajul NU EXISTA. (10p.)

Exemplu: Daca, pentru n = 6 se citesc numerele 2 10 3 20 5 6, se va afis,a 10 20 2 35 6.

4. Fis, ierul bac.in cont, ine pe prima linie trei numere ıntregi n, a, b, iar pe urmatoarea linien numere reale. Afis,at, i pe ecran cel mai mare numar real care se afla in intervalul [a, b]s, i numarul sau de aparit, ii, despart, ite prin cate un spat, iu. Daca fis, ierul bac.in nu cont, inenumere din intervalul [a, b] se va afis,a mesajul NU EXISTA.

a) Descriet, i ın limbaj natural un algoritm eficient de rezolvare a problemei. (4p.)b) Scriet, i un program C/C++ pentru rezolvarea problemei. (6p.)

Exemplu: Daca ın fis, ierul bac.in se afla numerele

112 PROBLEME DE INFORMATICA PENTRU EXAMENE

10 2 12

1 13 5 13 9 8 1 9 9 15

pe ecran se va afis,a 9 3.

Testul 2

Sofia-Elena Ciobanu 2

Limbajul C/C++

SUBIECTUL I (30 de puncte)

Pentru itemul 1, ıncercuit, i litera corespunzatoare raspunsului corect.

1. Expresia C/C++: 6/8*3%2 are valoarea: (4p.)

a) 3 b) 0 c) 1 d) 2

2. Se considera algoritmul urmator, reprezentat ın pseudocod:

intreg n, s, ucciteste n (numar natural cu cel putin 2 cifre)cat timp n>0 executa| s←0| cat timp n>0 executa| | uc←n%10| | s←s+uc| |_ n←n/10| daca s > 9 atunci|_ |_ n←sscrie s

a) Scriet, i ce se afis,eaza daca se cites,te numarul 12349. (6p.)b) Scriet, i trei numere distincte astfel ıncat, ın urma executarii algoritmului, pentru

fiecare dintre acestea sa se afis,eze valoarea 7. (4p.)c) Scriet, i ın pseudocod un algoritm echivalent cu cel dat, ınlocuind structura cat timp

. . . executa cu o alta structura repetitiva. (6p.)d) Scriet, i programul C/C++ corespunzator algoritmului dat. (10p.)

SUBIECTUL al II-lea (30 de puncte)

Pentru fiecare dintre itemii 1 s, i 2 ıncercuit, i litera corespunzatoare raspunsuluicorect.

1. Care este numarul total de atribuiri efectuate la executarea urmatoarei secvent,e deinstruct, iuni? (4p.)

t=1; z=19;while(z==19) z = z*z;if (t==z) t = t*3;

2 Profesor, Colegiul Nat, ional”Zinca Golescu”, Pites,ti, stefanescu [email protected]

PROBLEME DE INFORMATICA PENTRU EXAMENE 113

a) 3 b) 4 c) 1 d) 2

2. Variabilele ıntregi x s, i y memoreaza numere naturale Ce expresie de mai jos este echivalentacu expresia (x+y)%3==1 ? (4p.)

a) (x%3==0) && (y%3==0) && (x%3==1) && (y%3==1)b) (x%3==0) && (y%3==1) ‖ (x%3==1) && (y%3==0)c) (x%3==1) ‖ (y%3==0) && (x%3==0) ‖ (y%3==0)d) (x%3==0) && (y%3==0) && (x%3==0) ‖ (y%3==1)

Scriet, i raspunsul pentru fiecare din cerint,ele urmatoare.3. Pentru o dulceat, a sunt memorate pret,ul, precum s, i date despre numele fructului s, i

cantitatea de zahar folosita pentru un borcan.

Variabilele ıntregi pret1, fruct1 s, i zahar1 memoreaza pret,ul, tipul de fruct folosit s, icantitatea de zahar specifica unui anumit tip de dulceat, a, iar variabilele pret2, fruct2 s, izahar2 memoreaza pret,ul, tipul de fruct folosit s, i cantitatea de zahar specifica unui alttip de dulceat, a.

Sa se scrie o secvent, a de instruct, iuni C/C++ care, ın urma executarii, sa afis,eze pret,uldulcet,ei cu o cantitate mai mica de zahar, iar daca ambele borcane de dulceat, a au aceeas, icantitate de zahar sa se afis,eze mesajul ,,ZAHAR IN MOD EGAL”. (6p.)

4. Se citesc n numere naturale. Calculat, i produsul tuturor numerelor care ıncep cu cifra 7.

a) Scriet, i ın pseudocod algoritmul de rezolvare. (10p.)b) Precizat, i rolul tuturor variabilelor care au intervenit ın algoritmul realizat s, i indicat, i

datele de intrare s, i datele de ies, ire ale problemei enunt,ate. (6p.)

SUBIECTUL al III-lea (30 de puncte)

Pentru itemul 1, scriet, i litera corespunzatoare raspunsului corect.

1. Variabilele a s, i b sunt de tip ıntreg s, i memoreaza numere naturale nenule. Indicat, iinstruct, iunea care poate ınlocui punctele de suspensie, astfel ıncat ın urma executariisecvent,ei obt, inute sa se interschimbe valorile celor doua variabile.

..........b=a-b;a=a-b;

a) a=a+b b) a=a-b c) b=b+a d) a=b-a (4p.)

Scriet, i raspunsul pentru fiecare dintre cerint,ele urmatoare.2. Variabilele i s, i j sunt de tip ıntreg. Scriet, i expresia care poate ınlocui punctele de suspensie,

astfel ıncat, ın urma executarii secvent,ei, sa se afis,eze suma elementelor pare ce se afla peprimul patrat concentric al unei matrice cu 8 linii s, i 8 coloane, numerotate de la 1 la 8.(6p.)

for(i=1; i<=8; i++)for(j=1; j<=8; j++)

if (.................)if(a[i][j]%2==0)

s=s+a[i][j];cout <<s;

114 PROBLEME DE INFORMATICA PENTRU EXAMENE

3. Scriet, i un program C/C++ care cites,te de la tastatura un numar natural n s, i cele nelemente ale unui tablou unidimensional, numere ıntregi. Ordonat, i descrescator elementeleaflate pe pozit, iile impare ale tabloului unidimensional. (10p.)

4. Fis, ierul Numere.txt cont, ine pe o singura linie, separate prin cate un spat, iu cel mult 50de numere ıntregi. Scriet, i un program care cites,te numerele din fis, ierul Numere.txt s, iafis,eaza pe ecran, separate prin cate un spat, iu ın ordine crescatoare, toate numerele dinfis, ier care au proprietatea de a fi palindrom. Daca fis, ierul nu cont, ine astfel de numere seva afis,a mesajul NU EXISTA.

a) Descriet, i ın limbaj natural algoritmul utilizat. (4p.)b) Scriet, i programul C/C++ corespunzator. (6p.)

Testul 3

Maria Miroiu 3

Limbajul C/C++

SUBIECTUL I (30 de puncte)

Pentru itemul 1, ıncercuit, i litera corespunzatoare raspunsului corect.

1. Care este expresia echivalenta cu expresia C/C++: !(x>=0 && x<=9)? (4p.)

a) x<0 && x>9;b) x>=0 ||x<=9;

c) x<0 ||x>9;d) !(x>=0) && (x>9.

2. Se considera algoritmul de mai jos, descris ın pseudocod. S-a notat cu x%y restul ımpart, iriilui x la y.

citeste n (numar natural nenul)s←0cat timp n>0 executa| c←n%10| daca c%2=1 atunci| | pentru i←1,c executa| | _ |_ s←s+i|_ n←n/10scrie s

a) Scriet, i ce se va afis,a daca pentru variabila n se cites,te valoarea (6p.)b) Scriet, i un numar ıntreg strict pozitiv ce poate fi citit ca valoare a variabilei n astfel

ıncat algoritmul sa afis,eze valoarea 0. (4p.)c) Scriet, i ın pseudocod un algoritm, echivalent cu cel dat, ınlocuind structura pentru

... executa cu o structura repetitiva cu test init, ial. (6p.)d) Scriet, i programul C/C++ corespunzator algoritmului dat. (10p.)

SUBIECTUL al II-lea (30 de puncte)

Pentru fiecare dintre itemii 1 s, i 2 scriet, i pe foaia de examen litera corespunzatoareraspunsului corect.

1. Presupunand ca lungimile catetelor unui triunghi dreptunghic sunt memorate ın variabilelec1 s, i c2, care este expresia C/C++ cu care se poate determina lungimea ipotenuzei?

3 Lect. univ. dr., Universitatea din Pites,ti, [email protected]

PROBLEME DE INFORMATICA PENTRU EXAMENE 115

a) pow(c1,2)+pow(c2,2)b) sqrt(pow(c1,2)+pow(c2,2))

c) sqrt(c1+c2)d) pow(c1,2)+pow(c2,2)+2*c1*c2

2. Se considera secvent,a de cod C/C++ de mai jos, ın care n memoreaza valoarea unuinumar natural nenul dat, iar toate celelalte variabile sunt de tip ıntreg. (4p.)

ucp = 1;for (d=1; d<=n; d++)

if (n%d==0)......

Expresia care poate ınlocui punctele de suspensie astfel ıncat la sfars, itul secvent,ei C/C++obt, inute variabila ucp sa memoreze ultima cifra a produsului tuturor divizorilor variabilein este: (4p.)

a) ucp=(ucp+d)%10;b) ucp=n/d%10*upc;

c) ucp=d%10*ucp;d) ucp=(ucp*d)%10;

Scriet, i raspunsul pentru fiecare din cerint,ele urmatoare.3. Se considera doua date de nas,tere definite prin variabile ıntregi zi1, luna1, an1, respectiv

zi2, luna2, an2. Scriet, i o expresie C/C++ care sa aiba valoarea 1 daca cele doua persoanesunt nascute ın aceeas, i zi s, i aceeas, i luna, dar ani diferit, i. (6p.)

4. Pentru un numar natural, cifra de ordin 1 este cifra unitat, ilor, cifra de ordin 2 este cifrazecilor, cifra de ordin 3 este cifra sutelor, s, .a.m.d. Rotunjirea unui numar la cifra de ordink se realizeaza ın funct, ie de valoarea cifrei imediat din dreapta. De exemplu, pentrun=27432 rotunjirea la cifra de ordin k=2 (cifra zecilor) conduce la numarul 27430 pentruca cifra unitat, ilor este mai mica decat 5, iar pentru n=27472 rotunjire la cifra de ordink=3 (cifra sutelor) conduce la 27500 pentru ca cifra zecilor este mai mare sau egala decat5.

a) Scriet, i un algoritm ın pseudocod care cites,te doua numere naturale n s, i k (n≥1,k≥1) s, i determina numarul natural obt, inut prin rotunjirea lui n la ordinul k dacavariabila k are valoarea mai mica decat numarul de cifre ale lui n, iar ın caz contrarafis,eaza mesajul ,,Eroare”. (10p.)

b) Ment, ionat, i rolul fiecarei variabile s, i specificat, i care sunt datele de intrare s, i care suntdatele de ies, ire ale problemei. (6p.)

SUBIECTUL al III-lea (30 de puncte)

Scriet, i pe foaia de examen raspunsul corect pentru fiecare dintre cerint,eleurmatoare:

1. Indicat, i expresia care poate ınlocui punctele de suspensie astfel ıncat, ın urma executariisecvent,ei obt, inute, sa se afis,eze matricea de mai jos. (4p.)

for(i=1;i<=4;i++) for(j=1;j<=4;j++)

cout < <......; | printf ("%d" ,......);cout <<endl; | printf ("%n");

0 0 0 0

0 1 2 3

0 1 3 4

0 2 4 6

116 PROBLEME DE INFORMATICA PENTRU EXAMENE

a) i*j/2b) i*(j-1)/2

c) (i-1)*j/2d) (i-1)*(j-1)/2

Scriet, i pe foaia de examen raspunsul corect pentru fiecare dintre cerint,eleurmatoare

2. In secvent,a de mai jos, toate variabilele sunt de tip ıntreg. Inlocuit, i punctele de suspensieastfel ıncat, ın urma executarii secvent,ei obt, inute, valoarea variabilei ok sa fie 1 dacaexista cel put, in un numar par ıntre cele 10 numere citite, sau valoarea 0 ın caz contrar.(6p.)

ok = ......;for (i=1;i<=10;i++) cin >>k; | scanf ("%d",&k);

......;

3. Se citesc de la tastatura valorile a doua variabile ıntregi, n (2<n<100) s, i k (1<k<n),apoi un s, ir de n elemente, numere ıntregi. Scriet, i un program C/C++ care determinamodificarea s, irului init, ial, astfel ıncat primele k elemente sa fie ordonate descrescator, iarultimele n-k elemente sa fie ordonate crescator. Elementele s, irului modificat vor fi afis,atepe ecran, pe o singura linie, despart, ite prin cate un spat, iu. (10p.)

Exemplu: Pentru n = 8, k=3 s, i elementele 4 7 3 8 9 16 10 6, se va afis,a 7 4 3 6 89 10 16.

4. Fis, ierul bac.txt cont, ine pe prima linie valoarea naturala nenula a variabilei n, iar peurmatoarea linie un s, ir de n numere naturale, despart, ite prin spat, ii. Afis,at, i pe ecranvaloarea para cea mai mare s, i numarul/numerele de ordine din s, ir al/ale valorii paremaxima, despart, ite printr-un spat, iu. Daca fis, ierul bac.txt nu cont, ine numere pare, se vaafis,a pe ecran mesajul Nu exista.

a) Descriet, i ın limbaj natural un algoritm eficient de rezolvare a problemei. (4p.)b) Scriet, i un program C/C++ pentru rezolvarea problemei. (6p.)

Exemplu: Daca ın fis, ierul bac.txt se afla numerele:

9

3 16 2 7 15 4 16 6 8

pe ecran se va afis,a 16 2 7.

Testul 4

Maria Miroiu 4

Limbajul Pascal

SUBIECTUL I (30 de puncte)

Pentru itemul 1, scriet, i pe foaia de examen litera corespunzatoare raspunsuluicorect.

1. Care este expresia echivalenta cu expresia Pascal: not((x>=0) and (x<=9))? (4p.)

4 Lect. univ. dr., Universitatea din Pites,ti, [email protected]

PROBLEME DE INFORMATICA PENTRU EXAMENE 117

a) (x<0) and (x>9)b) (x>=0) or (x<=9)

c) (x<0) or (x>9)d) not(x>=0) and (x>9)

2. Se considera algoritmul urmator, descris ın pseudocod. S-a notat cu x%y restul ımpart, iriilui x la y.

a) Scriet, i ce se va afis,a daca pentru variabila n se cites,te valoarea 1234. (6p.)b) Scriet, i un numar ıntreg strict pozitiv ce poate fi citit ca valoare a variabilei n astfel

ıncat algoritmul sa afis,eze valoarea 0. (4p.)c) Scriet, i ın pseudocod un algoritm, echivalent cu cel dat, ınlocuind structura pentru

... executa cu o structura repetitiva cu test init, ial. (6p.)d) Scriet, i programul Pascal corespunzator algoritmului dat. (10p.)

citeste n (numar natural nenul)s←0cat timp n>0 executa| c←n%10| daca c%2=1 atunci| | pentru i←1,c executa| | _ |_ s←s+i|_ n←n/10scrie s

SUBIECTUL al II-lea (30 de puncte)

Pentru fiecare dintre itemii 1 s, i 2 scriet, i pe foaia de examen litera corespunzatoareraspunsului corect.

1. Presupunand ca lungimile catetelor unui triunghi dreptunghic sunt memorate ın variabilelec1 s, i c2, care este expresia Pascal cu care se poate determina lungimea ipotenuzei?(4p.)

a) sqr(c1)+sqr(c2)b) sqrt(sqr(c1)+sqr(c2))

c) sqrt(c1+c2)d) sqr(c1)+sqr(c2)+2*c1*c2

2. Se considera secvent,a de cod Pascal de mai jos, ın care n memoreaza valoarea unui numarnatural nenul dat, iar toate celelalte variabile sunt de tip ıntreg.

ucp :=1;for d:=1 to n do

if n mod d=0 then......

Expresia care poate ınlocui punctele de suspensie astfel ıncat la sfars, itul secvent,ei Pascalobt, inute variabila ucp sa memoreze ultima cifra a produsului tuturor divizorilor variabilein este: (4p.)

a) ucp:=(ucp+d) mod 10;b) ucp:=n/d mod 10*upc;

c) ucp:=d mod 10*ucp;d) ucp:=(ucp*d) mod 10.

Scriet, i pe foaia de examen raspunsul corect pentru fiecare dintre cerint,eleurmatoare:

3. Se considera doua date de nas,tere definite prin variabile ıntregi zi1, luna1, an1, respectivzi2, luna2, an2. Scriet, i o expresie Pascal care sa aiba valoarea 1 daca cele doua persoanesunt nascute ın aceeas, i zi s, i aceeas, i luna, dar ani diferit, i. (6p.)

118 PROBLEME DE INFORMATICA PENTRU EXAMENE

4. Pentru un numar natural, cifra de ordin 1 este cifra unitat, ilor, cifra de ordin 2 este cifrazecilor, cifra de ordin 3 este cifra sutelor, s, .a.m.d. Rotunjirea unui numar la cifra de ordink se realizeaza ın funct, ie de valoarea cifrei imediat din dreapta. De exemplu, pentrun=27432 rotunjirea la cifra de ordin k=2 (cifra zecilor) conduce la numarul 27430 pentruca cifra unitat, ilor este mai mica decat 5, iar pentru n=27472 rotunjire la cifra de ordink=3 (cifra sutelor) conduce la 27500 pentru ca cifra zecilor este mai mare sau egala decat5.

a) Scriet, i un algoritm ın pseudocod care cites,te doua numere naturale n s, i k (n≥1,k≥1) s, i determina numarul natural obt, inut prin rotunjirea lui n la ordinul k dacavariabila k are valoarea mai mica decat numarul de cifre ale lui n, iar ın caz contrarafis,eaza mesajul ,,Eroare”. (10p.)

b) Ment, ionat, i rolul fiecarei variabile s, i specificat, i care sunt datele de intrare s, i care suntdatele de ies, ire ale problemei. (6p.)

SUBIECTUL al III-lea (30 de puncte)

Pentru itemul 1 scriet, i pe foaia de examen litera corespunzatoare raspunsuluicorect.

1. Indicat, i expresia care poate ınlocui punctele de suspensie astfel ıncat, ın urma executariisecvent,ei obt, inute, sa se afis,eze matricea de mai jos. (4p.)

for i:=1 to 4 dobegin for j:=1 to 4 do

write (......);writeln;

end;

0 0 0 0

0 1 2 3

0 1 3 4

0 2 4 6

a) i*j div 2b) i*(j-1) div 2

c) (i-1)*j div 2d) (i-1)*(j-1) div 2

2. In secvent,a de mai jos, toate variabilele sunt de tip ıntreg. Inlocuit, i punctele de suspensieastfel ıncat, ın urma executarii secvent,ei obt, inute, valoarea variabilei ok sa fie true dacaexista cel put, in un numar par ıntre cele 10 numere citite, sau valoarea false ın caz contrar.(6p.)

ok := ......;for i:=1 to 10 dobegin

read(k);......;

end;

3. Se citesc de la tastatura valorile a doua variabile ıntregi, n (2<n<100) s, i k (1<k<n),apoi un s, ir de n elemente, numere ıntregi. Scriet, i un program Pascal care determinamodificarea s, irului init, ial, astfel ıncat primele k elemente sa fie ordonate descrescator, iarultimele n-k elemente sa fie ordonate crescator. Elementele s, irului modificat vor fi afis,atepe ecran, pe o singura linie, despart, ite prin cate un spat, iu. (10p.)

Exemplu: Pentru n = 8, k=3 s, i elementele 4 7 3 8 9 16 10 6, se va afis,a 7 4 3 6 89 10 16.

4. Fis, ierul bac.txt cont, ine pe prima linie valoarea naturala nenula a variabilei n, iar peurmatoarea linie un s, ir de n numere naturale, despart, ite prin spat, ii. Afis,at, i pe ecranvaloarea para cea mai mare s, i numarul/numerele de ordine din s, ir al/ale valorii pare

PROBLEME DE INFORMATICA PENTRU EXAMENE 119

maxima, despart, ite printr-un spat, iu. Daca fis, ierul bac.txt nu cont, ine numere pare, se vaafis,a pe ecran mesajul Nu exista.

a) Descriet, i ın limbaj natural un algoritm eficient de rezolvare a problemei. (4p.)b) Scriet, i un program Pascal pentru rezolvarea problemei. (6p.)

Exemplu: Daca ın fis, ierul bac.txt se afla numerele:

93 16 2 7 15 4 16 6 8

pe ecran se va afis,a 16 2 7.

120 PROBLEME DE INFORMATICA PENTRU EXAMENE

Teste pentru examenul de Bacalaureat, specializareaMatematica-Informatica

Testul 1

Cristina Constantinescu 1

Limbajul C/C++

SUBIECTUL I (30 de puncte)

Pentru itemul 1, scriet, i pe foaia de examen litera corespunzatoare raspunsuluicorect.

1. Indicat, i expresia care are valoarea 1 daca s, i numai daca valorile variabilelor x se afla ınafara intervalului (-3,3). (4p.)

a) x*x - 9 <= 0b) (x > -3 || x < 3)

c) x*x - 9 >= 0d) !(x < -3 && x > 3)

2. Se considera algoritmul urmator, reprezentat ın pseudocod. S-a notat cu x%y restulımpart, irii lui x la y.

citeste c (numar natural cuprins intre 1 si 9)citeste n (numar natural nenul)x ← 1y ← 0pentru i = 1, c executa| x ← x * 10|_ y ← y * 10 + 9x ← x / 10cat timp (x<=y si x<n) executa| z ← x| i ← z % 10| z ← z / 10| cat timp (z 6= 0 si z%10 = i) executa| |_ z ← z / 10| daca z = 0 atunci| |_ scrie x|_ x ← x + 1

a) Scriet, i numarul afis,at daca pentru variabila c se cites,te valoarea 4, iar pentru n se cites,tevaloarea 8784. (6p.)

b) Scriet, i un numar ce poate fi citit pentru c astfel ıncat daca pentru n se cites,te valoarea300, algoritmul sa nu afis,eze nimic. (4p.)

c) Scriet, i ın pseudocod un algoritm, echivalent cu cel dat, ın care sa se ınlocuiasca secvent,ace cont, ine cele doua structuri repetitive cu test init, ial cu o secvent, a ce cont, ine o singurastructura repetitiva. (6p.)

d) Scriet, i programul C/C++ corespunzator algoritmului dat. (10p.)

1 Profesor, Colegiul Nat, ional ,,Zinca Golescu”, [email protected]

PROBLEME DE INFORMATICA PENTRU EXAMENE 121

SUBIECTUL al II-lea (30 de puncte)

Pentru fiecare dintre itemii 1 s, i 2 scriet, i pe foaia de examen litera corespunzatoareraspunsului corect.

1. Se considera graful neorientat dat prin ma-tricea de adiacent, a alaturata.

Stabilit, i daca se poate obt, ine un arboreprin eliminarea unora dintre muchiile gra-fului. (4p.)

0 1 0 1 1

1 0 0 0 1

0 0 0 1 1

1 0 1 0 0

1 1 1 0 0

a) Da, prin eliminarea exact a unei muchii.b) Da, prin eliminarea exact a doua muchii.

c) Da, prin eliminarea exact a trei muchii.d) Nu.

2. Intr-un graf neorientat cu 10 muchii, fiecare nod are gradul 2. Care este numarul maximde componente conexe din care poate fi format graful? (4p.)

a) 1 b) 3 c) 4 d) 5

Scriet, i pe foaia de examen raspunsul pentru fiecare dintre cerint,ele urmatoare.3. Variabilele e1 s, i e2, declarate alaturat, memoreaza numele s, i data aparit, iei pentru cate o

carte. Stiind ca cele doua cart, i au aparut ın ani diferit, i, scriet, i secvent,a de instruct, iunicare afis,eaza pe ecran denumirea cart, ii aparuta mai recent. (6p.)

struct data_a int an , luna , zi; ;struct carte char nume [20]; data_a d; e1, e2;

4. Variabila n este de tip ıntreg (n par, 1≤n≤25), iar variabila a este un tablou bidimensionalcu n linii s, i n coloane, numerotate de la 1 la n, tablou care memoreaza elementele pozitivecu maxim 4 cifre. Scriet, i o secvent, a de program C/C++ care sa afis,eze pe ecran sumaelementelor aflate strict deasupra celor doua diagonale, diagonala principala s, i diagonalasecundara a matricei a. Se vor declara convenabil celelalte variabilele folosite ın secvent, a.

Exemplu: pentru n = 4 s, i a =

1 2 3 4

5 6 7 8

9 10 11 12

13 14 15 16

se va calcula 2+3=5 s, i se va afis,a 5.

(6p.)5. Se considera un text cu cel mult 100 de caractere (litere mici ale alfabetului englez, cifre,

spat, ii) ın care cuvintele sunt separate prin unul sau mai multe spat, ii. Un cuvant esteformat numai din litere sau numai din caractere cifre. Inaintea primului cuvant s, i dupaultimul cuvant nu exista spat, ii.

Scriet, i un program C/C++ care cites,te de la tastatura un cuvant s, i apoi determina s, iafis,eaza pe ecran numarul cuvintelor formate doar din caractere cifre, precum s, i textulobt, inut din textul init, ial prin ınlocuirea cuvantului 10 cu zece s, i s,tergerea celorlaltecuvinte formate doar din caractere cifre.

Exemplu: pentru textul

Ana are de rezolvat 10 probleme la informatica 100 la matematica 23 la chimie10 la fizica

122 PROBLEME DE INFORMATICA PENTRU EXAMENE

se va afis,a4Ana are de rezolvat zece probleme la informatica la matematica la chimie zecela fizica (10p.)

SUBIECTUL al III-lea (30 de puncte)

Pentru itemul 1, scriet, i pe foaia de examen litera corespunzatoare raspunsuluicorect.

1. Se considera un numar natural nenul n avand exact k cifre, cifrele lui fiind distincte douacate doua, iar printre cele k cifre se gases,te s, i cifra 0. Permutand cifrele lui n se obt, inalte numere naturale. Cate dintre numerele obt, inute, inclusiv n, au exact k cifre? (4p.)

a) k!-(k-1)! b) k! c) (k-1)! d) (k+1)!

Scriet, i pe foaia de examen raspunsul pentru fiecare dintre cerint,ele urmatoare.

2. Se considera subprogramul f definitalaturat, ın care a este numar ıntreg nenul.Ce se afis,eaza pe ecran la apelul lui f(3),dar la apelul lui f(123)? (6p.)

void f(int a)if (a>0)

cout <<a%10;f(a/10);cout <<a%10;

3. Se considera subprogramul divizori, cu trei parametri. Primeste prin intermediul parame-trului n un numar natural nenul 2≤n≤10000 s, i returneaza prin intermediul parametrilord1 s, i d2 cel mai mic, respectiv cel mai mare divizor prim al sau (1<d1≤d2≤n).

Pentru n=6, se obt, in d1=2 s, i d2=3, iar pentru n=7 se obt, in d1=7 s, i d2=7.

a) Scriet, i definit, ia completa a subprogramului divizori. (6p.)b) Scriet, i un program care cites,te doua numere naturale a s, i b, de cel mult 4 cifre

fiecare, s, i determina folosind apeluri utile ale subprogramului divizori toate numerelenaturale cuprinse ın intervalul [a,b] cu proprietatea ca sunt numere prime. Numereledeterminate se afis,eaza pe ecran cu spat, ii ıntre ele.

Exemplu: Pentru a=12, b=20 se vor afis,a numerele: 13 17 19 (4p.)4. Fis, ierul text numere.in memoreaza cel mult 1000000000 de numere naturale s, i cel

put, in 2, fiecare cu cel mult noua cifre. Numerele sunt separate prin cate un spat, iu.

Se considera ca un s, ir format din cel put, in trei termeni formeaza un ,,platou” dacadiferent,a dintre oricare termen al acestuia, ıncepand cu cel de-al doilea, s, i cel aflat pepozit, ia anterioara ın s, ir este constanta s, i mai mare ca zero.

Se cere sa se citeasca numerele din fis, ierul numere.in s, i sa se afis,eze pe ecran numarulmaxim de termeni ai unei secvent,e din s, ir, secvent, a care formeaza un ,,platou”. Daca nuexista astfel de secvent, a se afis,eaza pe ecran mesajul Nu exista.

Se utilizeaza un algoritm eficient din punctul de vedere al memoriei s, i al timpului deexecutare.

Exemplu: daca fis, ierul numere.in cont, ine numerele

PROBLEME DE INFORMATICA PENTRU EXAMENE 123

3 8 13 18 7 8 1 0 -4 -8 9 13 17 21 25 -6 -1 3 7 17 18

se afis,eaza pe ecran valoarea 5 (corespunzatoare secvent,ei 9 13 17 21 25, care reprezintao secvent, a mai lunga decat “platourile” cu elementele: 3 8 13 18 s, i -1 3 7 s, i lungimile 4,respectiv 3).

a) Descriet, i ın limbaj natural algoritmul utilizat, justificand eficient,a acestuia. (4p.)b) Scriet, i programul C/C++ corespunzator algoritmului descris. (6p.)

Testul 2

Nicoleta Voica 2, Adrian Voica 3

Limbajul C/C++

SUBIECTUL I (30 de puncte)

Pentru itemul 1, scriet, i pe foaia de examen litera corespunzatoare raspunsuluicorect.

1. Variabilele a, b, c, d s, i x sunt de tip real. Care dintre urmatoarele expresii C/C++ auvaloarea 1 daca s, i numai daca x /∈ [a, b] ∪ [c, d] ∪ −1, 0, 1? (4p.)

a) x>=a && x<=b ||x>=c && x<=d ||x==-1 ||x==0 ||x==1b) !(x>=a && x<=b) && !(x>=c && x<=d) && !( x==-1 ||x==0 ||x==1)c) x<a && x>b ||x<=c && x<=d ||(x!=-1 && x!=0 && x!=1)d) (x<a ||x>b ||x<c ||x>d) && (x!=-1 && x!=0 && x!=1)

Scriet, i pe foaia de examen raspunsul pentru fiecare dintre cerint,ele urmatoare.2. Se considera algoritmul de mai jos, scris ın pseudocod.

citeste x (numar intreg)m←0cat timp x>0 executa| d←2| k←0| y←x| cat timp y 6=1 executa| | p←0| | cat timp y%d=0 executa| | | y←[y/d]| | |_ p←1| |_ k←k+p; d←d+1| daca k≥m atunci m←k| |_ nr←x|_ citeste x (numar intreg)scrie m, nr

S-a notat cu [x] partea ıntreaga a numarului real x iar cu x%y restul ımpart, irii numaruluiıntreg x la numarul ıntreg nenul y.

2 Profesor, Colegiul Nat, ional ,,Zinca Golescu”, Pites,ti, [email protected] Profesor, Liceul Teoretic

”Ion Barbu”, Pites,ti, [email protected]

124 PROBLEME DE INFORMATICA PENTRU EXAMENE

a) Scriet, i ce valori vor fi afis,ate daca se citesc pe rand valorile: 15, 13, 305, 12600,72, 1155, 32, 56, 0. (6p.)

b) Precizat, i doua seturi de date de intrare distincte astfel ıncat sa se afis,eze pentru m,de fiecare data, valoarea 1. (6p.)

c) Scriet, i ın pseudocod un algoritm echivalent cu cel dat care sa utilizeze doar douastructuri repetitive. (4p.)

d) Scriet, i programul C/C++ corespunzator algoritmului dat. (10p.)

SUBIECTUL al II-lea (30 de puncte)

1. Fie declararile de mai jos

struct bac1

float a, b;;

struct bac2int x, y;struct bac1 z;float t;

;

si urmatoarele declaratii: bac1 y,b;bac2 x,a;

Care dintre urmatoarele constructii este corecta? (4p.)

a) x.b.z b) b.a.z c) a.z.a d) a.y.b

2. Se considera o structura statica de tip coada s, i o structura statica de tip stiva. Notam cuADS(x) operat, ia prin care se adauga informat, ia x ın stiva, ADC(x) operat, ia prin care seadauga informat, ia x ın coada, ELS operat, ia prin care se elimina un element din stiva s, i seadauga ın coada, ELC operat, ia prin care se elimina un element din coada s, i de adaugaın stiva. Care este cont, inutul stivei s, i cont, inutul cozii ın urma executarii urmatoareloroperat, ii?

ADS(5), ADS(8), ADC(3), ELS, ADC(7), ADS(1), ELC, ELC, ADS(2), ELS, ADC(6)(4p.)

a)S : 5, 1, 7, 8

C : 3, 2, 6b)

S : 5, 1, 3, 8

C : 6, 2, 7c)

S : 5, 1, 3, 8;

C : 7, 2, 6d)

S : 5, 1, 7, 8

C : 6, 2, 3

3. Se considera graful neorientat cu 10 noduri, reprezentat prin urmatoarea lista de muchii:[1, 4], [1, 5], [1, 7], [2, 6], [3, 4], [3, 10], [5, 6], [7, 8], [7, 9].

Sa se verifice daca graful este arbore. In caz afirmativ sa se precizeze lungimea celuimai lung lant, elementar din arbore, iar ın caz contrar sa se precizeze cate muchii trebuieeliminate sau adaugate ın graf astfel ıncat sa devina arbore. (6p.)

4. Stiind ca x este un tablou bidimensional cu 4 linii s, i 4 coloane (numerotate de la 1 la 4)s, i elemente de tip ıntreg sa se precizeze care va fi cont, inutul tabloului dupa executareasecvent,ei de instruct, iuni de mai jos? (6p.)

a=1; b=0;for (i=4; i>=1; i--)

for (j=4; j>=i; j--)x[i][j] = a+b;c = x[i][j];

PROBLEME DE INFORMATICA PENTRU EXAMENE 125

x[j][i] = c;a = b; b = c;

5. Scriet, i un program ın C/C++ care cites,te un text de maxim 250 de caractere litere alealfabetului englez sau spat, ii s, i determina s, i afis,eaza pe ecran numarul total de cuvinte dintext, iar pe urmatorul rand, separate prin cate un spat, iu, cuvintele din text ın care primuls, i ultimul caracter sunt ambele vocale sau ambele consoane. (10p.)

Exemplu: pentru textul La bacalaureat la proba de Informatica a fost un su-biect cu un sir de caracterese va afis,a:15bacalaureat Informatica a fost subiect sir

SUBIECTUL al III-lea (30 de puncte)

1. Utilizand metoda backtracking, sunt generate ın ordine lexicografica toate anagramelecuvantului ecran. Sa se precizeze care sunt cuvintele generate imediat ınainte s, i dupasecvent,a urmatoare: ernac, ernca, nacer . (4p.)

a) ercan,nacre

b) ercna,narce

c) ercna,nacre

d) ercan,narce

2. Fie urmatorul subprogram recursiv:

void bac (int x, int d) if(d<=x/d)

if(x%d==0) cout <<d<<" ";bac(x, d+1);if(x%d==0) cout <<x/d<<" ";

Ce va afis,a subprogramul ın urma apelurilor bac(13, 1) s, i bac(56, 1)? (6p.)3. a) Scriet, i definit, ia completa a subprogramului bac cu doi parametri, care primes,te prin

intermediul primului parametru x un numar natural nenul de minim 2 cifre s, i maxim9 cifre nenule s, i returneaza prin intermediul celui de-al doilea parametru, y, numarulobt, inut din x prin eliminarea primei s, i a ultimei cifre. Subprogramul returneazavaloarea 1 daca numarul x ıncepe s, i se termina cu aceeas, i cifra. (6p.)

Exemplu: daca x=12341, subprogramul returneaza valoarea 1 deoarece numarul xıncepe s, i se termina cu aceeas, i cifra, iar y=234.

b) Scriet, i un program C/C++ care cites,te de la tastatura n numere naturale nenule(1≤n≤100) de minim 2 cifre s, i maxim 9 cifre nenule s, i folosind apeluri utile alesubprogramului de la punctul a) determina s, i afis,eaza pe ecran cate dintre cele nnumere sunt palindrom. In cazul ın care nu exista ın s, ir numere palindrom se vaafis,a un mesaj corespunzator. (4p.)

Exemplu: pentru n=5 s, i numerele 12341, 454, 11, 89, 4567, se va afis,a 2(numerele 454 s, i 11).

4. Fis, ierul text bac.in cont, ine cel mult 100000 de numere naturale.

a) Folosind un algoritm eficient din punct de vedere al memoriei utilizate s, i al timpului

126 PROBLEME DE INFORMATICA PENTRU EXAMENE

de executare scriet, i un program C/C++ care cites,te numerele din fis, ier s, i determinaprimul s, i ultimul numar din s, ir, precum s, i pozit, iile acestora, care au cele mai multecifre egale cu 1 ın scrierea lor ın baza 2.

Rezultatul se va afis,a ın fis, ierul text bac.out . (6p.)b) Descriet, i ın limbaj natural metoda utilizata justificand eficient,a acesteia. (4p.)

Exemplu: daca fis, ierul bac.in cont, ine numerele 4, 5, 9, 13, 6, 11, 10, 14, 2, 1,se va afis,a ın bac.out 13, 14, pozit, iile 4, respectiv 8.

Testul 3

Nicoleta Voica 4, Adrian Voica 5

Limbajul Pascal

SUBIECTUL I (30 de puncte)

Pentru itemul 1, scriet, i pe foaia de examen litera corespunzatoare raspunsuluicorect.

1. Variabilele a, b, c, d s, i x sunt de tip real. Care dintre urmatoarele expresii Pascal auvaloarea 1 daca s, i numai daca x /∈ [a, b] ∪ [c, d ] ∪ -1, 0, 1? (4p.)

a) x>=a and x<=b or x>=c and x<=d or x==-1 or x==0 or x==1b) not(x>=a and x<=b) and not(x>=c and x<=d) and not( x==-1 or x==0 or

x==1)c) x<a and x>b or x<=c and x<=d or (x<>-1 and x<>0 and x<>1)d) (x<a or x>b or x<c or x>d) and (x<>-1 and x<>0 and x<>1)

Scriet, i pe foaia de examen raspunsul pentru fiecare dintre cerint,ele urmatoare.2. Se considera algoritmul de mai jos, reprezentat ın pseudocod.

citeste x (numar intreg)m←0cat timp x>0 executa| d←2| k←0| y←x| cat timp y 6=1 executa| | p←0| | cat timp y%d=0 executa| | | y←[y/d]| | |_ p←1| |_ k←k+p; d←d+1| daca k≥m atunci| |_ m←k; nr←x|_ citeste x (numar intreg)scrie m, nr

4 Profesor, Colegiul Nat, ional ,,Zinca Golescu”, Pites,ti, [email protected] Profesor, Liceul Teoretic

”Ion Barbu”, Pites,ti, [email protected]

PROBLEME DE INFORMATICA PENTRU EXAMENE 127

S-a notat cu [x] partea ıntreaga a numarului real x iar cu x%y restul ımpart, irii numaruluiıntreg x la numarul ıntreg nenul y.

a) Scriet, i ce valori vor fi afis,ate daca se citesc pe rand valorile: 15, 13, 305, 12600,72, 1155, 32, 56, 0. (6p.)

b) Precizat, i doua seturi de date de intrare distincte astfel ıncat sa se afis,eze pentru m,de fiecare data, valoarea 1. (6p.)

c) Scriet, i ın pseudocod un algoritm echivalent cu cel dat care sa utilizeze doar douastructuri repetitive. (4p.)

d) Scriet, i programul Pascal corespunzator algoritmului dat. (10p.)

SUBIECTUL al II-lea (30 de puncte)

1. Fie declararile de mai jos

type bac1 = recorda,b : real;

end;

type bac2 = recordx,y: integer;z: bac1;t: real;

end;

si urmatoarele declaratii:

var y, b: bac1;x, a: bac2;

Care dintre urmatoarele constructii este corecta? (4p.)

a) x.b.z b) b.a.z c) a.z.a d) a.y.b

2. Se considera o structura statica de tip coada s, i o structura statica de tip stiva. Notam cuADS(x) operat, ia prin care se adauga informat, ia x ın stiva, ADC(x) operat, ia prin care seadauga informat, ia x ın coada, ELS operat, ia prin care se elimina un element din stiva s, i seadauga ın coada, ELC operat, ia prin care se elimina un element din coada s, i de adaugaın stiva. Care este cont, inutul stivei s, i cont, inutul cozii ın urma executarii urmatoareloroperat, ii?

ADS(5), ADS(8), ADC(3), ELS, ADC(7), ADS(1), ELC, ELC, ADS(2), ELS, ADC(6)(4p.)

a)S : 5, 1, 7, 8

C : 3, 2, 6b)

S : 5, 1, 3, 8

C : 6, 2, 7c)

S : 5, 1, 3, 8;

C : 7, 2, 6d)

S : 5, 1, 7, 8

C : 6, 2, 3

3. Se considera graful neorientat cu 10 noduri, reprezentat prin urmatoarea lista de muchii:[1, 4], [1, 5], [1, 7], [2, 6], [3, 4], [3, 10], [5, 6], [7, 8], [7, 9].

Sa se verifice daca graful este arbore. In caz afirmativ sa se precizeze lungimea celuimai lung lant, elementar din arbore, iar ın caz contrar sa se precizeze cate muchii trebuieeliminate sau adaugate ın graf astfel ıncat sa devina arbore. (6p.)

4. Stiind ca x este un tablou bidimensional cu 4 linii s, i 4 coloane (numerotate de la 1 la 4)s, i elemente de tip ıntreg sa se precizeze care va fi cont, inutul tabloului dupa executareasecvent,ei de instruct, iuni de mai jos? (6p.)

a=1; b=0;for i:=4 downto 1 do

128 PROBLEME DE INFORMATICA PENTRU EXAMENE

for j:=4 downto i dobegin

x[i][j]:=a+b;c:=x[i][j];x[j][i]:=c;a:=b; b:=c;

end;

5. Scriet, i un program ın Pascal care cites,te un text de maxim 250 de caractere litere alealfabetului englez sau spat, ii s, i determina s, i afis,eaza pe ecran numarul total de cuvinte dintext, iar pe urmatorul rand, separate prin cate un spat, iu, cuvintele din text ın care primuls, i ultimul caracter sunt ambele vocale sau ambele consoane. (10p.)

Exemplu: pentru textul La bacalaureat la proba de Informatica a fost un su-biect cu un sir de caractere se va afis,a: 15 .

SUBIECTUL al III-lea (30 de puncte)

1. Utilizand metoda backtracking, sunt generate ın ordine lexicografica toate anagramelecuvantului ecran. Sa se precizeze care sunt cuvintele generate imediat ınainte s, i dupasecvent,a urmatoare: ernac, ernca, nacer . (4p.)

a) ercan,nacre

b) ercna,narce

c) ercna,nacre

d) ercan,narce

2. Fie urmatorul subprogram:

procedure bac (x,d:integer)begin

if d<=x/d thenbegin

if x div d=0 then write(d,’ ’);bac(x,d+1);if x div d=0 then write(x div d,’ ’);

end;end;

Ce va afis,a subprogramul ın urma apelurilor bac(13, 1) s, i bac(56, 1)? (6p.)3. a) Scriet, i definit, ia completa a subprogramului bac cu doi parametri, care primes,te prin

intermediul primului parametru x un numar natural nenul de minim 2 cifre s, i maxim9 cifre nenule s, i returneaza prin intermediul celui de-al doilea parametru, y, numarulobt, inut din x prin eliminarea primei s, i a ultimei cifre. Subprogramul returneazavaloarea 1 daca numarul x ıncepe s, i se termina cu aceeas, i cifra.

Exemplu: daca x=12341, subprogramul returneaza valoarea 1 deoarece numarul xıncepe s, i se termina cu aceeas, i cifra, iar y=234. (6p.)

b) Scriet, i un program Pascal care cites,te de la tastatura n numere naturale nenulede minim 2 cifre s, i maxim 9 cifre, 1 ≤ n ≤ 100, s, i folosind apeluri utile alesubprogramului de la punctul a) determina s, i afis,eaza pe ecran cate dintre cele nnumere sunt palindrom. In cazul ın care nu exista ın s, ir numere palindrom se vaafis,a un mesaj corespunzator. (4p.)

Exemplu: pentru n=5 s, i numerele 12341, 454, 11, 89, 4567, se va afis,a 2(numerele 454 s, i 11).

PROBLEME DE INFORMATICA PENTRU EXAMENE 129

4. Fis, ierul text bac.in cont, ine cel mult 100000 de numere naturale.

a) Folosind un algoritm eficient din punct de vedere al memoriei utilizate s, i al timpuluide executare scriet, i un program Pascal care cites,te numerele din fis, ier s, i determinaprimul s, i ultimul numar din s, ir, precum s, i pozit, iile acestora, care au cele mai multecifre egale cu 1 ın scrierea lor ın baza 2. Rezultatul se va afis,a ın fis, ierul text bac.out .(6p.)

b) Descriet, i ın limbaj natural metoda utilizata justificand eficient,a acesteia. (4p.)

Exemplu: daca fis, ierul bac.in cont, ine numerele 4, 5, 9, 13, 6, 11, 10, 14, 2, 1,se va afis,a ın bac.out 13, 14, pozit, iile 4, respectiv 8.

Testul 4

Grat,iela Ghiordunescu 6

Limbajul C/C++

SUBIECTUL I (30 de puncte)

Pentru itemul 1, scriet, i pe foaia de examen litera corespunzatoare raspunsuluicorect.

1. Ce afis,eaza expresia 28 / 5 + 28 % 5 / 3? (4p.)

a) 6 b) 6.6 c) 7 d) 6.5

Scriet, i pe foaia de examen raspunsul pentru fiecare dintre cerint,ele urmatoare.2. Se considera algoritmul urmator, descris ın pseudocod. S-a notat cu x%y restul ımpart, irii

numarului ıntreg x la numarul ıntreg nenul y s, i cu [a] partea ıntreaga a numarului real a.

citeste x (numar natural)y←0z←0p←1cat timp x 6=0 executa| daca x%2=0 atunci| | y←y?10+x%10| altfel| | z←p?(x%10)+z| |_ p←p?10;|_ x←[x/10]scrie y," ", z

a) Scriet, i valorile ce se vor afis,a ın urma executarii algoritmului de mai sus daca secites,te x=102035. (6p.)

b) Scriet, i un numar de 3 cifre care poate fi citit pentru variabila x astfel ıncat algoritmulsa afis,eze valorile 0 1. (4p.)

c) Scriet, i ın pseudocod un algoritm, echivalent cu cel dat, ın care sa se ınlocuiascastructura cat timp...executa cu o structura repetitiva de alt tip. (6p.)

6 Profesor, Colegiul Nat, ional ,,Zinca Golescu”, Pites,ti, grati [email protected]

130 PROBLEME DE INFORMATICA PENTRU EXAMENE

d) Scriet, i programul C/C++ corespunzator algoritmului dat. (10p.)

SUBIECTUL al II-lea (30 de puncte)

Pentru fiecare dintre itemii 1 s, i 2 scriet, i pe foaia de examen litera corespunzatoareraspunsului corect.

1. Matricea de adiacent, a a unui graf neorientat cu 8 linii are exact 20 valori nenule s, i sumaelementelor de pe fiecare linie este mai mare sau egala cu 2. Care este valoarea maximape care o poate avea gradul unui nod ıntr-un astfel de graf? (4p.)

a) 7 b) 6 c) 5 d) 4

2. Se considera un arbore cu radacina reprezentat ın memorie cu ajutorul vectorului detat, i T=(2, 3, 0, 3, 3, 2, 6, 6, 4, 9). Stabilit, i care dintre urmatoarele noduri suntextremitat, i finale ale unor lant,uri de lungime para care au ca extremitate init, iala radacinaarborelui. (4p.)

a) 2 2 4 5 b) 10 3 c) 1 6 9 d) 2 4 5

Scriet, i pe foaia de examen raspunsul pentru fiecare dintre cerint,ele urmatoare.3. In declararea de mai jos, campurile x s, i y ale ınregistrarii pot memora coordonatele

carteziene ale unui punct din planul xOy. Scriet, i condit, ia care verifica daca punctele A, Bs, i C formeaza un triunghi echilateral. (6p.)

struct punct float x,y; A,B,C;

4. Scriet, i ce se afis,eaza pe ecran ın urma executarii secvent,ei de program de mai jos, ın carevariabila s memoreaza un s, ir de cel mult 20 caractere, iar variabila i este de tip ıntreg.(6p.)

strcpy(s," bacalaureat ");for (int i=1;i<strlen(s);i++)

if (strchr (" aeiou",s[i]))strcpy(s+i,s+i+1);

cout <<s;

5. Se considera o progresie aritmetica cu primul termen p s, i rat, ia r. Sa se construiasca omatrice patratica cu n linii s, i n coloane care sa cont, ina termenii acestei progresii astfel:

- prima coloana va cont, ine primii n termeni, ın ordine, de sus ın jos- a doua coloana va cont, ine urmatorii n termeni, ın ordine, de sus ın jos s, i as,a mai

departe.

Programul va afis,a pe ecran elementele matricei, cate o linie a matricei pe o linei aecranului, elementele de pe fiecare linie fiind separate prin cate un spat, iu. (10p.)

Exemplu: Pentru p=-2, r=2 s, i n=3 matricea va fi

−2 4 10

0 6 12

2 8 14

PROBLEME DE INFORMATICA PENTRU EXAMENE 131

SUBIECTUL al III-lea (30 de puncte)

Pentru itemul 1, scriet, i pe foaia de examen litera corespunzatoare raspunsuluicorect.

1. Utilizand metoda backtracking se genereaza numere de 3 cifre impare, cifre care apart, inmult, imii 9, 8, 1, 6, 3, 2 Primele 4 solut, ii sunt 999, 991, 993, 919. Care este cea dea 9 - a solut, ie? (4p.)

a) 931 b) 939 c) 933 d) 193

2. Subprogramul sub este definit ın dreapta.Ce se afis,eaza ca urmare a apeluluisub(2024,2)? (6p.)

void sub(int x,int nr) if(x*nr >0)

if(x%4==0)cout <<x<<" ";

sub(x-4,nr -1);

cout <<x<<" ";

3. Scriet, i definit, ia completa a unui subprogram f cu trei parametri: a un numar naturalnenul avand maxim 8 cifre, n un numar natural nenul mai mic ca 10 s, i s care returneazasuma primelor n zecimale ale numarului real a−1. (10p.)

Exemplu: Pentru f(4,3,s), s va avea valoarea 7 deoarece 4−1 = 0.25000 s, i suma primelor3 zecimale este 2+5+0=7.

4. In fis, ierul bac.in se afla cel mult un milion de numere naturale cu maxim 9 cifre separateıntre ele prin cate un spat, iu.

a) Scriet, i ın limbajul C/C++ un algoritm, eficient din punct de vedere al memoriei s, ial timpului de execut, ie, care cites,te din fis, ier datele existente s, i determina s, i afis,eazaın fis, ierul bac.out, numarul cel mai mare din fis, ierul bac.in care este egal cu sumanumerelor vecine. In cazul ın care nu exista astfel de numere se afis,eaza mesajul NUEXISTA. (8p.)

b) Descriet, i succint ın limbaj natural, metoda de rezolvare folosita, explicand ın ceconsta eficient,a ei. (2p.)

Exemplu:

bac.in2 2 70 3 33 30 7 17 10

bac.out33

Testul 5

Catalina Enescu 7

Limbajul C/C++

SUBIECTUL I (30 de puncte)

Pentru itemul 1, scriet, i pe foaia de examen litera corespunzatoare raspunsuluicorect.

1. Variabila x este de tip real. Care dintre urmatoarele expresii C/C++ are valoarea 1 dacas, i numai daca numarul real memorat ın variabila x nu apart, ine intervalului (1,8]? (4p.)

7 Profesor, Liceul Teoretic ,,Ion Cantacuzino”, Pites,ti, [email protected]

132 PROBLEME DE INFORMATICA PENTRU EXAMENE

a) (x>1) && (x<=8)b) (x<=1) && (x>8)

c) (x<=1) || (x>8)d) (x<1) || (x>8)

2. Algoritmul de mai jos este reprezentat ın pseudocod.

citeste x (numar natural)nr←0cat timp x!=0 executa| citeste y (numar natural)| daca x%2 = y%2 atunci| |_ nr←nr + 1|_ x←yscrie nr

a) Ce se va afis,a pentru s, irul de valori 2 4 6 5 7 4 3 0? (6p.)b) Dat, i un exemplu de s, ir de cel put, in doua valori pentru care valoarea afis,ata va fi 0.

(4p.)c) Scriet, i ın pseudocod un algoritm echivalent cu cel dat, ınlocuind adecvat structura

cat timp... executa cu o structura repetitiva de alt tip. (6p.)d) Scriet, i programul C/C++ corespunzator algoritmului dat. (10p.)

SUBIECTUL al II-lea (30 de puncte)

Pentru fiecare dintre itemii 1 s, i 2 scriet, i pe foaia de examen litera corespunzatoareraspunsului corect.

1. Un graf neorientat are 9 noduri. Numarul minim de muchii pe care trebuie sa le aibagraful astfel ıncat sa nu existe noduri izolate este: (4p.)

a) 5 b) 8 c) 9 d) 36

2. Cat, i frat, i are nodul 1 din arborele cu radacina cu 7 noduri, numerotate de la 1 la 7, avandurmatorul vector ,,de tat, i”: (5,1,5,1,0,7,5)? (4p.)

a) 0 b) 1 c) 2 d) 3

Scriet, i pe foaia de examen raspunsul pentru fiecare dintre cerint,ele urmatoare.3. Pentru un elev, ın variabila e se memoreaza urmatoarele informat, ii: nume (un s, ir de

cel mult 20 de caractere), data nas,terii (ziua, luna s, i anul) s, i media anuala. Stiind caexpresiile C/C++ de mai jos au ca valori ziua (un numar natural din intervalul [1,31]),luna (un numar natural din intervalul [1,12]) s, i anul nas,terii (numar natural), respectivmedia anuala (un numar real), scriet, i definit, ia unei structuri cu eticheta elev, care permitememorarea informat, iilor pentru un elev, s, i declarat, i corespunzator variabila e.

e.nume, e.data.zi, e.data.luna, e.data.an, e.medie (6p.)4. Scriet, i un program C/C++ care cites,te de la tastatura un s, ir de cel mult 50 de caractere

(litere mici s, i mari ale alfabetului englez, cifre s, i spat, ii) s, i afis,eaza pe ecran litera marecel mai des ıntalnita ın s, irul citit. Daca exista mai multe litere mari cu numar maximde aparit, ii, programul o va afis,a pe prima dintre ele ın ordine alfabetica. Daca s, irul nucont, ine litere mari, atunci pe ecran se va afis,a mesajul nu.

Exemplu: Daca se cites,te s, irul

MERGEM LA MARE

PROBLEME DE INFORMATICA PENTRU EXAMENE 133

atunci se va afis,a: E (pentru ca literele E s, i M apar de cele mai multe ori ın s, ir s, i E esteprima dintre ele ın ordine alfabetica). (6p.)

5. Scriet, i un program C/C++ care cites,te de la tastatura un numar natural n (n∈[3,50]) s, iconstruies,te ın memorie un tablou bidimensional cu n linii s, i n coloane, astfel ıncat:

a) fiecare element aflat pe diagonala secundara va avea valoarea n;b) elementele aflate strict deasupra diagonalei secundare vor fi primele n(n-1)/2 patrate

perfecte nenule;c) elementele aflate strict sub diagonala secundara vor avea valoarea 1.

Programul afis,eaza pe ecran tabloul obt, inut, cate o linie a tabloului pe cate o linie aecranului, elementele fiecarei linii fiind separate prin cate un spat, iu. (10p)

Exemplu: Daca n=4, se afis,eaza pe ecran

1 4 9 4

16 25 4 1

36 4 1 1

4 1 1 1

SUBIECTUL al III-lea (30 de puncte)

Pentru itemul 1, scriet, i pe foaia de examen litera corespunzatoare raspunsuluicorect.

1. Se considera subprogramul f, declarat ındreapta.Valoarea f(23169) este: (4p.)

a) 21b) 15c) 5d) 8

int f(int n)if (n==0)return 0;

elseif (n%2==0)return n%10+f(n/10);

elsereturn f(n/10);

Scriet, i pe foaia de examen raspunsul pentru fiecare dintre cerint,ele urmatoare.

2. Utilizand metoda backtracking, se genereaza toate posibilitat, ile de a forma echipe diferitede cate 4 persoane din mult, imea Ionut, , Maria, Mihai, Ana, Mihaela, Gabriel,astfel ıncat ın fiecare echipa sa fie exact doi baiet, i s, i doua fete. Primele doua solut, iigenerate sunt, ın aceasta ordine, (Ionut, , Maria, Mihai, Ana), (Ionut, , Maria, Mihai,Mihaela,). Scriet, i, ın ordinea obt, inerii, ultimele doua solut, ii generate. (6p.)

3. Se considera subprogramul perechi, cu cinci parametri: a, b, c, n numere naturaledistincte, v vector cu cel mult 100 de elemente numere naturale distincte. Subprogramulafis,eaza toate perechile (x,y) cu proprietatea ca x s, i y sunt numere diferite din v, careverifica ecuat, ia ax2 + by2 = c. Daca nu exista astfel de valori, se va afis,a Nu.

Exemplu: Daca a=1, b=1, c=25, n=5, v=(3 18 5 0 4), atunci se vor afis,a perechile(3,4) (4,3) (0,5) (5,0), nu neaparat ın aceasta ordine. (10p.)

4. Fis, ierul bac.in cont, ine pe prima linie doua valori naturale, m s, i n (m≤100, n≤100), pea doua linie un s, ir de m numere ıntregi, iar pe a treia linie un s, ir de n numere ıntregi.Fiecare dintre aceste s, iruri sunt ordonate strict crescator, iar elementele lor au cel mult 9cifre fiecare. Numerele de pe acelas, i rand sunt separate prin cate un spat, iu. Se cere sa seafis,eze pe ecran cate dintre elementele celui de al doilea s, ir nu se regasesc s, i ın primul s, ir.

Exemplu: Daca fis, ierul are cont, inutul6 71 2 3 4 7 20

134 PROBLEME DE INFORMATICA PENTRU EXAMENE

3 5 7 8 9 20 24se va afis,a valoarea 4 (numerele care respecta condit, ia sunt 5 8 9 24).

a) Descriet, i ın limbaj natural algoritmul proiectat, justificand eficient,a acestuia. (2p.)b) Scriet, i programul C/C++ corespunzator algoritmului descris. (8p.)

PROBLEME DE INFORMATICA PENTRU EXAMENE 135

Teste pentru admiterea la facultate

Testul 1

Viorel Paun 1

1. Fie mult, imea M = 1, 2, . . . , n s, i A1, A2, . . . , Am submult, imi ale mult, imii M .

Spunem ca doua mult, imi Ai s, i Aj fac parte din aceeas, i familie daca Ai s, i Aj au cel put, in

un element comun. In plus daca Ai s, i Aj sunt din aceeas, i familie s, i Ai s, i Ak sunt dinaceeas, i familie atunci Ai, Aj s, i Ak sunt din aceeas, i familie.

Elaborat, i un program C++ pentru determinarea numarului de familii ce se formeaza s, ia mult, imilor ce formeaza fiecare familie. Pentru implementare consideram n ≤ 1000 s, im ≤ 50.

2. a) Fiind date n numere ıntregi a1, a2, . . . , an, nu ın mod necesar diferite, sa se determineprintr-un program C++ o submult, ime a acestei mult, imi de numere cu proprietateaca suma elementelor sale este divizibila prin n.

b) Evaluat, i complexitatea s, i eficient,a algoritmului utilizat.

Testul 2

Viorel Paun 1

1. Fie mult, imea M = 1, 2, . . . , n s, i A1, A2, . . . , Am submult, imi ale mult, imii M .

Elaborat, i un program C++ pentru a determina un numar minim de mult, imi

Ai1 , Ai2 , . . . , Aik ∈ A1, A2, . . . , Am

astfel ıncatAi1 ∪ Ai2 ∪ . . . ∪ Aik = A1 ∪ A2 ∪ . . . ∪ Am.

Pentru implementare consideram n ≤ 1000 s, i m ≤ 50.2. a) Elaborat, i un program C++ pentru calculul integralei

π2∫

0

sinn x cosm xdx

utilizand formulele de recurent, a dependente de n s, i m.b) Elaborat, i un program C++ pentru calculul aproximativ al integralei

1∫0

e−x2

dx.

Indicat, ie. Se pot utiliza, de exemplu, sumele Riemann sau Darboux.

1 Lect. univ. dr., Universitatea din Pites,ti, [email protected]

136 PROBLEME DE INFORMATICA PENTRU EXAMENE

Teste grila pentru admiterea la facultate

Testul 1

Cristina Tudose 1

1. Se considera urmatoarea secvent, a de cod:

int a = 15, b = 023, c = 0x52;int x = a - b + c;cout <<x;

Care dintre urmatoarele variante este rezultatul afis,at dupa executarea acestei secvent,e decod?

a) 15 b) 78 c) 116 d) 44

2. In urma secvent,ei de cod

int a = 5, *p, b = 2;p = &a;*p *= a;p = &b;b *= a;cout <<a<<" "<<b<<" "<<*p;

se tipares,te:

a) 5 2 2 b) 5 25 50 c) 25 50 50 d) 25 2 50

3. Ce se va afis,a ın urma executarii programului urmator?

int f1(int );int f2(int );int f1(int n)

if(n==0) return 1;return f1(n -1)+ f2(n -1);

int f2(int n)

if(n==0) return 2;return 2+f1(n-1)-f2(n -1);

int main ()

cout <<f1(4)<<" "<<f2 (4);return 0;

a) 3 1 b) 2 7 c) 10 10 d) 10 8

4. Fie funct, ia:

void f(int &a, int b)int t;t=a;a=b;b=t;

1 Lect. univ. dr., Universitatea din Pites,ti, [email protected]

PROBLEME DE INFORMATICA PENTRU EXAMENE 137

Se considera doua variabile ıntregi a = 200, b = 300. Ce valoare va avea suma a+ b dupaapelul f(a, b)?

a) 500 b) 501 c) 600 d) 401

5. Fie secvent,a de cod

int i, j, s=0;for(i=0;i<n;i++)

for(j=0;j<n;j++)if(a[i][j] % 2) continue ;s+=a[i][j];

unde a este o matrice patratica de numere ıntregi, avand dimensiunea n, cu liniile s, icoloanele numerotate de la 0. Ce se va salva ın variabila s dupa executarea secvent,ei?

a) Suma elementelor pare.c) Suma elementelor impare.

b) Primul element par.d) Primul element impar.

6. Se considera urmatoarea secvent, a de cod:

int **a;int x[3]=2 ,7 ,3;int y[3]=33 ,44 ,22;a = new int *[2];a[0]=y;a[1]=x;x [1]=55;int s=0;for(int i=0;i <2;i++)

s+=a[i][i];cout <<s;

Care va fi valoarea variabilei s dupa executarea acesteia?

a) 88 b) 40 c) 46 d) 50

7. Fie v un vector sortat cu n elemente ıntregi, pe pozit, ii numerotate de la 1 la n. Secvent,ade cod de mai jos realizeaza cautarea binara a numarului x ın vectorul v.

int p=1, u=n, poz =-1;while(p<=u)

int m=(p+u)/2;if(v[m]==x)

poz=m;break;

if(x<v[m])

u=m -1;else

p=m+1;cout <<poz;

Daca v = (2, 4, 7, 10, 25, 36, 45, 78, 89, 92, 95, 98, 100), n = 13 s, i x = 2, numarul deınjumatat, iri ale intervalului este:

138 PROBLEME DE INFORMATICA PENTRU EXAMENE

a) 1 b) 2 c) 3 d) 0

8. Se considera o stiva si o coada init, ial vide. Se introduc pe rand ın coada primele s,asenumere prime, ın ordine descrescatoare. Se extrag apoi din coada trei elemente s, i seadauga ın stiva, ın ordinea ın care au fost extrase. Care este elementul din varful stiveidupa executarea acestor operat, ii?

a) 5 b) 13 c) 7 d) 2

9. Fie v un vector cu n elemente ıntregi, pe pozit, ii numerotate de la 1 la n s, i secvent,a decod de mai jos care realizeaza ordonarea crescatoare a elementelor acestuia.

for(i=1;i<n;i++)for(j=i+1;j<=n;j++)if(v[i]>v[j])

aux=v[i]; v[i]=v[j]; v[j]= aux;

Numarul total de comparat, ii ıntre elementele vectorului este:

a) n(n+ 1)/2c) (n− 1)(n− 2)/2

b) n(n− 1)/2d) n/2

10. Ce se afis,eaza ca urmare a executarii secvent,ei de cod urmatoare?

char a [12]=" INFORMATICA ";for(int i=2; i <12; i++)

a[i -2]=a[i];a[0]= ’D’;a[2]= ’G’;cout <<a;

a) IORMATICAc) DOGMATICA

b) CADOGMATICAd) eroare

11. Ce calculeaza urmatoarea funct, ie?

int functie (int a, int b)while(a!=b)

if(a>b)a-=b;

elseb-=a;

return b;

a) Cel mai mic multiplu comun al lui a s, i b.c) Modulul diferent,ei a− b.

b) Cel mai mare divizor comun al lui a s, i b.d) Cel mai mare numar dintre a s, i b.

12. Se foloses,te metoda backtracking pentru a genera submult, imile nevide ale mult, imii11, 12, 13, 14, 15 care cont, in numai elemente impare. Cate solut, ii s-au generat?

a) 6 b) 7 c) 8 d) 9

13. Numarul maxim de muchii ale unui graf neorientat ce are 10 noduri s, i doua componenteconexe este egal cu:

a) 20 b) 100 c) 45 d) 36

14. Daca G este un graf neorientat cu 10 noduri s, i 10 muchii atunci:

PROBLEME DE INFORMATICA PENTRU EXAMENE 139

a) G este conexc) G cont, ine cicluri

b) G este un arbored) toate gradele nodurilor sunt egale cu 1.

15. Urmatoarele numere nu pot fi gradele nodurilor unui graf cu 6 noduri:

a) 1, 2, 2, 3, 4, 4c) 1, 2, 2, 3, 3, 5

b) 1, 1, 2, 4, 4, 4d) 1, 1, 3, 3, 4, 4.

Testul 2

Cristina Tudose 2

1. Ce se va tipari dupa executarea urmatoarei secvent,e de cod?

float x=2.25e+02;int t=2*x;cout <<t;

a) 450 b) 4 c) 4.5 d) 5

2. In urma executarii secvent,ei

int a = 5, *p=&a;a*=*p;cout <<a<<" "<<*p;

se va afis,a:

a) 5 5 b) 25 25 c) 25 5 d) 5 25

3. Fie doi vectori A s, i B ordonat, i crescator, cu m respectiv n elemente, pe pozit, ii numerotatede la 1. Prin algoritmul de interclasare, prezentat mai jos, se obt, ine vectorul ordonat Ccare cont, ine toate elementele din A s, i B.

i=1; j=1; k=1;while (i<=m && j<=n)

if (A[i]<=B[j])

C[k]=A[i]; i++;else

C[k]=B[j]; j++;k++;

while (i<=m)

C[k]=A[i]; i++; k++;while (j<=n)

C[k]=B[j]; j++; k++;

2 Lect. univ. dr., Universitatea din Pites,ti, [email protected]

140 PROBLEME DE INFORMATICA PENTRU EXAMENE

Care este complexitatea algoritmului?

a) O(m+ n)c) O(log2(m · n))

b) O(log2(m+ n))d) O(n · log2(n))

4. Care va fi valoarea salvata ın variabila s dupa executarea codului urmator?

int **a,i,j,s;a = new int *[3];for(int i=0;i <3;i++)

a[i] = new int [3];for(j=0;j <3;j++)

a[i][j]=2*i+j+2;

s=0;for(int i=1;i <3;i++)

for(int j=1;j <3;j++)s+=a[i][j];

a) 38 b) 40 c) 26 d) 30

5. Ce se va tipari dupa executarea urmatoarei secvent,e?

int i, j;for(i=1;i <=15;i++)

for(j=1;j <=15;j++)if(i>j) break;

cout <<i+j;

a) 16 b) 3 c) 15 d) 17

6. Ce valoare va avea variabila intreaga y dupa executarea secvent,ei de cod urmatoare?

int x=30, y=40;if ((x<y ? y : x) > 30)

y+=x;else

y-=x;

a) 10 b) 70 c) 40 d) 50

7. Fie funct, ia:

int f(int n)if(n==0) return -1;return n*f(n -1)+2;

Ce valoare va avea variabila intreaga x ın urma apelului x = f(4)?

a) -1 b) 1 c) 58 d) 5

PROBLEME DE INFORMATICA PENTRU EXAMENE 141

8. Se foloses,te metoda backtracking pentru a genera toate numerele de patru cifre care ausuma cifrelor egala cu 12. Primele cinci solut, ii generate sunt: 1029, 1038, 1047, 1056, 1065.A cata solut, ie este 1137?

a) a zeceac) a unsprezecea

b) a douasprezecead) nu este solut, ie

9. Ce se va afis,a ca urmare a executarii urmatoarei secvent,e?

int i=5, t=8;switch (i%4)

case 0: t -=3;case 1: t/=4;case 2: t+=5; break;case 3: t*=6;

i+=--t;cout <<t--;t-=i++;

a) 5 b) 2 c) 6 d) 7

10. Se considera o matrice patratica de dimensiune n avand liniile s, i coloanele numerotate dela 0 la n− 1. Condit, ia ca elementul de pe linia i s, i coloana j sa fie situat sub diagonalasecundara este:

a) i+ j > n− 1b) i+ j > n

c) i+ j < n− 1d) i+ j < n

11. Ce se va afis,a ın urma executarii programului urmator?

void func(int a[])for(int i=1;i <4;i++)

a[i]++;int main ()int x[5]=1 ,1 ,0 ,5 ,3;func(x);cout <<x[1]+x[4];return 0;

a) 3 b) 4 c) 7 d) 5

12. In urma executarii secvent,ei

unsigned int t=13, i = 7;t = (t >>2) + (iˆ5);cout <<t;

se tipares,te:

a) 10 b) 5 c) 20 d) 30

142 PROBLEME DE INFORMATICA PENTRU EXAMENE

13. Inalt, imea minima a unui arbore binar strict (orice nod are 0 sau 2 descendent, i) cu 2019noduri este egala cu:

a) 11 b) 8 c) 9 d) 10

14. Numarul maxim de cicluri ale unui graf neorientat ce are 2018 noduri s, i toate gradelenodurilor din mult, imea 0, 1, 2 este:

a) 672 b) 504 c) 1009 d) 673

15. Numarul minim de componente conexe ale unui graf neorientat ce are 10 noduri s, i 6muchii este egal cu:

a) 2 b) 3 c) 4 d) 5

PROBLEME DE MATEMATICA PENTRU CONCURSURI 143

PROBLEME DE MATEMATICA PENTRU

CONCURSURI

Probleme propuse

Clasa a V-a

M 1. Aratat, i ca oricare ar fi k numar natural, k ≥ 4, exista numere de forma a1a2 . . . ak astfel

ıncat suma resturilor ımpart, irii lor la a1, a2, . . . , ak−1, respectiv ak sa fie 8k − 1.

Marian Haiducu, Pites,ti

M 2. Un ogar urmares,te o vulpe care are 12 sarituri avans. Cate sarituri va face ogarul pana sa

ajunga vulpea s,tiind ca, ın timp ce ogarul face 7 sarituri, vulpea face 8 sarituri s, i ca distant,a

parcursa de ogar ın 5 sarituri este aceeas, i cu cea parcursa de vulpe ın 6 sarituri?

Marius Antonescu, Cos,es,ti, Arges,

M 3. Figura 1 este formata din 4 patrate ın care sunt scrise numerele naturale a, b, c, d. Printr-o

operat,ie ınt,elegem ınlocuirea numerelor din doua patrate care au o latura comuna cu succesorii

acestora. Dupa mai multe operat, ii se obt, ine Figura 2. Stiind ca a + b + c + d = 43, aflat, i

numarul b.

Mugurel Simion, Stefanes,ti, Arges,

M 4. Fie x, y, z numere naturale astfel ıncat x are ultima cifra 5, y este un numar prim, y ≥ 5,

iar restul ımpart, irii lui z la 15 este egal cu 7. Daca N = 3x+y+ 2z, determinati valorile posibile

ale restului obtinut ımpartindu-l pe N la 6.

Andreea Oprea s, i Adrian Turcanu, Pites,ti

M 5. Sa se determine cifra x astfel ıncat 19x74... x3.

Iulia Cucu s, i Adrian Turcanu, Pites,ti

Clasa a VI-a

M 6. Pe o tabla sunt scrise numerele de la 1 la 2n− 1, n ≥ 4. Se s,terg, la ıntamplare, doua

numere de pe tabla s, i se scrie suma lor. Este posibil ca, repetand procedeul, sa obt, inem n− 1

numere distincte divizibile cu 5? Justificat, i raspunsul.

Marian Haiducu, Pites,ti

144 PROBLEME DE MATEMATICA PENTRU CONCURSURI

M 7. a) Cate fract, ii cu numaratorul 2018 sunt cuprinse ıntre1

11s, i

1

10.

b) Cate fract, ii cu numaratorul cel mult n sunt cuprinse ıntre1

n+ 1s, i

1

n, n ∈ N??

Radu Deaconescu, Pites,ti

M 8. Produsul divizorilor naturali ai numarului natural n este 2103240. Scriind divizorii

numarului n ın ordine crescatoare d1 = 1 < d2 < d3 < ... < dk = n, aratat, i ca d1289 are cel put, in

11 cifre.

Florin Antohe, Galat, i

M 9. In triunghiul ABC bisectoarea unghiului A s, i mediatoarea laturii AB se intersecteaza

ıntr-un punct M ∈ BC. Sa se demonstreze ca masura unghiului dintre ele este mai mare de 30.

Marius Antonescu, Cos,es,ti, Arges,

M 10. Fie p s, i q numere prime s, i S = p+ q + p2 + q2 + pq + p2q + pq2 + p2q2.

a) Aratat, i ca S este un numar compus.

b) Daca p+ q este divizibil cu 3, atunci aratat, i ca S nu este patrat perfect.

Mugurel Simion, Stefanes,ti, Arges,

Clasa a VII-a

M 11. Fie a, b, c numere naturale astfel ıncat n =Ä7a − 2b

ä Ä7b − 2c

ä(7c − 2a) este natural.

Aratat, i ca n nu este patrat perfect.

Marian Haiducu, Pites,ti

M 12. Fie ∆ABC cu m(^A) =m(^B) +m(^C)

2. Daca BD, respectiv CE sunt ınalt, imi ale

triunghiului ABC, atunci aratat, i ca DE =BC

2.

Corina Roxana Marcus,anu, Pites,ti

M 13. Daca a, b, c, d sunt numerele reale pozitive demonstrat, i ca:

a4

b2c+

b4

c2d+

c4

d2a+

d4

a2c≥ a+ b+ c+ d. Cand are loc egalitatea?

Dorin Marghidanu, Corabia

M 14. In triunghiul ABC, mediatoarea laturii [BC] s, i bisectoarea unghiului A sunt concurente

ın punctul Q situat pe linia mijlocie [MN ] a triunghiului ABC, M ∈ (AB), N ∈ (AC). Aratat, i

ca triunghiurile MQB s, i NQC au arii diferite.

Mihai Burdus,a, Pites,ti

M 15. a) Fie S1 =1

1 · 2 · 3 · 4+

1

2 · 3 · 4 · 5+

1

3 · 4 · 5 · 6+ . . .+

1

101 · 102 · 103 · 104. Aratat, i ca

S1 <1

18.

b) Fie S2 = 1 · 2 · 3 · 4 + 2 · 3 · 4 · 5 + 3 · 4 · 5 · 6 + . . .+ 101 · 102 · 103 · 104. Aratat, i ca S2 nu

este patrat perfect.

Radu Deaconescu, Pites,ti

PROBLEME DE MATEMATICA PENTRU CONCURSURI 145

Clasa a VIII-a

M 16. Fie a, b ∈ R?. Sa se determine funct, ia f : R∗ → R, pentru care

a

b·Åf(x)− ax

b

ã≤ 1

x≤ b

a·Çf

Ç1

x

å− bx

a

å,

oricare ar fi x ∈ R∗.

Dorin Marghidanu, Corabia

M 17. Fie a, b, c numere reale pozitive cu abc = 1. Aratat, i ca

a(b+ c)4 + b(a+ c)4 + c(a+ b)4 ≥ 12(ab+ ac+ bc) + 4(a2 + b2 + c2) ≥ 16

Ç1

a+

1

b+

1

c

å.

Mihai Florea Dumitrescu, Potcoava

M 18. Fie ABCD tetraedru ın care m(^BAC)=m(^CBD)=90. Daca E este mijlocul

segmentului [CD], demonstrat, i inegalitatea√

3 ·CD > PABE, unde PABE reprezinta perimetrul

triunghiului ABE.

Florin Stanescu, Gaes,ti

M 19. Fie funct, ia f : (0,∞)→ R, definita prin f(x) = ”partea ıntreaga a volumului piramidei

triunghiulare regulate cu latura bazei de lungime x s, i ınalt, imea de lungime√

3”.

Sa se calculeze partea ıntreaga a numarului

A =1

f(2)+

1

f(3)+

1

f(4)+ . . .+

1

f(2018)

Mihai Burdus,a s, i Adrian Turcanu, Pites,ti

M 20. Fie V A1A2 . . . An piramida cu baza poligonul A1A2 . . . An, n ∈ N, n ≥ 3, situat ıntr-un

plan α. Pentru p ∈ 2, 3, . . . , n − 1 arbitrar fixat, fie G1 centrul de greutate al triunghiului

V Ap−1Ap s, i G2 centrul de greutate al triunghiului V ApAp+1. Fie P ∈ (V Ap−1) s, i R ∈ (V Ap+1).

Aratat, i ca PR‖α daca si numai daca PG1, V Ap si RG2 sunt concurente.

Marian Haiducu, Pites,ti

146 PROBLEME DE MATEMATICA PENTRU CONCURSURI

Clasa a IX-a

M 21. Demonstrat, i ca pentru orice n ∈ N∗ s, i x ≥ 0 are loc inegalitatea

n∑k=1

(2 + x)k

2 + kx≥ 2n − 1.

Cand are loc egalitatea?

Dorin Marghidanu, Corabia

M 22. Demonstrat, i identitatea[2n−1∑k=n

»k(k + 1)

]=

ñn− 1

2

ô, ∀ n ∈ N∗,

unde [x] s, i x reprezinta partea ıntreaga, respectiv partea fract, ionara a numarului real x.

Nicolae Staniloiu, Bocs,a

M 23. Rezolvat, i ın (0,∞)6 sistemula1 + a2 + a3 − a4 − a5 − a6 = 3

a21 + a22 + a23 + a24 + a25 + a26 = 9

a1a2a3a4a5a6 = 1

.

Leonard Giugiuc s, i Diana Trailescu, Drobeta Turnu Severin

M 24. Rezolvat, i ın mult, imea numerelor reale ecuat, iile:

a) (4 cos2 x− 3)(4 cos2 3x− 3) = 2 sinx;

b)cos3 x+ cos3 5x+ cos3 9x+ cos3 13x

cosx+ cos 5x+ cos 9x+ cos 13x=

3

4.

Marin Chirciu, Pites,ti

M 25. Fie ABC un triunghi avand toate unghiurile mai mici decat 2π3

s, i fie T punctul Torricelli-

Fermat al acestuia. Bisectoarele unghiurilor ^BTC, ^CTA s, i ^ATB intersecteaza laturile [BC],

[CA] s, i [AB] ın punctele D, E s, i respectiv F . Aratat, i ca AB+BC+CA ≥ 2(DE+EF +FD).

Leonard Giugiuc, Romania, Kadir Altintas, Turcia

s, i Miguel Ochoa Sanchez, Peru

PROBLEME DE MATEMATICA PENTRU CONCURSURI 147

Clasa a X-a

M 26. a) Determinat, i funct, ia strict crescatoare f : R→ R care satisface condit, ia

2f(x) + f(f(x)) = 3x, ∀x ∈ R.

b) Rezolvat, i ın mult, imea numerelor reale ecuat, ia

3Äxlog5 6 − 1

ä= 2x+ (x+ 1)log6 5.

Marin Chirciu, Pites,ti

M 27. Rezolvat, i ın R× R sistemul x− 3√y + 2x = y − 3

√x+ 2y

22x − (9− y) · 2x + 8 + 6x− 2y2 = 0.

Sorin Ulmeanu, Pites,ti

M 28. Fie rn =

√n

(n− 1)

…(n− 2) . . .

√3»

2√

1, unde n ∈ N∗. Aratat, i ca:

a) rn ≤Ç

(n− 1)2n + 1

2n − 1

å1− 12n

.

b)n

rn∈ [1,

√n ].

Dorin Marghidanu, Corabia

M 29. Fie ABC un triunghi cu AB 6= AC s, i fie D mijlocul laturii BC. In exteriorul triun-

ghiului ABC se construiesc triunghiurile BAM s, i CAN astfel ıncat AM = AB, AN = AC s, i

m(^BAM) = m(^CAN) = α. Demonstrat, i ca AD ⊥MN daca s, i numai daca α = 90.

Nicolae Staniloiu, Bocs,a

M 30. Fie ABC un triunghi nedreptunghic, O centrul cercului circumscris acestuia, iar D, E

s, i F mijloacele laturilor [BC], [AC] s, i respectiv [AB]. Fie DO ∩ AC = X, EO ∩ AB = Y s, i FO ∩BC = Z. Aratat, i ca

AX

XC· BYY A· CZZB

=

∣∣∣∣∣(a2 − b2)(b2 − c2)(c2 − a2)a2b2c2

∣∣∣∣∣(notat, iile fiind cele obis,nuite).

Van Khea, Cambodgia s, i Leonard Giugiuc, Romania

148 PROBLEME DE MATEMATICA PENTRU CONCURSURI

Clasa a XI-a

M 31. Fie A,B ∈Mn(C) astfel ıncat A = 2AB −BA. Demonstrat, i ca

det(AB −BA) = 0.

(In legatura cu problema XI.459, R.M.T. nr. 2/2017, punctul a).)

Daniel Jinga, Pites,ti

M 32. Aratat, i ca pentru orice matrice A,B ∈M2(R) au loc urmatoarele inegalitat, i:

a) det(A2 +B2 −BA) ≥ det(AB −BA);

b) det ((A−B)(A+B)− 2(A2 +B2)) ≥ 4 det(AB −BA).

Florin Stanescu, Gaes,ti

M 33. Rezolvat, i ın mult, imea numerelor reale ecuat, ia

1 + 3 · 23x−2 · 15x−1 = 83x−2 − 153x−3.

Marin Chirciu, Pites,ti

M 34. a) Fie f : [a, b]→ R o funct, ie derivabila, a, b ∈ R, a < b. Aratat, i ca exista c1, c2 ∈ (a, b)

astfel ıncat f ′(c1) =f(c1)− f(a)

b− c1s, i f ′(c2) =

f(b)− f(c2)

c2 − a.

b) Demonstrat, i ca pentru orice a, b ∈ R cu a < b exista o infinitate de funct, ii f derivabile

pe [a, b] pentru care valorile c1 s, i c2 definite la punctul a) sunt unice s, i egale.

Dorin Marghidanu, Corabia

M 35. Fie a, b, c, d ∈ [−1,∞) astfel ıncat a+ b+ c+ d = 0. Aratat, i ca

a2 + b2 + c2 + d2 + 5(abc+ abd+ acd+ bcd) ≥ 4abcd.

Cand are loc egalitatea?

Leonard Giugiuc s, i Diana Trailescu, Drobeta Turnu Severin

PROBLEME DE MATEMATICA PENTRU CONCURSURI 149

Clasa a XII-a

M 36. Pentru orice grup G s, i orice numar natural nenul n notam

Sn(G) = a ∈ G | axna = x, ∀x ∈ G.

a) Pentru n ≥ 2, demonstrat, i ca daca Sn(G) 6= Ø, atunci exista m ∈ N∗ astfel ıncat

xm = e, ∀x ∈ G (e reprezinta elementul neutru al lui G). Ramane adevarata afirmat, ia pentru

n = 1?

b) Demonstrat, i echivalent,a: exista un grup G astfel ıncat Sn(G) \ e 6= Ø daca s, i numai

daca n este impar.

Marin Chirciu, Pites,ti

M 37. Fie s un numar real apart, inand intervalului (12, 18) s, i ABC un triunghi astfel ıncat

a + b + c = 6, a2 + b2 + c2 = s s, i R + r =√

3 (a = BC, b = AC, c = AB, iar R s, i r sunt

raza cercului circumscris, respectiv raza cercului ınscris triunghiului ABC). Exprimat, i aria

triunghiului ABC ın funct, ie de s.

Leonard Giugiuc, Cristinel Mortici, Romania s, i Kadir Altintas, Turcia

M 38. Fie a, b ∈ R, a < b. Calculat, i integrala I =∫ b

a

1

(x− a)4 + (x− b)4dx.

Daniel Jinga, Pites,ti

M 39. Determinat, i funct, iile continue f, g :î0, π

2

ó→ R care verifica simultan relat, iile

f 3(x)− 3f(x)g2(x) = cos x, g3(x)− 3f 2(x)g(x) = − sinx, ∀x ∈ï0,π

2

òs, i pentru care aria suprafet,ei plane cuprinse ıntre graficele lor este:

a) maxima; b) minima.

Dorin Marghidanu, Corabia

M 40. Fie f : [0, 1] → R o funct, ie derivabila astfel ıncat f(0) = 0, f ′(0) = 1, 0 < f ′(x) < 1

pentru orice x ∈ (0, 1] s, i exista f ′′(0) ∈ R∗. Fie s, irul (xn)n≥0 astfel ıncat x0 ∈ (0, 1] s, i

xn+1 =xn + f(xn)

2, pentru orice n ≥ 0. Demonstrat, i ca 0 < xn+1 < xn ≤ 1 pentru orice n ∈ N

s, i aratat, i ca

limn→∞

Çn3∫ xn

xn+1

f(x) dx

å=

Ç4

f ′′(0)

å2

.

Florin Stanescu, Gaes,ti

150 PROBLEME DE INFORMATICA PENTRU CONCURSURI

PROBLEME DE INFORMATICA PENTRU

CONCURSURI

Probleme propuse

Clasa a IX-a

I 1 (becuatii). Tica s, i Ionica sunt buni prieteni, pasionat, i de doua lucruri: matematica s, i

informatica. Aces,tia participa la un concurs de propunatori de probleme pe echipe. Pentru a

avea succes Tica vrea sa propuna o problema cu ecuat, ii, iar Ionica o problema cu mult, imi. Dupa

consultari repetate se decid sa participe la concurs cu o problema care foloses,te atat ecuat, ii cat s, i

mult, imi. In aceasta problema se dau N ecuat, ii de forma x2 = Ai + y2, 1 ≤ i ≤ N , s, i se noteaza

cu Si mult, imea solut, iilor celei de-a i-a ecuat, ie, Si = (x, y)|x2 = Ai + y2, x, y numere naturale.Cerint,a problemei este determinarea a doua numere B = card(S1) + card(S2) + . . .+ card(Sn),

respectiv C = card (S1 ∪ S2 ∪ . . . ∪ Sn).

Cerint, a

Se cunosc N s, i numerele A1, A2, . . . , AN s, i se cere sa se determine numerele B s, i C.

Date de intrare

Din fis, ierul becuatii.in se citesc: numarul natural p de pe prima linie (ce poate avea doua

valori: 1 sau 2, corespunzatoare uneia din cele doua cerint,e, adica 1 pentru determinarea lui B,

iar 2 pentru determinarea lui C), numarul N de pe a doua linie s, i de pe a treia linie numerele

A1, A2, . . . , AN cu cate un spat, iu ıntre ele.

Date de ies, ire

In fis, ierul becuatii.out se va scrie numarul B, daca p = 1 s, i respectiv numarul C, daca p = 2,

cu semnificat, ia din enunt, .

Restrict, ii s, i precizari

• 1 < N ≤ 10000, numar natural

• Ai sunt numere naturale nenule mai mici strict decat 1000, 1 ≤ i ≤ N

• Pentru o mult, ime M , card(M) reprezinta numarul de elemente al mult, imii M

• Pentru mult, imile M s, i P , notam cu M ∪ P reuniunea dintre M s, i P

Exemple

becuatii.in becuatii.out Explicat, ie

1

2

1 21

3 p = 1 s, i deci afis, am B. Prima ecuat, ie

este x2 = 1 + y2, iar a doua ecuat, ie este

x2 = 21 + y2. Mult, imile solut, iilor sunt

S1 = (1, 0) s, i S2 = (11, 10), (5, 2). In

acest caz B = 1 + 2 = 3.

PROBLEME DE INFORMATICA PENTRU CONCURSURI 151

becuatii.in becuatii.out Explicat, ie

2

2

1 21

3 p = 2 s, i deci afis, am C. Prima ecuat, ie

este x2 = 1 + y2, iar a doua ecuat, ie este

x2 = 21 + y2. Mult, imile solut, iilor sunt

S1 = (1, 0) s, i S2 = (11, 10), (5, 2), deci

S1 ∪ S2 = (1, 0), (11, 10), (5, 2), deci C =

card(S1 ∪ S2) = 3.

Timp maxim de execut, ie: 0.04 secunde/test.

Memorie totala disponibila 4 MB.

Doru Anastasiu Popescu, Pites,ti (Micul Gates, 2017)

I 2 (castel). In pivnit,a unui castel se afla un cufar ce cont, ine obiecte de valoare foarte mare.

Toate us, ile pana la pivnit, a se deschid daca se introduce un cod numeric format din cifre zecimale,

cod care este scris pe us, a pentru a facilita deplasarea mai us,oara prin castel. Pentru a obt, ine

aceste obiecte trebuie sa se treaca de doua obstacole:

1. us,a de la intrarea ın pivnit, a, care se deschide doar daca se introduce un cod format din

cifre zecimale, notat cu codP ;

2. capacul cufarului, care se deschide daca se cunoas,te un cod format din cifre zecimale,

notat cu codC.

Codurile de deschidere a celor N us, i ale castelului, prin care se trece pentru a ajunge ın fat,a

pivnit,ei, se cunosc: x1, x2, . . . , xN , numere naturale.

Codul codP , necesar deschiderii pivnit,ei, se obt, ine determinand numarul de zerouri de la

sfars, itul numarului Max!, unde Max este cel mai mare dintre codurile celor N us, i, x1, x2, . . . , xN .

Codul codC, necesar deschiderii cufarului, se obt, ine determinand numarul de divizori ai

produsului factorialelor codurilor de pe cele N us, i ale castelului: x1! · x2! · . . . · xN !.

Cerint, a

Cunoscand numarul de us, i, N , s, i codurile lor x1, x2, . . . , xN , se cere:

1. codul codP de deschidere a us, ii pivnit,ei;

2. codul codC de deschidere a cufarului.

Date de intrare

Fis, ierul de intrare castel.in cont, ine pe prima linie un numar natural p. Pentru toate testele

de intrare, numarul p poate avea doar valoarea 1 sau 2.

Pe linia a doua se afla N , iar pe linia a treia se afla numerele x1, x2, . . . , xN , separate prin

cate un spat, iu, reprezentand codurile de deschidere a celor N us, i din castel.

Date de ies, ire

Daca valoarea lui p este 1, se va rezolva numai punctul 1) din cerint, a.

In acest caz, ın fis, ierul de ies, ire castel.out se va scrie un singur numar natural reprezentand

codul codP de deschidere a us, ii pivnit,ei.

152 PROBLEME DE INFORMATICA PENTRU CONCURSURI

Daca valoarea lui p este 2, se va rezolva numai punctul 2) din cerint, a.

In acest caz, ın fis, ierul de ies, ire castel.out se va scrie un singur numar natural, reprezentand

codul de deschidere a cufarului modulo 9973, adica numarul codC % 9973.

Restrict, ii s, i precizari

• 1 ≤ N ≤ 10000

• 1 ≤ x1, x2, . . . , xN ≤ 10000

• a%b reprezinta restul ımpart, irii lui a la b

• k! este notat, ia pentru produsul 1 · 2 · 3 · . . . · k• Pentru determinarea corecta a codului pivnit,ei, codP , se acorda 30% din punctaj

Exemple

castel.in castel.out Explicat, ie

1

5

3 6 4 6 2

1 p = 1

codP este numarul de zerouri ın care se

termina numarul 6! = 720, adica 1.

castel.in castel.out Explicat, ie

2

5

3 6 4 6 2

294 p = 2

codC este numarul de divizori ai produsului

3! · 6! · 4! · 6! · 2!, adica 294.

Timp maxim de execut, ie: 1 secunda/test.

Memorie totala disponibila 4 MB, din care 2 MB pentru stiva.

Violeta Balan, Slatina

I 3 (relief). Vladino a ınceput sa studieze foarte mult formele de relief. Ca sa nu lase informatica

pe planul doi, profesorul sau ıi propune sa rezolve o problema cu trei forme de relief: munt, i,

dealuri s, i vai, pentru un s, ir cu n numere naturale x1, x2, . . . , xn.

Muntele se defines,te ca fiind o secvent, a maximala de numere din s, irul dat de forma

xi, . . . , xj, . . . , xk, unde xi < xi+1 < . . . . < xj, xj > xj+1 > . . . > xk, i < j < k.

Dealul se defines,te ca fiind o secvent, a maximala de numere din s, irul dat de forma

xi, . . . , xj, . . . , xk, . . . , xh, unde xi < xi+1 < . . . < xj, xj = xj+1 = . . . = xk, xk > xk+1 >

. . . > xh, i < j < k < h.

Valea se defines,te ca fiind o secvent, a maximala de numere din s, irul dat de forma xi, . . . , xj ,

unde xi = xi+1 = . . . = xj, i < j.

Cerint, a

Cunoscand numarul n s, i cele n numere x1, x2, . . . , xn, se cere:

1. de cate ori apare cel mai mic numar ın s, irul x;

2. numerele de munt, i, dealuri s, i vai.

PROBLEME DE INFORMATICA PENTRU CONCURSURI 153

Date de intrare

Fis, ierul de intrare relief.in cont, ine pe prima linie un numar natural p. Pentru toate testele

de intrare, numarul p poate avea doar valoarea 1 sau 2.

Pe linia a doua se afla n, iar pe urmatoarea linie numerele x1, x2, . . . , xn separate ıntre ele

prin cate un spat, iu.

Date de ies, ire

Daca valoarea lui p este 1, se va rezolva numai punctul 1) din cerint, a.

In acest caz, ın fis, ierul de ies, ire relief.out se va scrie un singur numar natural reprezentand

de cate ori apare cel mai mic numar ın s, irul x.

Daca valoarea lui p este 2, se va rezolva numai punctul 2) din cerint, a.

In acest caz, ın fis, ierul de ies, ire relief.out se vor scrie trei numere separate prin cate un

spat, iu, reprezentand numerele de munt, i, dealuri s, i vai.

Restrict, ii s, i precizari

• 1 ≤ n ≤ 100000

• Numerele x1, x2, . . . , xn sunt ≤ 1010

• Pentru rezolvarea corecta a cerint,ei 1 se acorda 20% din punctaj

Exemple

relief.in relief.out Explicat, ie

1

14

4 5 6 5 5 5 7 8 11 11 10 6 4 4

3 p = 1

Cel mai mic numar ın s, ir este 4, care apare

de 3 ori.

relief.in relief.out Explicat, ie

2

14

4 5 6 5 5 5 7 8 11 11 10 6 4 4

1 1 3 p = 2

Avem muntele:

4 5 6 5

Avem dealul:

5 7 8 11 11 10 6 4

Avem vaile:

5 5 5

11 11

4 4.

Timp maxim de execut, ie: 1 secunda/test.

Memorie totala disponibila 4 MB, din care 2 MB pentru stiva.

Doru Anastasiu Popescu, Pites,ti (Info-Oltenia, 2016)

I 4 (prime). Lui Vladino ıi plac mult jocurile cu piese ce se as,eaza pe table de dimensiune

m × n, precum cele de s,ah. De aceasta data Vladino are tot atatea piese cate patrat,ele are

154 PROBLEME DE INFORMATICA PENTRU CONCURSURI

tabla de s,ah, adica m · n, pe ele aflandu-se numerele 1, 2, . . . ,m · n. Toate piesele sunt as,ezate

pe tabla ın ordine crescatoare pe linii de la stanga la dreapta, ca mai jos:

1 2 . . . n

n+ 1 n+ 2 . . . 2n

. . .

(m− 1)n (m− 1)n+ 1 . . . m · nRobertino vine ın vizita la Vladino s, i ıl pune pe acesta din urma sa t, ina ochii ınchis, i, timp ın

care ia de pe tabla de s,ah trei piese, apoi ıi propune sa scrie un program care sa determine

numerele de pe piesele lipsa, cunoscand suma S a numerelor ramase pe tabla s, i faptul ca acestea

sunt numere prime. Daca exista mai multe solut, ii, se va alege cea care da numarul cel mai mic

prin alaturarea celor trei piese.

Cerint, a

Cunoscand numerele m, n s, i S se cere:

1. numerele de pe piesele luate de Robertino;

2. pozit, ia ın tablou (linie coloana) a celui mai mare numar de pe piesele luate de Robertino.

Date de intrare

Fis, ierul de intrare prime.in cont, ine pe prima linie un numar natural p. Pentru toate testele de

intrare, numarul p poate avea doar valoarea 1 sau 2.

Pe linia a doua se afla m, n si S, separate ıntre ele prin cate un spat, iu.

Date de ies, ire

Daca valoarea lui p este 1, se va rezolva numai punctul 1) din cerint, a.

In acest caz, ın fis, ierul de ies, ire prime.out se vor scrie cele trei numere de pe piesele luate

de Robertino, ın ordine crescatoare.

Daca valoarea lui p este 2, se va rezolva numai punctul 2) din cerint, a.

In acest caz, ın fis, ierul de ies, ire prime.out se va scrie pozit, ia ın tablou (linie coloana) a

celui mai mare numar de pe piesele luate de Robertino.

Restrict, ii s, i precizari

• 1 ≤ m,n ≤ 1000

• 1 ≤ S ≤ 1010

Exemple

prime.in prime.out Explicat, ie

1

3 4 58

2 7 11 p = 1

In tabloul de mai jos numerele de pe piesele

luate de Robertino sunt scrise ıngros,at:

1 2 3 4

5 6 7 8

9 10 11 12

PROBLEME DE INFORMATICA PENTRU CONCURSURI 155

prime.in prime.out Explicat, ie

2

3 4 58

3 3 p = 2

In tabloul de mai jos numerele de pe piesele

luate de Robertino sunt scrise ıngros,at:

1 2 3 4

5 6 7 8

9 10 11 12

Cel mai mare numar de pe o piesa luata de

Robertino este 11, aflat pe linia 3, coloana

3.

Timp maxim de execut, ie: 1 secunda/test.

Memorie totala disponibila 4 MB, din care 2 MB pentru stiva.

Doru Anastasiu Popescu, Pites,ti (Info-Oltenia, 2016)

I 5 (elevi). O s,coala nu are biblioteca s, i de aceea elevii hotarasc sa doneze cart, i pentru a o

ınfiint,a. O mare parte dintre elevii s,colii doneaza cart, i. Pentru ca sa existe o evident, a a cart, ilor

din biblioteca se ia decizia ca fiecare elev care doneaza cart, i sa ıs, i aleaga un numar natural s, i sa

ıl foloseasca la scrierea codurilor pe cart, ile lui.

Codurile de pe cart, i respecta urmatoarele reguli:

1. Codurile scrise de un elev pe cart, ile donate de el sunt toate cifrele numarului ales, eventual

ın alta ordine.

2. Doua cart, i donate de acelas, i elev au coduri diferite.

3. Numerele alese de orice doi elevi au cel put, in o cifra diferita sau o cifra care apare de un

numar diferit de ori.

4. Codurile scrise pe cart, i nu ıncep cu cifra 0.

Dupa ce s-a terminat operat, ia de donat, ie s-au obt, inut n cart, i cu codurile x1, x2, . . . , xn. Se

dores,te sa se cunoasca numarul de cifre folosite la scrierea codurilor pe toate cart, ile s, i numarul

de elevi care au donat cart, i.

Cerint, a

Cunoscand numarul de cart, i n s, i codurile lor x1, x2, . . . , xn, se cere:

1. numarul de cifre folosite la scrierea codurilor pe toate cart, ile;

2. numarul de elevi care au donat cart, i.

Date de intrare

Fis, ierul de intrare elevi.in cont, ine pe prima linie un numar natural p. Pentru toate testele de

intrare, numarul p poate avea doar valoarea 1 sau 2.

Pe linia a doua se afla n, iar pe linia a treia se afla codurile x1, x2, . . . , xn, separate prin cate

un spat, iu.

156 PROBLEME DE INFORMATICA PENTRU CONCURSURI

Date de ies, ire

Daca valoarea lui p este 1, se va rezolva numai punctul 1) din cerint, a.

In acest caz, ın fis, ierul de ies, ire elevi.out se va scrie un singur numar natural, reprezentand

numarul de cifre folosite la scrierea codurilor pe toate cart, ile.

Daca valoarea lui p este 2, se va rezolva numai punctul 2) din cerint, a.

In acest caz, ın fis, ierul de ies, ire elevi.out se va scrie un singur numar natural, reprezentand

numarul de elevi care au donat cart, i.

Restrict, ii s, i precizari

• 1 ≤ n ≤ 10000

• x1, x2, . . . , xn sunt numere naturale cu maxim 9 cifre

• Pentru rezolvarea corecta a primei cerint,e se acorda 20 de puncte, iar pentru cerint,a a

doua se acorda 80 de puncte

Exemple

elevi.in elevi.out Explicat, ie

1

5

121 1204 77 112 4201

16 p = 1

Numerele au ın ordine 3, 4, 2, 3, 4

cifre, ın total 16 cifre.

elevi.in elevi.out Explicat, ie

2

5

121 1204 77 112 4201

3 p = 2

Au donat cart, i trei elevi. Primul

elev a donat cart, ile cu codurile 121,

112, al doilea cart, ile cu codurile

1204, 4201, iar al treilea o carte cu

codul 77.

Timp maxim de execut, ie: 1 secunda/test.

Memorie totala disponibila 4 MB, din care 2 MB pentru stiva.

Violeta Balan, Slatina

Clasa a X-a

I 6 (Scufit,a Ros, ie). Scufit,a Ros, ie pleaca de acasa prin padure s, i trebuie sa ajunga la bu-

nica.Traseul prin padure este plin de obstacole. Ea trebuie sa ajunga pe cel mai scurt traseu la

bunica. Scufit,a Ros, ie s, i-a facut un plan s, i pentru a evita obstacolele a desenat harta padurii

pe un caiet de matematica ın care a colorat, ın patrat,elele caietului, obstacolele cu ros,u iar pe

celelalte patrat,ele le-a lasat libere (necolorate). A observat ca harta are forma de patrat. Pe

PROBLEME DE INFORMATICA PENTRU CONCURSURI 157

harta se cunosc pozit, iile Scufit,ei s, i bunicii. Deplasarile sunt permise doar pe orizontala s, i pe

verticala.

Ajutat, i-o pe Scufit, a sa ajunga pe traseul cel mai scurt de la locul ei la cel al bunicii, ocolind

obstacolele. Harta are latura de n patrat,ele, avand aspectul unei matrice patratice cu n linii

s, i n coloane. Liniile s, i respectiv coloanele sunt numerotate de la 1 la n. Elementele matricei

corespund zonelor patrat,elelor din foaia de caiet. O astfel de zona poate fi colorata cu ros,u sau

cu alb. Scufit,a trebuie sa treaca printr-un numar minim de zone libere.

Cerint, a

Scriet, i un program care sa determine numarul minim de zone libere pentru a alcatui traseul de

la Scufit, a la bunica.

Date de intrare

Fis, ierul de intrare scufita.in cont, ine pe prima linie doua valori naturale n s, i m separate

printr-un spat, iu, reprezentand dimensiunea hart, ii, respectiv numarul de obstacole aflate pe harta.

Fiecare dintre urmatoarele m linii cont, ine cate doua numere naturale x s, i y separate printr-un

spat, iu, reprezentand pozit, iile obstacolelor de pe harta (x reprezinta linia, iar y reprezinta

coloana zonei ın care se afla obstacolul). Ultima linie a fis, ierului cont, ine patru numere naturale

x1, y1, x2, y2, separate prin cate un spat, iu, x1, y1 reprezinta linia respectiv coloana pozit, iei

Scufit,ei, iar x2, y2 reprezinta linia respectiv coloana pozit, iei bunicii.

Date de ies, ire

Fis, ierul de ies, ire scufita.out va cont, ine o singura linie pe care va fi scris un numar natural

care reprezinta numarul minim de zone prin care a trecut Scufit,a.

Restrict, ii s, i precizari

• 1 ≤ n ≤ 175

• 1 ≤ m < n2

• Traseul ıncepe cu zona unde se gases,te Scufit,a s, i se termina cu zona unde se gases,te bunica

• Pentru datele de test exista ıntotdeauna solut, ie

Exemplu

scufita.in scufita.out Explicat, ie

8 5

2 3

3 6

4 4

7 3

7 5

1 1 8 8

15 O modalitate de a constitui traseul este

parcurgerea primei linii, apoi a ultimei

coloane.

Timp maxim de execut, ie: 1 secunda/test.

Memorie totala disponibila 5 MB.

Grat,iela Ghiordunescu, Pites,ti (Dan Barbilian, 2018)

158 PROBLEME DE INFORMATICA PENTRU CONCURSURI

I 7 (parola). Un informatician dores,te sa-l stimuleze pe fiul sau sa foloseasca reguli matematice

pentru a determina o parola. Parola este formata din doua numere ce sunt obt, inute din calcule.

Scopul informaticianului este ca fiul sau sa-s, i ınsus,easca fract, iile zecimale neperiodice, periodice

simple, respectiv periodice mixte s, i sa transforme o fract, ie zecimala ın fract, ie ordinara. Fiul sau

primes,te de la tata fract, ia zecimala, iar el pentru a sparge parola trebuie sa determine doua

numere ce reprezinta numaratorul respectiv numitorul fract, iei ordinare echivalente, cu numar

minim de cifre la numitor, cu numitorul fiind o cifra de 1 urmata eventual de zero-uri sau o

cifra de 9 sau mai multe, urmate eventual de zero-uri.

De exemplu, 1,2(3) =123− 12

90=

111

90, iar 12,500 =

12500

1000=

125

10.

Cerint, a

Scriet, i un program care cites,te o fract, ie zecimala strict pozitiva s, i afis,eaza numaratorul s, i

numitorul unei fract, ii ordinare echivalente constuita conform regulii de mai sus.

Date de intrare

Fis, ierul parola.in cont, ine pe o singura linie un s, ir de maxim 80 de caractere reprezentand

fract, ia zecimala. Caracterele din acest s, ir pot fi: cifre, eventual virgula zecimala ’,’ s, i parantezele

’(’ respectiv ’)’.

Date de ies, ire

Fis, ierul parola.out va cont, ine doua linii. Pe prima linie se va scrie numaratorul fract, iei, pe a

doua linie se va scrie numitorul fract, iei.

Restrict, ii s, i precizari

• Partea ıntreaga a unei fract, ii zecimale este formata din cel put, in o cifra

• S, irul citit poate cont, ine cel mult 77 de cifre s, i reprezinta o fract, ie zecimala corecta

Exemplu

parola.in parola.out Explicat, ie

5,67 567

100 5, 67 =567

100

Timp maxim de execut, ie: 1 secunda/test.

Memorie totala disponibila 5 MB.

Grat,iela Ghiordunescu, Pites,ti (Dan Barbilian, 2018)

I 8 (joc). De Craciun, Ionel a primit cadou un joc de potrivire imagini. Toate piesele au aceleas, i

dimensiuni, forma patratica s, i sunt realizate din lemn de cea mai buna calitate. O piesa se

ımparte ın N × N celule patrate, identice ca dimensiuni, unele celule fiind pline, altele fiind

decupate. Pentru a dezvolta abilitat, ile de coordonare mana-ochi, copilul trebuie sa as,eze piesele

pe un suport tot patratic de dimensiune N ×N . Piesele pot fi as,ezate cu oricare din cele doua

fet,e deasupra. Astfel ele pot fi rotite atat pe orizontala cat s, i pe verticala. Anumite celule de

pe suport sunt prevazute cu tije de lemn verticale, prin care se va ıncerca plasarea pieselor.

PROBLEME DE INFORMATICA PENTRU CONCURSURI 159

Dimensiunile tijelor permit as,ezarea chiar s, i a tuturor pieselor, daca acestea sunt decupate ın

celulele corespunzatoare tijelor. Fiecare piesa este prevazuta cu elemente speciale de sust, inere,

astfel ca daca exista N ×N tije pe toata suprafat,a suportului, iar o piesa este decupata ın toate

celulele patrate, atunci piesa se poate as,eza pe suport. Nu este permisa as,ezarea unei piese care

are mai multe celule decupate decat numarul tijelor. Cum Ionel este prea mic, neavand suficienta

ındemanare de a manevra piesele pentru a le introduce prin tijele de pe suport, ajutat, i-l sa le

as,eze, daca este posibil.

Cerint, a

Scriet, i un program care determina care dintre piese pot fi as,ezate pe suport.

Date de intrare

Fis, ierul de intrare joc.in cont, ine pe prima linie doua numere naturale N s, i P despart, ite

printr-un spat, iu. N este dimensiunea suportului, precum s, i a pieselor, iar P reprezinta numarul

pieselor. Urmeaza P + 1 blocuri de cate N linii fiecare. Primul bloc de N linii codifica suportul.

Valoarea 1 apare daca suportul este prevazut cu tija verticala, iar valoarea 0 ın caz contrar.

Urmatoarele P blocuri de cate N linii codifica fiecare cate o piesa. Pe fiecare linie sunt cate

N valori ıntregi, despart, ite printr-un singur spat, iu, valoarea 1 codifica partea decupata, iar

valoarea 0 partea care nu este decupata.

Date de ies, ire

In fis, ierul de ies, ire joc.out se vor scrie P linii, cate o valoare pe linie. Pe linia i se va scrie

valoarea 1 daca piesa i poate fi as,ezata pe suport astfel ıncat tijele sa patrunda prin port, iunile

decupate ale piesei s, i valoarea 0 ın caz contrar.

Restrict, ii s, i precizari

• 1 < N,C ≤ 50

Exemplu

joc.in joc.out Explicat, ie

3 2

0 1 0

0 0 1

1 0 0

1 0 0

0 0 1

0 1 0

0 0 1

0 0 1

0 1 0

1

0

Piesa 1 se potrives,te perfect suportu-

lui, daca se rotes,te ın sens trigonome-

tric cu 90 de grade.

Piesa 2 nu se potrives,te suportului, in-

diferent de modul ın care se ıncearca

as,ezarea sa prin tijele aflate pe suport.

Timp maxim de execut, ie: 1 secunda/test.

Memorie totala disponibila 5 MB.

Cristina Constantinescu, Pites,ti (Dan Barbilian, 2018)

160 PROBLEME DE INFORMATICA PENTRU CONCURSURI

I 9 (descifrare). Taberele sunt de fiecare data pline de activitat, i care raman mult timp ın

mint, ile copiilor. Timpul petrecut ın compania altor copii ofera ocazia de a socializa, de a

face prieteni noi s, i de a participa la diverse activitat, i. Secretul unei tabere reus, ite este sa fie

organizate mai multe tipuri de activitat, i. In vacant,a de iarna, mai mult, i elevi ai clasei a IV-a

de la s,coala Amicii Istet,i au mers ımpreuna ın tabara. Unul dintre jocurile care le-a trezit

interes a fost cel care le oferea posibilitatea sa descifreze un mesaj. Astfel, ei aveau la dispozit, ie

un dict, ionar format din cuvinte uzuale precum s, i o ıns, iruire de cifre, fara cifra 0. Echipa

cas,tigatoare trebuie sa determine prima mesajul ce se ascunde ın spatele cifrelor, constand

dintr-o succesiune de cuvinte separate prin cate un spat, iu.

Fiecare cifra are asociata un grup de litere. Cifra 2 este asociata literelor A, B sau C, cifra 3

este asociata literelor D, E s, i F, cifra 4 este asociata literelor G, H s, i I, cifra 5 este asociata

literelor J, K s, i L, cifra 6 este asociata literelor M, N s, i O, cifra 7 este asociata literelor P, Q, R

s, i S, cifra 8 este asociata literelor T, U s, i V, cifra 9 este asociata literelor W, X, Y s, i Z, iar cifra

1 este asociata tastei Spat,iu. Prin urmare, daca mesajul este format din cifrele 2 8 1, textul

ascuns poate fi A T Spat,iu sau A U Spat,iu, etc, cuvintele corespunzatoare s, irurilor de cifre

putand fi: AT, AU, etc.

Pentru un posibil text ascuns obt, inut prin descifrarea consecutiva a cifrelor mesajului, se

opresc doar cuvintele care exista ın dictionar, iar fiecare litera a unui cuvant care nu este ın

dictionar va fi ınlocuita cu un caracter *. Se vor verifica toate posibilitat, ile de formare a unui

cuvant conform regulilor de mai sus. Daca exista mai multe cuvinte care corespund unei anumite

secvent,e de cifre, va fi afis,at primul.

Cerint, a

Scriet, i un program care sa descifreze mesajul transmis.

Date de intrare

Fis, ierul de intrare desc.in cont, ine:

M - numarul de cuvinte din dict, ionar;

C1

C2

. . .

CM - C1, C2, . . . , CM reprezinta cuvintele din dict, ionar;

N - numarul de cifre din mesajul ce urmeaza a fi decriptat;

cif1 cif2 . . . cifN - cifrele ce urmeaza a fi decriptate.

Date de ies, ire

Fis, ierul de iesire desc.out cont, ine pe o singura linie mesajul generat respectand regulile date.

Restrict, ii s, i precizari

• 1 ≤M,N ≤ 100

• cifi ∈ 1, 2, . . . , 9, oricare i ∈ 1, 2, . . . , N

PROBLEME DE INFORMATICA PENTRU CONCURSURI 161

• Cuvintele C1, C2, . . . , CM sunt ordonate alfabetic s, i sunt scrise cu majuscule, formate doar

din litere ale alfabetului englez

• Lungimea oricarui cuvant este de maxim 100 de litere

Exemplu

desc.in desc.out Explicat, ie

5

ACASA

ANA

LA

MERGE

SCOALA

11

2 1 2 3 1 2 2 2 7 2 1

* ** ACASA Pentru cele 5 cuvinte existente ın

dict, ionar, mesajul ascuns ın spatele

cifrelor se afla ın fis, ierul de ies, ire.

Timp maxim de execut, ie: 1 secunda/test.

Memorie totala disponibila 5 MB.Cristina Constantinescu, Pites,ti (Dan Barbilian, 2018)

I 10 (cecuatii). Ionica s, i Tica sunt buni prieteni, pasionat, i de doua lucruri: matematica s, i

informatica. Aces,tia participa la un concurs de propunatori de probleme pe echipe. Pentru a

avea succes, Tica vrea sa propuna o problema cu ecuat, ii, iar Ionica o problema cu mult, imi. Dupa

consultari repetate se decid sa participe la concurs cu o problema care foloses,te atat ecuat, ii cat s, i

mult, imi. In aceasta problema se dau N ecuat, ii de forma Aix+Biy +Ciz = Di, 1 ≤ i ≤ N s, i se

noteaza cu Si mult, imea x+ y + z|Aix+Biy + Ciz = Di, x, y, z numere naturale, 1 ≤ i ≤ N .

Cerint,a problemei este determinarea a doua numere naturale N1 s, i N2, unde N1 = Max+FMax,

respectiv N2 = card(S1∪S2 . . .∪SN ). Max este cel mai mare numar dintre card(Si), 1 ≤ i ≤ N ,

iar FMax este de cate ori apare Max printre card(S1), card(S2), . . . , card(SN).

Cerint, a

Se cunosc N s, i numerele A1, B1, C1, D1, A2, B2, C2, D2, . . . , AN , BN , CN , DN s, i se cere sa se

determine numerele N1 s, i N2.

Date de intrare

Din fis, ierul cecuatii.in se citesc: numarul natural p de pe prima linie, ce poate avea doua

valori: 1 sau 2, corespunzatoare uneia din cele doua cerint,e, adica 1 pentru determinarea lui N1

s, i 2 pentru determinarea lui N2; numarul N de pe a doua linie s, i, de pe urmatoarele N linii,

cate patru numere naturale separate prin cate un spat, iu reprezentand coeficient, ii ecuat, iilor:

Ai, Bi, Ci, Di, 1 ≤ i ≤ N .

Date de ies, ire

In fis, ierul cecuatii.out se va scrie numarul N1, daca p = 1 s, i respectiv numarul N2, daca

p = 2, cu semnificat, ia din enunt, .

Restrict, ii s, i precizari

• 1 < N ≤ 1000, numar natural

162 PROBLEME DE INFORMATICA PENTRU CONCURSURI

• Ai, Bi, Ci, Di sunt numere naturale nenule mai mici strict decat 1000, 1 ≤ i ≤ N

• Pentru o mult, ime M , card(M) reprezinta numarul de elemente ale mult, imii M

• Pentru mult, imile M s, i P , notam cu M ∪ P reuniunea dintre M s, i P

Exemple

cecuatii.in cecuatii.out Explicat, ie

1

2

1 1 1 1

1 2 1 2

3 p = 1 s, i deci afis, am N1. Prima ecuat, ie este

x + y + z = 1, iar a doua x + 2y + z = 2.

Obt, inem solut, iile (1,0,0), (0,1,0), (0,0,1)

pentru prima ecuat, ie s, i (1,0,1), (0,1,0),

(0,0,2), (2,0,0) pentru a doua ecuat, ie. S1 =

1, S2 = 1, 2, Max = 2, FMax = 1,

deci N1 = 3.

cecuatii.in cecuatii.out Explicat, ie

2

2

1 1 1 1

1 2 1 2

2 p = 2 s, i deci afis, am N2. Prima ecuat, ie este

x + y + z = 1, iar a doua x + 2y + z = 2.

Obt, inem solut, iile (1,0,0), (0,1,0), (0,0,1)

pentru prima ecuat, ie s, i (1,0,1), (0,1,0),

(0,0,2), (2,0,0) pentru a doua ecuat, ie. S1 =

1, S2 = 1, 2, S1 ∪ S2 = 1, 2, deci

N2 = 2.

Timp maxim de execut, ie: 0.04 secunde/test.

Memorie totala disponibila 4 MB.

Doru Anastasiu Popescu, Pites,ti (Micul Gates, 2017)

Clasele a XI-a s, i a XII-a

I 11 (arici). Intr-o livada de forma dreptunghiulara exista doi arici care doresc sa ajunga la

casut,a aricilor. In livada sunt m randuri de pomi, iar pe fiecare din cele m randuri exista n

pomi la o distant, a de 1 dam. La baza fiecarui pom exista un numar de mere (un numar natural

de exact doua cifre).

Fiecare dintre cei doi arici dores,te sa ajunga cat mai repede la casut,a aricilor s, i astfel se

organizeaza o competit, ie ıntre ei, cas,tigatorul fiind ariciul care ajunge pe drumul cel mai scurt,

drum masurat ın distant,a parcursa de arici ın dam de la pozit, ia lui din livada la casut,a aricilor.

Fiecare din cei doi arici are un cod de acces, un numar natural de exact 5 cifre nenule. Pe baza

acestui cod aricii au sau nu acces la merele cazute din pom. Un arici are acces la merele de la

baza unui pom daca numarul de mere cazute are cel put, in o cifra care se gases,te ın codul de

acces al ariciului. Fiecare arici se gases,te init, ial langa un anumit pom din livada. Aricii se pot

PROBLEME DE INFORMATICA PENTRU CONCURSURI 163

deplasa prin livada, de la un pom la fiecare din cei cel mult 4 pomi vecini la care au acces, doar

pe direct, iile paralele cu laturile livezii.

Cerint, a

Cunoscand dimensiunile livezii m s, i n, numarul de mere cazute din fiecare din cei m · n pomi,

coordonatele L, C unde se afla casut,a aricilor, coordonatele celor doi arici s, i codurile lor de

acces, determinat, i:

1. numarul X de pomi la care au acces ambii arici;

2. numarul Nr al ariciului cas,tigator (1 sau 2, corespunzator cu numarul de ordine al

ariciului), distant,a minima Dmin pe care o parcurge ariciul cas,tigator pentru a ajunge la casut,a

aricilor s, i numarul NrM de pomi de unde ar putea pleca ariciul cas,tigator astfel ıncat sa

parcurga aceeas, i distant, a minima pana la casut,a aricilor.

Date de intrare

Fis, ierul de intrare arici.in cont, ine:

- pe prima linie un numar natural p; pentru toate testele de intrare, numarul p poate avea

doar valoarea 1 sau 2;

- pe a doua linie doua numere naturale nenule m,n, separate prin cate un spat, iu, cu

semnificat, iile din enunt, ;

- pe linia a treia doua numere naturale nenule L,C, separate prin cate un spat, iu, cu

semnificat, iile din enunt, ;

- pe liniile a patra s, i a cincea cate trei numere naturale reprezentand coordonatele pomilor

unde se gasesc init, ial cei doi arici (linia, coloana) s, i codul de acces al aricilor: x1, y1, c1, respectiv

x2, y2, c2;

- pe urmatoarele m linii cate n numere naturale de exact doua cifre, separate prin cate un

spat, iu, reprezentand numarul de mere cazute din fiecare pom al livezii.

Date de ies, ire

Daca valoarea lui p este 1, se va rezolva numai punctul 1) din cerint, a.

In acest caz, ın fis, ierul de ies, ire arici.out se va scrie un singur numar natural, X, repre-

zentand numarul total de pomi la care au acces ambii arici.

Daca valoarea lui p este 2, se va rezolva numai punctul 2) din cerint, a.

In acest caz, fis, ierul de ies, ire arici.out va cont, ine numerele naturale Nr, Dmin s, i NrM ,

ın aceasta ordine, separate prin cate un spat, iu.

Restrict, ii s, i precizari

• 2 ≤ m,n, L, C, x1, y1, x2, y2 ≤ 300

• 11111 ≤ c1, c2 ≤ 99999

• 10 ≤ numarul de mere cazute din fiecare pom ≤ 99

164 PROBLEME DE INFORMATICA PENTRU CONCURSURI

• Pentru toate testele problemei exista solut, ie, adica exista cel put, in un arici care poate

ajunge la casut,a aricilor

• Daca cei doi arici parcurg aceeas, i distant, a pana la casut,a aricilor, este considerat cas,tigator

primul arici

• Mere cazute din pom se pot gasi s, i ın locuri la care aricii nu au acces

• Pentru rezolvarea corecta a primei cerint,e se acorda 20% din punctaj, iar pentru cerint,a a

doua 80% din punctaj

• Pentru primele 30% dintre testele care verifica cerint,a 2) avem m ≤ 20 s, i n ≤ 20

• Pentru determinarea corecta a numerelor Nr s, i Dmin se acorda 80% din punctajul testului,

iar pentru determinarea corecta a numarului NrM se acorda 20% din punctaj

Exemple

arici.in arici.out Explicat, ie

1

5 6

3 5

1 2 12351

5 1 46123

15 12 37 12 78 19

16 17 18 19 25 20

21 25 26 89 25 25

24 78 29 31 24 89

23 25 46 25 96 25

24 p = 1

Exista 24 de pomi ın livada la care au

acces ambii arici, s, i anume tot, i pomii cu

except, ia celor 6 pomi care au numarul de

mere cazute 46, 78, 89 sau 96.

arici.in arici.out Explicat, ie

2

5 6

3 5

1 2 12351

5 1 46123

15 12 37 12 78 19

16 17 18 19 25 20

21 25 26 89 25 25

24 78 29 31 24 89

23 25 46 25 96 25

1 5 3 p = 2

Ariciul cu numarul de ordine 1 se afla la

pomul de coordonate (1,2) s, i poate ajunge

la casut,a aricilor pe traseul:

(1,2),(1,3),(1,4),(2,4),(2,5),(3,5),

parcurgand o distant, a de 5 dam.

De asemenea mai exista s, i alt, i doi pomi de

la care ar putea pleca ariciul 1 s, i ar parcurge

aceeas, i distant, a de 5 dam pana la casut,a

aricilor, s, i anume pomii de coordonate (2,1)

s, i (3,2).

Ariciul 2 poate ajunge la casut,a aricilor

parcurgand o distant, a de 6 dam, pe traseul:

(5,1),(5,2),(5,3),(5,4),(5,5),(4,5),(3,5).

Timp maxim de execut, ie: 0.5 secunde/test.

Memorie totala disponibila 8 MB, din care 2 MB pentru stiva.

Adriana Oprea, Pites,ti

PROBLEME DE INFORMATICA PENTRU CONCURSURI 165

I 12 (cuvinte). Se considera doua cuvinte formate numai din litere mici ale alfabetului englez,

notate cu c1 si c2. Pentru aceste cuvinte se dores,te sa se rezolve doua cerint,e.

Prima cerint, a este legata de verificarea condit, iei de subs, ir de caractere a lui c2 ın c1, adica

daca se pot s,terge caractere din c1 astfel ıncat sa se obt, ina c2. Daca c2 este subs, ir ın c1 se dores,te

s, irul de pozit, ii pe care se afla, fara spat, ii. Numarul obt, inut se va numi POZ. De exemplu

pentru c1 =ababcd s, i c2 =abd, raspunsul este afirmativ, POZ putand fi 126 sau 346. Pozit, iile

ın cuvinte sunt 1, 2, 3, . . . .

A doua cerint, a este sa se construiasca un nou cuvant c3 cu litere distincte sau cu o singura

litera care sa se repete de doua ori, astfel ıncat sa respecte regulile:

1. literele din c3 trebuie sa apara ın c1 sau c2;

2. litera care se repeta trebuie sa apara ın total de cel put, in doua ori ın c1 s, i c2.

Cerint, a

Cunoscand cele doua cuvinte c1 s, i c2 se cere:

1. sa se verifice daca c2 este subcuvant ın c1 s, i sa se determine cel mai mic numar POZ, ın

caz afirmativ, altfel se va afis,a cifra 0;

2. numarul de cuvinte c3 distincte ce se pot construi cu regulile din enunt, , de la a doua

cerint, a.

Date de intrare

Fis, ierul de intrare cuvinte.in cont, ine pe prima linie un numar natural p. Pentru toate testele

de intrare, numarul p poate avea doar valoarea 1 sau 2.

Pe linia a doua se afla cuvantul c1, iar pe linia a treia cuvantul c2.

Date de ies, ire

Daca valoarea lui p este 1, se va rezolva numai punctul 1) din cerint, a.

In acest caz, ın fis, ierul de ies, ire cuvinte.out se va scrie rezultatul de la cerint,a 1, adica 0

sau cel mai mic POZ cu semnificat, ia din enut, .

Daca valoarea lui p este 2, se va rezolva numai punctul 2) din cerint, a.

In acest caz, ın fis, ierul de ies, ire cuvinte.out se va scrie numarul de la cerint,a 2, modulo

9901.

Restrict, ii s, i precizari

• 1 ≤ numar caractere din c1 ≤ 250,

• 1 ≤ numar caractere din c2 ≤ 250

• Pentru rezolvarea corecta a cerint,ei 1 se acorda 20% din punctaj

• k modulo p reprezinta restul ımpart, irii ıntregi a lui k la p

166 PROBLEME DE INFORMATICA PENTRU CONCURSURI

Exemple

cuvinte.in cuvinte.out Explicat, ie

1

ababcd

abd

126 p = 1

POZ poate fi 126 sau 346, iar

126 este cel mai mic. Pen-

tru POZ=126, ınseamna ca se

pastreaza din c1 literele de pe

pozit, iile 1, 2, 6, se s,terg cele de

pe pozit, iile 3, 4, 5 s, i se obt, ine c2.

cuvinte.in cuvinte.out Explicat, ie

2

adcd

ad

29 p = 2

Cuvintele ce se pot forma cu res-

pectarea regulilor din enunt, sunt:

a, d, c,

ad, da, ac, ca, dc, cd, aa, dd,

dac, dca, adc, acd, cda, cad,

daa, ada, aad, caa, aca, aac,

cdd, dcd, ddc, add, dad, dda.

Timp maxim de execut, ie: 1 secunda/test.

Memorie totala disponibila 4 MB, din care 2 MB pentru stiva.

Doru Anastasiu Popescu, Pites,ti (Info-Oltenia, 2016)

I 13 (litere). In fiecare patrat,el al unei table de s,ah de dimensiune m× n se afla cate o piesa

pe care este scrisa o litera mica din alfabetul englez. Georgica dores,te sa-s, i verifice cunos,tint,ele

de informatica folosind tabla de s,ah s, i piesele. Astfel el dores,te sa determine:

1. numarul de cuvinte Nr de lungime k ce se pot forma, cu litere distincte s, i ın ordine

alfabetica, folosind piese de pe tabla de s,ah;

2. lungimea minima L a unui cuvant format cu piese vecine de pe tabla de s,ah ce cont, in

numai vocale, astfel ıncat sa cont, ina o piesa de pe prima linie s, i o piesa de pe ultima linie;

doua piese sunt vecine, daca au o latura comuna.

Cerint, a

Cunoscand numerele m,n, k s, i piesele de pe tabla de s,ah se cere:

1. numarul Nr cu semnificat, ia de mai sus;

2. numarul L cu semnificat, ia de mai sus.

Date de intrare

Fis, ierul de intrare litere.in cont, ine pe prima linie un numar natural p. Pentru toate testele

de intrare, numarul p poate avea doar valoarea 1 sau 2.

Pe linia a doua se afla m, n s, i k, separate prin cate un spat, iu, iar pe urmatoarele m linii

literele tabloului, fara spat, iu ıntre ele.

PROBLEME DE INFORMATICA PENTRU CONCURSURI 167

Date de ies, ire

Daca valoarea lui p este 1, se va rezolva numai punctul 1) din cerint, a.

In acest caz, ın fis, ierul de ies, ire litere.out se va scrie numarul Nr.

Daca valoarea lui p este 2, se va rezolva numai punctul 2) din cerint, a.

In acest caz, ın fis, ierul de ies, ire litere.out se va scrie numarul L.

Restrict, ii s, i precizari

• 1 ≤ m,n ≤ 100

• 1 ≤ k ≤ 26

• Pentru rezolvarea corecta a cerint,ei 1 se acorda 40% din punctaj

Exemple

litere.in litere.out Explicat, ie

1

3 4 2

abab

eaaa

bbbe

3 p = 1

Se pot forma 3 cuvinte:

ab, ae, be

litere.in litere.out Explicat, ie

2

3 4 2

abab

eaaa

bbbe

4 p = 2

Se pot forma doua cuvinte

(ıngros,ate mai jos):

abab

eaaa

bbbe

respectiv:

abab

eaaa

bbbe

Cel mai scurt, aaae, are 4 litere.

Timp maxim de execut, ie: 1 secunda/test.

Memorie totala disponibila 4 MB, din care 2 MB pentru stiva.

Doru Anastasiu Popescu, Pites,ti (Info-Oltenia, 2016)

I 14 (veverit,a). Intr-un parc din oras, se afla n pomi, pentru care se cunosc coordonatele

carteziene pe harta oras,ului. Pomii sunt numerotat, i cu 1, 2, . . . , n. In parc se mai afla o veverit, a

foarte isteat, a, dar care poate sari din pom ın pom doar daca distant,a dintre aces,tia este cel mult

168 PROBLEME DE INFORMATICA PENTRU CONCURSURI

egala cu k. In cazul ın care veverit,a vrea sa se deplaseze de la un pom la altul mai ındepartat,

aceasta trebuie sa ris,te sa fie alergata de caini.

Veverit,a dores,te sa afle:

- de cate ori trebuie sa ris,te sa fie alergata de caini pentru a se deplasa ın fiecare pom;

- distant,a cea mai mica pe care trebuie sa o parcurga pe jos pentru a se deplasa ın tot, i pomii

din parc.

Cerint, a

Cunoscand numerele n, k s, i coordonatele pomilor, determinat, i:

1. de cate ori trebuie sa ris,te veverit,a sa fie alergata de caini pentru a se deplasa ın fiecare

pom;

2. distant,a cea mai mica pe care trebuie sa o parcurga pe jos veverit,a pentru a se deplasa ın

tot, i pomii din parc.

Date de intrare

Fis, ierul de intrare veverita.in cont, ine pe prima linie un numar natural p. Pentru toate testele

de intrare, numarul p poate avea doar valoarea 1 sau 2.

Pe linia a doua se afla n si k, separate printr-un spat, iu, iar pe urmatoarele n linii perechi de

numere naturale de forma x y, ce reprezinta coordonatele pomilor.

Date de ies, ire

Daca valoarea lui p este 1, se va rezolva numai punctul 1) din cerint, a.

In acest caz, ın fis, ierul de ies, ire veverita.out se va scrie un singur numar natural repre-

zentand de cate ori trebuie sa ris,te sa fie alergata de caini veverit,a pentru a se deplasa ın fiecare

pom.

Daca valoarea lui p este 2, se va rezolva numai punctul 2) din cerint, a.

In acest caz, ın fis, ierul de ies, ire veverita.out se va scrie distant,a cea mai mica pe care

trebuie sa o parcurga pe jos veverit,a pentru a se deplasa ın tot, i pomii din parc.

Restrict, ii s, i precizari

• 1 ≤ n ≤ 500

• Coordonatele punctelor sunt numere naturale mai mici sau egale cu 1000000

• Pentru rezolvarea corecta a cerint,ei 1 se acorda 20% din punctaj

• 20% din testele cu p = 2 au n ≤ 20

• Pentru distant,a dintre doua puncte se va folosi distant,a Manhattan, adica distant,a dintre

P (x1, y1) s, i Q(x2, y2) este

|x1 − x2|+ |y1 − y2|

PROBLEME DE INFORMATICA PENTRU CONCURSURI 169

Exemple

veverita.in veverita.out Explicat, ie

1

5 8

5 12

100 14

2 8

100 8

10 8

1 p = 1

Veverit,a poate sa sara ıntre pomii:

1 s, i 3, 3 s, i 5, 2 s, i 4.

Trebuie sa ris,te sa mearga pe jos

o singura data ıntre doi pomi.

veverita.in veverita.out Explicat, ie

2

5 8

5 12

100 14

2 8

100 8

10 8

90 p = 2

Veverit,a poate sa sara ıntre pomii:

1 s, i 3, 3 s, i 5, 2 s, i 4.

Trebuie sa mearga pe jos ıntre po-

mii 4 s, i 5 pe o distant, a egala cu

90.

Timp maxim de execut, ie: 1 secunda/test.

Memorie totala disponibila 4 MB, din care 2 MB pentru stiva.

Doru Anastasiu Popescu, Pites,ti, Gabriel Nicolae, Bucures,ti (Info-Oltenia, 2016)

I 15 (case). Cu mult timp ın urma, mai multe case s-au construit fara un plan urbanistic,

formand o comuna. In zilele noastre, primarul comunei vrea sa ımparta comuna ın sate pentru

a gestiona mai us,or problemele comunitat, ii. Pentru a simplifica lucrurile, primarul dores,te

ca satele sa aiba suprafat,a patratica cu laturile paralele cu axele hart, ii. Primarul cunoas,te

coordonatele caselor, casele fiind numerotate cu 1, 2, . . . , n, coordonatele colt,urilor din stanga-jos

s, i laturile suprafet,elor celor k sate. Satele sunt numerotate cu 1, 2, . . . , k. Suprafet,ele satelor

sunt disjuncte. Din pacate se poate ıntampla ca unele case sa fie ın afara satelor.

Primarul dores,te sa acceseze fonduri europene pentru un sistem de canalizare pe care sa-l

dezvolte ulterior. Pentru acest lucru ıntocmes,te un plan fiecarui sat, ın care traseaza segmente

ıntre anumite case astfel ıncat sa se formeze un poligon convex; pe acolo va trece sistemul de

canalizare, astfel ıncat toate casele satului care nu sunt legate prin aceste segmente sa fie ın

interiorul poligonului.

Cerint, a

Cunoscand numerele n, k, coordonatele caselor s, i datele suprafet,elor satelor (coordonatele

colt,ului stanga-jos s, i lungimea suprafet,ei), determinat, i:

1. numarul de case care nu fac parte din niciun sat;

2. numarul de case ce vor face parte din sistemul de canalizare al fiecarui sat i, i = 1, 2, . . . , k.

170 PROBLEME DE INFORMATICA PENTRU CONCURSURI

Date de intrare

Fis, ierul de intrare case.in cont, ine pe prima linie un numar natural p. Pentru toate testele de

intrare, numarul p poate avea doar valoarea 1 sau 2.

Pe linia a doua se afla n s, i k, pe linia a treia se afla n perechi de numere, una dupa alta,

separate prin cate un spat, iu, perechi de forma abscisa ordonata, reprezentand coordonatele

caselor, iar pe linia a patra se afla k triplete de forma abscisa ordonata lungime, separate ıntre

ele prin spat, ii, reprezentand coordonatele colt,ului din stanga-jos s, i lungimea suprafet,ei satelor.

Date de ies, ire

Daca valoarea lui p este 1, se va rezolva numai punctul 1) din cerint, a.

In acest caz, ın fis, ierul de ies, ire case.out se va scrie un singur numar natural reprezentand

numarul de case care nu fac parte din niciun sat.

Daca valoarea lui p este 2, se va rezolva numai punctul 2) din cerint, a.

In acest caz, ın fis, ierul de ies, ire case.out se vor scrie, separate prin cate un spat, iu, numerele

de case ce vor face parte din sistemul de canalizare al fiecarui sat i, i = 1, 2, . . . , k.

Restrict, ii s, i precizari

• 1 ≤ n ≤ 1000

• 1 ≤ k ≤ 10

• Pentru rezolvarea corecta a cerint,ei 1 se acorda 20% din punctaj

• Coordonatele caselor sunt numere naturale mai mici sau egale cu 109

• O casa daca se gases,te pe un segment al sistemului de canalizare ınseamna ca face parte

din acest sistem

• O casa aflata chiar pe frontiera satului se presupune ca face parte din sat

• Satele sunt disjuncte, adica nu au interior sau frontiera comuna

Exemple

case.in case.out Explicat, ie

1

7 1

6 10 8 0 20 0 2 5 4 5 4 0 6 0

2 0 10

1 p = 1

Avem casele cu coordonatele:

C1(6, 10), C2(8, 0), C3(20, 0),

C4(2, 5), C5(4, 5), C6(4, 0), C7(6, 0)

s, i un singur sat cu suprafat,a

patratica avand colt,ul din stanga-

jos de coordonate (2, 0) s, i latura de

lungime 10.

C3 este singura casa care nu face

parte din sat.

PROBLEME DE INFORMATICA PENTRU CONCURSURI 171

case.in case.out Explicat, ie

2

7 1

6 10 8 0 20 0 2 5 4 5 4 0 6 0

2 0 10

5 p = 2

Avem casele cu coordonatele:

C1(6, 10), C2(8, 0), C3(20, 0),

C4(2, 5), C5(4, 5), C6(4, 0), C7(6, 0)

s, i un singur sat cu suprafat,a

patratica avand colt,ul din stanga-

jos de coordonate (2, 0) s, i latura de

lungime 10.

Casele C1, C2, C4, C6, C7 vor face

parte din sistemul de canalizare.

Timp maxim de execut, ie: 1 secunda/test.

Memorie totala disponibila 4 MB, din care 2 MB pentru stiva.

Doru Anastasiu Popescu, Pites,ti, Gabriel Nicolae, Bucures,ti (Info-Oltenia, 2016)

ISTORIOARE DIN LUMEA MATEMATICII S, I A

INFORMATICII

Formula de alungare a lui L. Euler

Stelian Corneliu Andronescu 1

Marele matematician Leonhard Euler ıs, i cas,tigase reputat, ia de a rezolva orice problema cu

care era confruntat.

In timpul retragerii sale la curtea Ecaterinei cea Mare, el l-a cunoscut pe marele filosof

francez Denis Diderot. Diderot era un ateu convins s, i avea sa-s, i petreaca viat,a convertindu-i pe

rus, i la ateism. Infuriata, Ecaterina i-a cerut lui Euler sa puna capat eforturilor acestui francez

fara Dumnezeu. Euler s-a gandit o vreme la aceasta problema s, i a pretins ca are o dovada

algebrica a existent,ei lui Dumnezeu. Ecaterina cea Mare i-a invitat pe Euler s, i pe Diderot la

palat s, i s, i-a adunat curtenii spre a asculta dezbaterea teologica. Euler a aparut ın fat,a publicului

s, i a spus:

,,Domnule,a+ bn

n= x, deci Dumnezeu exista. Raspundet, i!”

Fara a ınt,elege mare lucru din algebra, Diderot n-a fost ın stare sa-i replice lui Euler s, i a

ramas fara grai. Umilit, a parasit St. Petersburg-ul s, i s-a ıntors la Paris.

L. Euler (15.04.1707 − 18.09.1783) a fost cel mai prolific s, i unul dintre cei mai mari ma-

tematicieni ai tuturor timpurilor, dominand cu autoritate matematica secolului al XVIII-lea.

Marele savant, a reunit o intuit, ie formidabila cu o imaginat, ie creatoare except, ionala, o memorie

inegalabila, abilitat, i de calcul extraordinare s, i o putere de munca fantastica.

Opera lui L. Euler a impulsionat puternic ıntreaga dezvoltare a matematicii.

Bibliografie

[1] S. Singh, Marea Teorema a lui Fermat, Ed. Humanitas, Bucures,ti, 1998.

[2] P. Minut, , 300 de ani de la nas, terea lui Leonhard Euler (1707-1783), Recreat, ii Matematice,

nr. 1/2007.

1Lect. univ. dr., Universitatea din Pites,ti, [email protected]

172

Revista sponsorizata de

ROWEB Development SRL

s, i

OSF Global Services SRL